Is Nuclear Power Worth the Risk?

One of the interesting features during the Fukushima reactor crisis were the fistfights that broke out in comments between the defenders of nuclear power and the opponents. The boosters argued that the worst case scenario problems were overblown, both in terms of estimation of the odds of occurrence and the likely consequences. The critics contended that nuclear power was not economical ex massive subsidies, that there was no “safe” method of waste disposal, and that nuclear plants were always subject to corners-cutting, both in design and operation, so the ongoing hazards were greater than they appeared.

Reader Crocodile Chuck passed along a story from the Bulletin of Atomic Scientists, “The Lessons of Fukushima“, by anthropologist Hugh Gusterson. Here is the key section:

And presumably there are other complicated technological scenarios that we have not foreseen, earthquake faults that are undetected or underestimated, and terrorists hatching plans for mayhem as yet unknown. Not to mention regulators who place too much trust in those they regulate.

Thus it is hard to resist the conclusion reached by sociologist Charles Perrow in his book Normal Accidents: Living with High-Risk Technologies: Nuclear reactors are such inherently complex, tightly coupled systems that, in rare, emergency situations, cascading interactions will unfold very rapidly in such a way that human operators will be unable to predict and master them. To this anthropologist, then, the lesson of Fukushima is not that we now know what we need to know to design the perfectly safe reactor, but that the perfectly safe reactor is always just around the corner. It is technoscientific hubris to think otherwise.

This leaves us with a choice between walking back from a technology that we decide is too dangerous or normalizing the risks of nuclear energy and accepting that an occasional Fukushima is the price we have to pay for a world with less carbon dioxide. It is wishful thinking to believe there is a third choice of nuclear energy without nuclear accidents.

Readers will correctly argue that other energy sources have considerable human and environmental costs. Coal fired electrical plants are major CO2 emitters, and the older ones also spew a lot of particulates. Many communities in the US are fighting fracking out of well warranted concerns about the damage it might do to underground water supplies. Others readers have contended that we need to get over our growth addiction and start adapting to less energy intensive lifestyles (which if we were really serious about it, means much more urbanization in the US).

Is nuclear power worth the risk? And if you argue against it, what energy/economic strategy do you recommend in its place?

Print Friendly, PDF & Email

349 comments

  1. bob goodwin

    I am going to vote for nuclear. Lots of jets crashed until they figured out how to do it right. As an engineer I believe we can eventually make it reliable and safe. But we are not there yet. I think when you measure the cost of nuclear energy in human life and evironmental damage it does well compared to fossil fuels.

    I will further venture to say that this accident will do less harm to public opinion than it should. Many people I know have immersed themselves in this issue and have not become more anti nuclear (to my surprise).

    I am appalled at some of the practices that are coming to light. But I remember the cars we made in the 70’s too (not to mention the computers). We must do far, far, better.

    1. Hugh

      Your analogy is a poor one. A jet crash does not put hundreds of thousands, even millions of people at risk. It does not have the capacity to inflict significant damage to a large industrial economy. It does not render large areas unfit for human habitation for a hundred plus years.

      Nor am I sure what you mean by getting it right. Jets still crash.

      Nothing you said addresses the issues of the industry’s subsidies or what to do with the nuclear wastes produced by both a plant’s operation and decommissioning/dismantling.

      1. Pete

        I thought the analogue was fine. A good example is the stress fractures that caused commercial airliners to crash in the 50s. Re stated the point is we are always working the kinks. Your irritation seems misplaced.

        1. DownSouth

          Oh geeze! In Pete we have another of these numerical illiterates who doesn’t know the difference between a hundred or a thousand or a million or a billion.

          Chernobyl-Related Cancer Estimates:

          • Intl. Agency for Research on Cancer—-16,000 deaths
          • Russian Academy of Sciences—-200,000 deaths
          • Ukrainian National commission for Radiation Protection—-500,000 deaths

          The Guardian, January 10, 2010

          And so the “analogue” between a jet crash that kills a couple of hundred people and a nuclear disaster that kills thousands or even hundreds of thousands of people is “just fine.”

          Go figure.

          1. Fifi

            Oh geeze! Oh geeze! In DownSouth we have another of these numerical illiterates who pick their sources to fit their conclusions, while ignoring any notion of epidemiology.

            http://www.who.int/mediacentre/factsheets/fs303/en/index.html

            The WHO concluded to about 4,000 man-life excess mortality by cancer and to another 4,000 excess thyroid cancers caused by iodine-131 exposure from Chernobyl, most of them cured. Everything else amounts to statistical noise within normal mortality.

          2. DownSouth

            Fifi,

            So you handpick a source tailor made to bolster your case, and then turn right around and blast me as being a a “numerical illiterate who picks his sources to fit his conclusions?”

            And on top of that, even the source you cite doesn’t make the case you want it to. 4,000 deaths plus 4,000 other cancers still adds up to 8,000, which is still more than an order of magnitude greater than the 200 or so that might be killed in a plane crash. But of course most of the thyroid cancers were “cured,” so they don’t count. I suppose that’s your reasoning. But even if you leave them out, you’re still talking more than an order of magnitude difference, so the numbers are hardly “analagous.”

            So not only is your comment loaded with hypocrisy, it also exhibits the same ignorance of numbers that Pete’s did.

          3. metcalfe slaw


            go figure

            Nuclear power is merely an integral part of natures master plan. Here’s how it works. When boys and girls forget to use the saran-wrap before they hang together, not to worry. Nuclear cancer gets the job done.

            Let it be!

            Let it be
            !

          4. Fifi

            @DownSouth

            4,000 deaths plus 4,000 other cancers still adds up to 8,000 what? 8,000 dead ? Well, no. That’s the point. That’s still 4,000 excess mortality.

            Then you trumpet 500,000 dead from the report of the Ukrainian National commission for Radiation Protection. Their report is a fascinating read in obtuseness applied to epidemiology.

            They essentially look around at everything which looks wrong then assign it to Chernobyl. They also conveniently forget everything about reporting bias aka “If you look for something, you’ll find it”. So if you are actively looking for cancers in an area, you’ll find a lot of them that would have gone unreported otherwise. Which is why having a control group is so important.

            On this matter, the WHO report is really interesting and illuminating. When looking at stillbirths and congenital malformations, they found a slight but noticeable increase in areas affected by the Chernobyl accident. But then, they also found the same increase in uncontaminated areas of Belarus where the same diagnosis effort was underway as in contaminated areas. In the end, it turns out that all the increase was entirely attributable to reporting bias and the accident itself had had no impact.

          5. Fifi

            Oh, and calling the World Health Organization report a hand picked reference to bolster my case is pretty amusing. It’s a little like climate-change deniers calling the IPCC a “hand picked reference to bolster the case for so so-called global warming”.

            Well, sorry, the WHO is to epidemiology what the IPCC is to climate monitoring, the top-level authority in its domain.

          6. Maju

            There is no such thing as “uncontaminated areas of Belarus”. Actually, since Chernobyl you really have to look hard to find uncontaminated areas in much of Europe: for example Germany is still polluted, enough for boars eating from its forests not to be apt for public consumption.

            Just hiding your head under the wing is not going to remove the massive problems we have already because of the nuclear nightmare, Fifi.

            We have to face reality: the first step to solve a problem is facing it.

            Then we have to quit: replace nuclear for renewables.

            Then we have to heal as much as we can: find realistic long term solutions for the already existing nuclear problems scattered all around.

            But ignoring the widespread destruction that nuclear power generates is not going to solve anything. Hopefully Fukushima will be an eye opener – though I wish it would have never happened.

      1. Hal Horvath

        mmmm…the more typical possibility: small plane hits nuke, would tend, once any part of the plane hits the containment vessel, the hard containment, to, well…er…stop. As in full stop, as in ant hitting a large rock. Larger planes — usually much the same result.

        1. DownSouth

          Hey Hal, why don’t you go watch some Glen Beck and give the rest of us a fuc_ing break?

          And speaking of the Glenn Beck Skool of Nu Clear Fiziks that you seem to be so enamored of, there’s this video:
          God’s Bigots: Rush Limbaugh & Glenn Beck Laugh at Japan’s Quake, Nuclear Tragedy

          And if you get tired of listening to Beck, there’s always the Rush Limbaugh Skool of Nu Clear Fiziks:

          Rush Limbaugh – The Media Wants A Disaster In Japan

          Money quote:

          This is just the wanton spreading of fear, when in fact most of these nuclear reactors in Japan are behaving as designed. They are containing.

          1. Hal Horvath

            my poor friend, you are going off the deep end. Definitely, more sun and outdoors would do you a world of good. Why do I think so? Because I’ve been warning people that Limbaugh/Beck for years now. So your guesses are hitting 180 degrees opposite of reality at this point.

            Who would I fall back on, if I needed (as you may): I recommend to you to try reading Emerson, as an antidote to various aspects of modernity.

          2. Anonymous Jones

            I agree with Hal. Being unsure of the entire cost matrix of nuclear energy is not the same as agreeing with fuc_ing Glenn Beck. That is a preposterous categorization. And I mean, fuc_ing *preposterous*. None of us knows all the costs and benefits associated with the types of fuel we use. At least every single person who purports to know this seems, to me, to be completely out of their mind.

            Yes, the occasional catastrophe is horrific. But the daily pain and suffering the attends other fuel sources, is this really better? I’m not sure. I certainly have not been persuaded by others that it is. I would need a lot, and I mean *a lot* more data for that. [Full D: I have no investments in any way in nuclear energy]

          3. DownSouth

            My “guesses”?

            Afraid not.

            You’re March 19 comment at 1:33 p.m. (down this thread) pretty much said it all.

            Anyone who would make a statement like “working engineers simply have an average integrity that is better than in the finance field” is a person who lives in a delusional fantasy world of his own making.

            I come from a whole family of engnieers. My brother is a PE, my nephew is an EE, my cousin is a CE, my cousin’s son was just graduated with a degree in nuclear engineering, and I’m a ME.

            Trust me on this, engineers are human, just like everybody else.

            Of course you don’t give a shit about engineers. You’re more than willing to sacrifice their huamnity on the altar of your ideological conceits, or your financial interests.

          4. DownSouth

            Anonymous Jones,

            I take it you haven’t looked down through this thread to read the full extent of Horvath’s comments.

            Horvath invokes the good name of science and technology in the service of his ideological conceits, or his financial interests. You uncritically buy into his claims and conclude that he is “unsure of the entire cost matrix of nuclear energy.”

            Of course that’s not where Horvath is coming from at all. If you read down through the thread, he is quite sure of himself. His comments overflow with hubris and arrogance. He pompously decrees that what he presents is “fact,” “engineering,” or “science,” and dismisses what his detractors say as beneath consideration. His methods are not those of science, but of rhetoric. His mission is not to find truth, but to persuade and to win an argument.

            And that’s why his methods are those of Glenn Beck or Rush Limbaugh, and not those of science or engineering.

    2. ian

      We have built many more cars and planes than we have nuclear plants. Because of this, we rely much more on foresight rather than experience with nuclear plants. It amuses me when I hear people say that the Fukushima reactors actually performed splendidly because they were only designed for much smaller earthquakes and tsunamis.
      Another point is that no matter how technologically perfect we make the reactors, you really have to consider the human beings who build and operate them as part of the system. No matter how diligent, people have bad days, get bored, are occasionally forgetful, and will cut some corners (we have already seen evidence of this with TEPCO).

  2. bmeisen

    Thanks for the summary and invitation Yves. I suggest that it isn’t a choice between growth addiction and normieness, it’s not a choice between urbanization and suburbanization. The answer for this planet – and the emphasis is on this planet and it’s irrefutable limitations – is sustainability. Nuclear power technology is in it’s inherent risk of widespead and long-term devastation antithetical to sustainability. The choice is between denying technocrats and oligarchs their toy, and willed ignorance. Greater energy efficiency and renewable energy sources are the answer. Arguing that a world without nuclear power will increase CO2 emissions is as valid as arguing that outlawing handguns will increase unemployment. Apparently instead of attempting sustainable energy policy people prefer running their dryers in the desert.

  3. Jani

    Absolutely it is worth the risks. We now have the 2nd worst nuclear accident ever and once the dust settles we will be looking at consequences that are not apocalyptic. We have in the worst case few hundred rescue workers exposed to fairly large doses of radiation and some might even get radiation sickness (we will know more soon), but unless the dose is more than about 5000 mSv this is unlikely to be fatal. (In Chernobyl there were few who got this much.) At 1000mSv level they have about 5% risk of getting fatal cancer due to radiation which implies at maximum a body count of around 10 due to radiation. This is too much, but hardly the end of the world. Deepwater horizon explosion alone killed 11 and caused a massive spill of toxic subtances. Ichibara oil refinery exploded and who knows how many people died there. A dam for hydropower in Fukushima also broke, washing away 1800 homes, gas pipes fractured causing fires. etc. etc.

    For the people outside the plant area the extra doses will, even in the worst case scenario, be so small that they do not cause any observable effect what so ever.
    (Panic causing extra road accidents and blocking roads so that help cannot reach those who actually need it, are far more serious threats…as is replacement of nuclear with fossil fuels. Extra stress due to fear is also a health risk.)

    As for renewable alternatives, please do the math. In Europe biofuels kill about 11-12 people for each TWh produced (not that there is enough living things to burn in any case). This implies hundreds of extra dead people EVERY YEAR in my small home country of Finland alone (5 million people), if one assumes the sort of forest burning promoted by our greens. For wind the death rate is about 0.15/TWh and that translates to about 50 dead people globally every year at current capacities…so about 1000 dead people during the 20 year lifetime of the turbines. Rooftop solar is more dangerous than wind.

    Nuclear security must be taken seriously and obsolete reactors replaced with new ones ASAP. But now that the worst has happened, we do not have to argue about apocalypse anymore. We can see that this was not the end of the world and japanese enjoy pretty much the same life as before except for those unfortunate ones to have lost the loved ones and livelyhoods in the earthquake+tsunami.

      1. bmeisen

        … and make sure that the cost of keeping them there for 30,000 years, and the cost of insuring against those not-so-serious fails, and the cost of developing the whole technology never show up in the utility bills we send to consumers many of whom will be able to continue to trivialize energy efficiency standards.

        1. Mark P.

          The good news is that industrial nuclear transmutation looks a couple of decades ahead. It has the potential basically to eliminate nuclear waste and rationalize nuclear energy. So your assumptions about nuclear waste are false.

          The bad news is that industrial nuclear transmutation looks a couple of decades ahead. Hence, we now stand potentially at the dawn of the golden age of nuclear arms proliferation.

          Generation IV reactor designs like the IFR (Integral Fast Reactor) and the LFTR (Liquid Flouride Thorium Reactor) can achieve high burnup of nuclear fuel and close the fuel cycle. Simultaneously, various fusion-fission and fast-neutron reactor designs could burn down the long-lived actinides and eliminate radioactive waste altogether.

          http://nextbigfuture.com/2010/11/nuclear-power-should-close-fuel-cycle.html
          http://www.japannuclear.com/nuclearpower/fuelcycle/what.html

          Such advances are dual-use, however. For a specific instance, Uranium-plutonium fast burner reactors and fast breeder reactors are much alike. Developing one technology means developing the other.

          So it goes.

          1. Firebird1

            Thanks for mentioning this. Not many people know about the possibility of using thorium, and the potential for consuming spent fuel. That is the only way forward, in my view.

          2. Hugh

            A problem is not solved because you have a potential technology decades, as you say, down the road. What do you do with the current stockpiles of spent nuclear fuel? What do you do with all the spent nuclear fuel produced between now and then? And while some kinds of thorium reactors may be able to use some spent fuel, that is a far cry from disposing of all of it. Nor does it touch all the materials with radioactive residues produced during the operation and eventual dismantlement of a nuclear plant.

      2. John L

        No, we should reprocess them into fuel for more nuclear reactors. Spent fuel rods aren’t devoid of nuclear fuel; if reprocessed the unused portion is a much smaller percentage of the rod’s volume. The rest can be reused in another power reactor.

        The Fukushima crisis shows where mistakes were made in both design and response to the crisis. Neither are unsolvable. Placing spent fuel rods on top of the reactor for convenience sake was obviously a bad design; not having backup generators and high rise, high capacity firefighting equipment stored offsite nearby was another one.

        But, the design of the plant was sound, and survived this one-two punch of the earthquake and tsunami. No containment or safety system failed from either one; had backup power been restored immediately after the tsunami hit, this crisis would have been averted completely. All the reactors are stable and contained; water appears to be in the storage pools and more is being brought to them via pumping equipment.

        In the other thread I was questioned as to what my sources were; some said they “contradicted everything they had read”. Well, my response is stop watching Fox News and CNN, and do some of your own research. Start with these:

        http://mitnse.com/2011/03/18/news-update-318/

        http://www.iaea.org/newscenter/news/tsunamiupdate01.html

        http://www.news.com.au/world/japan-attempts-to-cool-nuclear-reactor/story-e6frfkyi-1226020670565

        http://en.wikipedia.org/wiki/Timeline_of_the_Fukushima_nuclear_accidents

        1. Wile E. Capote

          “But, the design of the plant was sound, and survived this one-two punch of the earthquake and tsunami. No containment or safety system failed from either one; had backup power been restored immediately after the tsunami hit, this crisis would have been averted completely. All the reactors are stable and contained; water appears to be in the storage pools and more is being brought to them via pumping equipment.”

          Wow. Just wow.

          1. ya ya

            So said John L, of the Nuclear Lobby.

            Yves, you have to start moderating these commenters, maybe even a log in requirement. Even if it means I never post again, it is better than the miscreants that have been here lately.

            Too much agenda!!

          2. John L

            Agenda? I was challenged to provide the sources of my data in the earlier thread, I chose to put them here after the other thread was closed. Oh, and I don’t work for the nuclear industry in any way, shape, form or fashion; I do know enough about it to distrust nearly anything the near-hysterical US media has been throwing out there, and also have enough common sense to do my own research on this subject.

            Now, it appears that recent developments are showing my sources to be accurate; radiation levels decreasing, reactors under control, water being added to spent fuel ponds, power being restored. I’m sure a lot of you anti-nuclear types will find something to nitpick about, but will any of you admit that YOUR comments were the ones more inaccurate? I won’t be holding my breath waiting, though; I know the type all too well.

      3. Lyle

        But if you consider it we are wasting energy the way we handle the spent fuel. It is hot, hot can become energy, as used on the outer planet probes. Take the waste put a thermocouple on it and generate electricity for a few years till it cools down, and then reprocess into a fraction that is still heat hot and repeat. So actually the way we handle spent fuel is just throwing money away right now. There are lots of examples of direct heat to electricity or electricity to cold conversion. We just need to think differently about the products of the reactors. If you do some chemistry you can separate out the small volume of isotopes that stay radioactive for a long time, then the volume is much smaller. However today because of a fear of nuclear proliferation we intend to bury the whole rod in a hole in the ground.

        1. Electrical Engineer

          Nuclear power is not worth the risk, and here’s why:
          All nuclear power plants are permitted regular venting of CFC’s, and radioactive materials into the environment, counting on dilution by natural water and air to hide the radioactive isotopes being added to background radiation, but they are slowly being added nonetheless, and are not the same as natural background radiation. Nuclear is not worth the price no matter how cheap its short-term electricity is claimed to be. The long-term products are emissions, waste, weapons and accidents that WILL happen, because the risk is not zero.

          Nuclear is not domestic; according to the DOE, America imports 86% of its nuclear fuel. With the risk and cost of building nuclear plants, plus the trade deficit, I would have thought that the fuel cost, just to heat water, would be a tenth of coal’s, or less? I was wrong! The World Nuclear Association states that nuclear fuel costs amount to a third of the same size coal-fired plant. Americans can’t afford to pay for nuclear electricity, just so uranium can be profitably mined, and weapons material created. The waste of heat that could be converted to electricity by thermocouple proves nuclear is just a money game. Also, we wouldn’t see the persistent defense by shills; nobody’s so ignorant that they don’t get these facts. Forget these pointless debates and write your representatives.

          Now the economic/energy strategy to replace nuclear and fossil: Build no new nuclear plants, phase out existing ones and over the next 30 years replace nuclear and fossil with solar, now. No waiting 40 years. No more “drawing board” solutions, thorium, fusion, pie in the sky, space-based solar, etc. Sorry, this is just human nature wanting to get something for nothing, or very cheaply. From here on history, fact, and what we know. Dr. John O. Blackburn and Sam Cunningham have shown that nuclear has already lost its cost advantage to solar, in a heavily-referenced paper for the nonprofit North Carolina Waste Awareness and Reduction Network. For them to make this claim, how could solar not be cheaper than nuclear everywhere soon? And especially in ten years, when new nuclear plants could first start adding power to our grid, if construction started today? If we can afford tax breaks for the richest and largest corporations and bailouts for the largest financial institutions, we can afford solar.

          Solar energy can meet all our energy needs, while being far safer than nuclear or coal. The Solar Energy Industries Association has factual, irrefutable information on the scale and speed possible for solar panel installation http://www.seia.org/cs/news_detail?pressrelease.id=342 The United States has more solar potential for a given area than Germany does, and Germany has installed a total of 12,000 MW of solar panels, and installed 3800 MW in 2009 alone. In 2010 alone the EU, with a comparable population to the US, installed 12,000 MW of solar, and 9300 MW of windpower, but only 145 MW of nuclear. The US installed less than a tenth of the solar the EU did in 2010.
          http://www.renewablepowernews.com/archives/2212
          Solar can be installed far faster than nuclear plants, which take 10 years or more to build one plant. There is no physical limit to how many megawatts of solar panels can be manufactured or installed, just political resistance, There is no excuse anymore. Are you going to try and tell me that the United States can’t do even better than Europe has? Are you going to tell me that the greatest, richest country in the world can’t afford power sources free of nuclear risk? We are falling behind the rest of the developed world, providing pork-barrel tax breaks to fossil and nuclear industries who have experienced large profits, and don’t care that we are falling far behind in renewables.

          Statements that solar can only meet a fraction of our electricity needs are based on past rates of installation and current use, projected linearly into the future, and are refuted by Europe’s example. In the same way, it could be predicted that nuclear energy will comprise 0% of our energy mix in decades, it the Atomic Energy Act amendments to the Constitution were repealed (which should be done if that’s what it takes to kick the nuclear habit).

          Statements that we need nuclear for baseload power are false. Solar plants with thermal energy storage are now being built. The American Hydrogen Association has information on the technology to store non-toxic, carbon-free liquid fuel, and there now exist hydrogen-powered cars, trucks and a tractor by Massey-Ferguson, which means we can have scalable generators fo nigttime generation. If current nuclear investments were put into solar, wind and storage technologies, we could build the fuel infrastructure for production, transport and fueling, like America did a century ago for gasoline. What is needed is political pressure to cut current government subsidies and tax breaks from nuclear (twice as much is spent on nuclear research and development than on solar) and fossil energy.

          The priority for siting utility-scale plants should be closed nuclear plants, (if the radiation could be totally contained with zero emissions, so as not to endanger the health of construction or maintenance workers) and coal plants, since the transmission lines are already in place. Contaminated nuclear sites, some quite large, are the obvious choice for waste storage, encased with concrete, covered with solar panels. Other good sites are abandoned strip mines, factories, Superfund sites, landfills, ore milling plants, south-facing dams. This type of of siting will remove the environmental objections to large-scale solar and the safety objections raised by nuclear defenders criticizing rooftop solar. But, we would only have to cover a third of our total building roof area to absorb enough energy to power our whole country. Of course excess capacity and storage of heat or hydrogen need to be installed, for generation during sunless times. As an engineer, I believe wind power should generate primarily hydrogen for transportation and nighttime baseload power (which would eliminate the intermittency problem of connecting wind to the grid). Generation of electricity from stored hydrogen, made with wind power, would work even in the Alaskan winter.

          More jobs will be created by renewables, in the manufacturing, installation, cleaning and security of large solar installations, than nuclear ever will. The tops of prison buildings would not be bad solar sites, since they have guarded expansive roofs, and could meet all their needs and supply their current load to the grid with existing power lines. Farms, food processing plants, warehouses, truck stops, schools, government buildings, malls could all be sufficient and supply their current load to the grid, and sooner than nuclear can be built.

    1. Mark P.

      “We can see that this was not the end of the world.”

      Not so fast.

      It’s far from clear yet that we’ve avoided a catastrophe. That spent fuel closely stacked in the Fukushima storage pools is many, many times more radioactive material than Chernobyl ever spewed into the atmosphere. If those rods in pool #4 overheat, it could all still go up.

      This could still be a terrible catastrophe.

      1. John L

        While the storage ponds being placed on top of the reactors in hindsight was obviously a bad idea, as was not adequately providing backup pumping and shielding for these ponds, your comment that the spent fuel is “much more radioactive than Chernobyl” is misleading.

        At Chernobyl the combination of an interior core explosion, no containment shielding around the reactor, and a graphite moderator fire all caused the ejection of highly radioactive core material high into the atmosphere. The core material was also spread around the plant, making the region very radioactive. At Fukushima none of those elements are present, so how could this material “all go up”?

        Visual inspection of the spent fuel pond on Reactor #4 indicates there is water still there. In cold weather, warmer water ‘steams’ above the water’s surface. Video clearly shows this happening at Reactor #4, indicating there is water in that pool. Evaporation calculations estimate that the pool (holding over 200,000 gallons) will take well over a week before the level of the water starts to approach the top of the rods. Clearly they need to act to get power restored to this pond, but it’s not in danger yet.

        If the worst case does happen and one of these ponds empties of water (Reactor #3’s is more at risk actually), what happens? First, the radiation levels near the pool go way up, making it hard to work around there without getting a high dose of radiation. Second, the cladding around the fuel rods could heat to the point where they crack open, releasing long-lived radioactive materials. I’ve seen some tests where zirconium alloy shavings could catch fire if heated enough; it’s not clear if the solid tubes will do so though.

        Let’s say a fire does start; the lighter isotopes would be carried into the air to land further downwind. The heavier isotopes (the long lived ones) won’t get carried far at all. An area downwind of the plant certainly would be contaminated, as would the plant grounds itself, but even at Chernobyl (where a MUCH higher amount of radiation was released due to the violence and severity of the fire) the contaminated area was limited to 10 miles.

        So, a very bad situation, absolutely. A “catastrophe”, though? Very doubtful.

        1. Wile E. Capote

          You forgot to mention that only one person died due to Chernobyl and it was because he choked on a bit of meat.

          This means that meat is more dangerous than a nuclear reactor meltdown.

          1. John L

            Spare the sarcasm. You cheapen the lives lost at Chernobyl, as well as the workers risking their lives in Japan.

        2. Mark P.

          @ John L

          [1] Don’t misquote. I didn’t write ““much more radioactive than Chernobyl.”

          I wrote: “…stacked in the Fukushima storage pools is many, many times more radioactive material than Chernobyl ever spewed into the atmosphere.” That’s fact.

          [2] Here’s Tepco’s own inventory of stored fuel at Fukushima Daiichi from November 10th, 2020 — before they shut down reactor 4 for maintenance and dropped the whole core into the pool in October/December. After Tepco came clean with a presentation last Wednesday, someone leaked this to Zero Hedge.
          http://www.zerohedge.com/sites/default/files/Containment%20Pools.pdf

          Essentially, Fukushima Daiichi appears to have had a total of 1760 metric tons of fresh and used nuclear fuel on site in November, 2010. Merely the storage pool at reactor 4, which the NRC’s Gregory Jaczko claimed last week had already lost its cooling water, contained 135 tons of spent fuel at that time.

          For comparison, the Chernobyl reactors had about 180 tons when the accident there occurred. Hence, the storage pool at Fukushima Daiichi 4 in November 2010 alone held 75 percent as much nuclear fuel as all of Chernobyl.

          [3] What does it hold now since the dumping of the old core — still pretty hot — of reactor 4? That’s not clear. Overall, according to a current Forbes/Associated Press report, there are — as of the earthquake and tsunami — 3,400 tons of fuel in seven spent fuel pools plus 877 tons of active fuel in the cores of the reactors. That totals 4,277 tons of nuclear fuel at Fukushima.
          http://www.forbes.com/feeds/ap/2011/03/18/business-financial-impact-as-japan-earthquake-plutonium_8363305.html

          Again, Chernobyl had about 180 tons of nuclear fuel.

          [4] Downplaying the unique potentials associated with a nuclear plant’s catastrophic failure as compared, say, to that of a chemical plant is a self-defeating strategy. It not only won’t win ordinary peoples’ trust, it’ll turn them against nuclear proponents.

          Specifically, for instance, if one compares Chernobyl and Bhopal, it’s true that many more people died because of the latter than the former. Nevertheless, the global dispersal of Chernobyl’s consequences is a problem unique to nuclear.

          Thus the general public believe that not even one nuclear accident is acceptable. And frankly on this score they’re correct, however ignorant they may be about nuclear in general.

          [5] You may turn out to be the ignorant one, however. Nobody knows what they don’t know and we’re in uncharted territory with the disaster at Fukushima.

          Here’s a paper presenting a worst-case scenario by significant scientists at MIT and elsewhere, which was heavily resisted by the nuclear industry (submitted 2000, published 2003)and predicted a situation like the possibility we’re approaching at Fukushima pool 4: potential burning of zirconium cladding as spent fuel is exposed to air, then release of cesium & strontium, then fire spreading to older fuel and, finally, “long-term land-contamination consequences … significantly worse than those from Chernobyl.”

          Reducing the Hazards from Stored Spent Power-Reactor Fuel in the United States
          by Robert Alvarez, Jan Beyea, Klaus Janberg, Jungmin Kang, Ed Lyman, Allison Macfarlane, Gordon Thompson, Frank N. von Hippel

          “Because of the unavailability of off-site storage for spent power-reactor fuel, the NRC has allowed high-density storage of spent fuel in pools …virtually all U.S. spent-fuel pools have been re-racked to hold spent-fuel assemblies at densities that approach those in reactor cores. In order to prevent the spent fuel from going critical, the fuel assemblies are partitioned off from each other in metal boxes whose walls contain neutron-absorbing boron. It has been known for more than two decades that, in case of a loss of water in the pool, convective air cooling would be relatively ineffective in such a “dense-packed” pool. Spent fuel recently discharged from a reactor could heat up relatively rapidly to temperatures at which the zircaloy fuel cladding could catch fire and the fuel’s volatile fission products including 30-year half-life 137Cs, would be released. The fire could well spread to older spent fuel. The long-term land-contamination consequences of such an event could be significantly worse than those from Chernobyl.”

          More here –

          http://www.irss-usa.org/pages/documents/11_1Alvarez.pdf

          http://www.nrc.gov/reading-rm/doc-collections/fact-sheets/reducing-hazar

          http://www.irss-usa.org/pages/documents/SGS_213-223_response.pdf

          1. John L

            Paul,

            There’s a gigantic difference between an active reactor holding 180 tons of nuclear fuel (Chernobyl) and a reactor facility (Fukushima) containing thousands of pounds of spent reactor fuel in separate storage ponds and other locations. Chernobyl’s fuel was actively producing energy; the majority of the fuel stored at Fukushima is not anywhere that enriched since it’s considered “spent” and in storage ponds. Turn off the coolant pumps to the storage ponds and yes, eventually you will have a problem. It won’t melt, and there’s a difference of opinion on whether the cladding will burn or not; I’ve seen the results of a French test on spent fuel that showed the cladding will deform and split open, but not burn spontaneously. It just won’t get that hot. The fuel taken from Reactor #4 and stored in the pond (note: not “dumped”) is more reactive and presumably helps generate more heat, which is why there is concern over this pool, but newer accounts indicate it still has water covering the rods so the threat is not there yet.

            Now, turn off all the safety features on an active reactor, as was done at Chernobyl, and within the hour you get what happened there.

            You do understand the difference between six separate storage ponds of spent fuel generating some heat and being cooled, whether there’s more fuel overall there or not, and an active, full strength reactor with all the safety features turned off, right? You’re trying to make it sound like the Japanese had thousands of pounds of active nuclear fuel in a pile, awaiting only a match.

    2. william

      @Jani says: March 19, 2011 at 3:58 am
      “but unless the dose is more than about 5000 mSv this is unlikely to be fatal. (In Chernobyl there were few who got this much.) At 1000mSv level they have about 5% risk of getting fatal cancer due to radiation which implies at maximum a body count of around 10 due to radiation”

      Oh dear, oh dear, oh dear! I fear you’ve been ‘mushroomed’ (kept in the dark and fed —-) by the pro nuclear lobby, or I hope your not one of them trying it out on us.

      Because your figures are totally wrong, and underestimate the levels of danger out by at least an order of magnitude.

      “At 1000mSv level they have about 5% risk of getting fatal cancer”

      NO, at 90mSv there is a 5% risk of getting cancer. 1000mSv is acute radiation poisoning, nausea, diarrhoea, with a 10% chance of instant death.

      ” dose is more than about 5000 mSv this is unlikely to be fatal.”

      NO, 5000mSv is the certainty of *immediate* death (within a few days or weeks), a much lower dose is sufficient to set off cancers or organ damage that can kill you several months or years later
      The last time I checked 400mSv was the highest release from Fukushima enough to cause a 25% likelihood of developing cancer (on a time scale of years), but this was an one off, very brief incident at Fukushima and only of harm to those within the building.

      “which implies at maximum a body count of around 10 due to radiation”

      Sorry but this seems to suggest that you are saying 10 people died at Chernobyl? In which case you are seriously deluded, but it’s very hard to understand what you’re suggesting as none of your maths adds up; you give two numbers 5000 and 1000 and then suggest a 5% cancer rate, and say this caused 10 deaths? Ten is five percent of neither 1000 nor 5000. 5% of 1000 is 50 and 5% of 5000 is 250

      It’s all too familiar; like a report I heard the other day, a PR guy for the Nuke industry said that radiation dose in Toyko was “300 times background but that was ‘safe’ and would require 3 years of constant exposure at that level to present any danger to the public” and I thought to my self “ho hum, another day another wave of bull with Nuke PR people pulling numbers out of their orifices!”

      300 times background (4mSv is background) is 1200mSv which would in fact be an extremely dangerous *if true* there would have been bodies piling up in the streets, fortunately is a totally improbable number and manifestly just made up to sound impressive and fool the general public.

      The safest thing for joe-public to do is when anyone from the Nuclear industry says something automatically assume it’s a bare-faced lie (99% of the time it will be), and don’t listen to a word they say unless overwhelming evidence convinces you.

      Nuclear power is potentially extremely dangerous for those that live within eyeline of a station, however more importantly it’s an insanely expensive white elephant, and a total waste of time and resources! Oh and the waste makes lovely bombs.

      1. Jani

        I am sorry, but you clearly do not know what you are talking about. People receiving radiation therapy often get doses much higher than 5000mSv and are CURED of cancer as a result. 5000mSv dose does lead to ARS for sure, but not necessarily death. 1-2Sv dose is rarely lethal.

        As for the source of my information. It is the research consensus based on decades of research on radiation safety bu thousands of scientists. I think it is immoral to cherry pick loony researchers (“sceptics”) in case of climate change and just as immoral in other fields. Your claim that 90mSv leads to 5% cancer death rate is clearly nonsense. Here our yearly dose is 3.7mSv on average so during our lifetimes we get about 270mSv. Based on your reasoning MOST of the cancers would be due to radiation as opposed to chemicals and tobacco for example. Just crazy…

        1. Wile E. Capote

          In Soviet Russia, after Chernobyl meltdown, cancer rate went down since so many people received free treatment.

        2. Glenn Condell

          I think that sometimes visuals are required in debates like this. It’s too easy to sit back and shoot the shit verbally and in the safe context of debate to accept dangers that you would never, having seen rather than talked about them, entertain in a million years. It’s a bit like Iraq; you’d get into arguments with people who believed in Enduring Freedom, then you’d show them some choice pics of young victims of our goodness, from Fallujah say, and that had a tendency to shut them up.

          To put meat on the bones of a fairly arid debate, please see the work of Magnum’s Paul Fusco on the victims of Chernobyl. Watch it through, then after (perhaps reluctantly) accepting that it is impossible to guarantee nuclear safety in all possible circumstances, try to tell me it is still am option worth pursuing. Imagine if you can that these are your children:

          http://inmotion.magnumphotos.com/essay/Chernobyl

          1. kievite

            To put meat on the bones of a fairly arid debate, please see the work of Magnum’s Paul Fusco on the victims of Chernobyl. Watch it through, then after (perhaps reluctantly) accepting that it is impossible to guarantee nuclear safety in all possible circumstances, try to tell me it is still am option worth pursuing. Imagine if you can that these are your children:

            http://inmotion.magnumphotos.com/essay/Chernobyl

            I would call it pretty clinical speculation on the tragedy. We don’t know how he selected those subjects. And how many of them are related to radiation not, say, pesticide poisoning of mothers, consequences of chronic alcoholism including being intoxicated during pregnancy for mothers. Same for fathers.

          2. Maju

            You should know better Kievite – but seems you do not. IF there was nothing to report, nobody would report because nobody gains anything reporting about the effects of radiation.

            Ukrainian children have come every summer here to the Basque Country (West Saharan children do too for different albeit somehow similar reasons: minimizing children’s suffering), being hosted by local families and/or summer camps. I am used to listen to what their host families say: they improve a lot here, but lose most of it when they go back.

            Living under radiation is a long-term punishment against your health. You should be more compassionate with the people of your own country.

          3. DownSouth

            Hey Glenn,

            Thanks. Those fotos are absolutely heart wrenching and help put a human face on what we’re talking about here. They cut right through all the jargon and technical busllshit that the nuclear trolls use to give themselves credibility and divert people’s attention.

            They deserve wider publication.

      2. Jani

        ..also 4mSv is NOT the background rate, it is about right for a yearly dose (except there are places where natural rate can be more than 100mSv/year. Ramsar, Iran for example). The natural rate is 4mSv/(365*24)=0.46 micro Sieverts/hour.

    3. okie farmer

      Jani, you’re uninformed. Try reading this recent book, “Chernobyl: Consequences of the Catastrophe for People and the Environment”, which carefully documents, from Russian health care sources and other Russian documents, over 985,000 deaths, mostly from cancer, directly attributable to the Chernobyl disaster since 1986.

    4. Hal Horvath

      Good comment. In case you come back to check, let me tell you at least a few people read and appreciated what you said.

  4. attempter

    “Less energy intensive” is a meaningless concept in itself. It has to be placed in the overall physical and politico-economic context. In that context, we know urbanization has never been anything but a malevolent process.

    Physically, it represents the metabolic rift between the soil and the city, where a perfectly harmonized natural system is broken in order to transfer calories and nutrients from the farm to the unproductive city, generating two artificial problems, waste in the city and the need for synthetic fertilizers to replace the exported nutrients which would naturally have been used to replenish the soil.

    The result is an insolvent zombie whose collapse will be far worse than that of the banks, since it involves our physical sustenance itself.

    http://monthlyreview.org/081124clark-york.php

    Politically and economically, we know that the concept of driving people off the land and into the city as something which would increase everyone’s prosperity is a Big Lie, and we don’t need greenwashing as another justification for the feudal/corporate land grab. The world’s shantytowns, including the ones coming home to the West, will never be “green”. They’ll never be anything but pits of misery and testaments to the radical evil of corporatism.

    As Thomas Paine said in his agrarian essay, civilization is only any good if it improves the lot of everyone. But any “order” which drives the people off the land and into ghettos makes them far worse off than they were prior to this so-called civilization.

    The only way forward for a true civilization, which has to mean the further development of democracy toward its full fruition, including economic democracy, can only mean our return to the land. Peak Oil dictates that America will need millions of small farmers. Freedom and democracy demand even more, that individual smallholders and cooperative growers become the basis of the entire economy. As our history has proven, it’s impossible for democracy and “property” to coexist in any other way.

    Food Sovereignty, the human right to grow food, which must include access to the necessary land, is the only way forward for democracy, justice, and freedom.

    Meanwhile, no city dweller can ever be a citizen of anything or have any rights at all unless he has money.

    1. Mark P.

      ‘The only way forward for a true civilization … can only mean our return to the land.’

      Pol Pot thought the same thing.

      1. attempter

        Evidently you have no argument against what I wrote, only a schoolyard insult. That’s encouraging.

        Pol Pot, of course, stood for exactly the same thing your system does: Top-down coercive hierarchy.

        As usual, it’s me on one end of the democracy-to-elite rule spectrum, almost everyone else cramming the opposite end.

      2. Wile E. Capote

        You are very right to mention Pol Pot. If he thought something, how could it be good? We must naturally figure out what he thought, and do the opposite.

    2. jonboinAR

      Will an old fashioned agrarian lifestyle with a much higher ratio of small farms, a greater percentage of people living as farmers, support the much larger population we have today? I believe there’s some kind of energy efficiency gain with dense urban populations supported by large farms in the hinterlands. I don’t know this to be a fact. I suppose it is. Could some kind of optimum be reached by keeping populations more or less as they are, but farming a little more organically?

      1. attempter

        There’s no way to support all the non-productive parasites we have today, nor should we want to.

        But our best chance of growing enough food for ourselves, at any population level, is to simply do it, right there on all the land we have. Plenty of studies have proven that diversified organic growing maximizes caloric output per acre.

        That means we’ll have lots of work to do to restore all the stripped and depleted soil in all the places where this ridiculous bank-based “society” has prevailed.

        I’ll say again that the fact that this will require so much work can be either a blessing or a curse, depending upon what the people choose. If we choose to do it democratically, we can easily become a fully, fulfillingly employed society. If we choose to continue with “property” and “capitalism”, we’ll all first become slaves, and then starve.

        So the answer is that over the long term we’ll maximize our absolute production and especially the production which we’ll eat ourselves by transforming to smallholder and cooperative growing on the basis of a land dispensation based on useful possession.

        There will be no other way to grow enough food as fossil fuels deplete. Your alternative is to slave in the corporate fields to keep feeding the elites as the labor force gradually starves to death.

    3. kevin de bruxelles

      As you allude to, and as Fernand Braudel often confirms, the history of capitalism is really just the story of a transfer of power from rural to urban zones. Peasants, and eventually wealth, flocked to the city. Industrial goods increased in value while agricultural goods decreased. We see this process on-going even today in China.

      But with this transfer has come an astounding increase in the standard of living. In the fifteenth century 80-90% of European people were subsistence farmers. There is no doubt that the transfer to a capitalistic economy was not purely top-down. Many, most people were more than happy to give up a rural life on the edge of starvation for the dream (although often not the reality) of a comfortable urban (and lately sub-urban) middle class existence.

      So how exactly does a bottom-up process of overturning the 600 years of urban-oriented capitalistic development actually happen? How many people today in Western societies are interested in going back to the land — outside of a few naive foodies like me? People complain about austerity, wouldn’t sending huge percentages of people back to subsistence farming would be the mother of all austerity programs. Is it really possible if we went back to a huge percentage of the population growing their own food that the standard of living would not drop precipitously?

      Historically independent farming is a weak organizational model for military success. All a few nomadic shepherds (a powerful model for dominating others) have to do is attack these guys a few times at harvest time and our heroic farmers will find themselves year after year with the dilemma of fighting to avoid slavery with only the reward for victory being starvation during the following winter since they were forced to leave their crops in the field.

      I read recently somewhere that America’s drive towards suburbs and exurbs is all linked to the fact that American poor, along with their pathological and dysfunctional cultural habits, are housed in centrally located urban ghettoes. People aspiring towards middle class status flee further and further away from these centers of despair, requiring longer commutes and more and more wasteful use of energy. The idea expressed was that instead of giving food stamps and welfare so the urban poor can stay in inner-city areas, they should instead be sent to the countryside, and after a year or two of training, given some land and seed and told if they want to eat they better get busy farming. This would create a cycle where urban losers would be kicked back to peasant status to start all over again the process of making back to the city. This isn’t bottom up but maybe something along these lines would at least start the process of transferring portions of the population back to the land.

      1. Anonymous Jones

        I love your comments as always, but the drive toward the exurbs and the suburbs is a story that was relevant in the past. It doesn’t apply as much in the present. Of course, the texts and studies haven’t caught up yet.

        Old cores have rehabilitated all over the U.S. Many citizens have recognized what was left behind (living in a vibrant community) and seek to re-establish what was lost. It is the urban poor that has been exiled to the exurbs in the last ten years. [No subjective judgments here, just an objective observation.]

        We’re not all idiots, and we’re not all the same. Stereotypes and heuristics are, as always, useful, but they do not tell the entire story. That should always be acknowledged.

      2. attempter

        Kevin, the potential differences are that during the Oil Age we achieved tremendous new knowledge about farming and about democracy. Combine the two new wellsprings of knowledge, and we can do much better than pre-oil farming society. And we can definitely do much better than what the “elites” have planned for us.

        We’ve also learned a lot about vigilance, and a vibrantly democratic community which regularly trains for self-defense (a citizen reponsibility) ought to be able to destroy the kind of “nomads” you refer to. I suppose, at least initially, the model would have to be a contested homestead rather than an Amish idyll. Except that unlike in previous times, we’d be democratically and cooperatively reclaiming our own land rather than stealing it from others. We’d only have to fight and destroy the kind of scum which refuses to work and knows only theft.

    1. bmeisen

      Absolutely – the alternatives are there. The alternatives do not include timber and diesel from food crops. The alternatives are not being developed because big energy is locked onto fossil fuels and nuclear and sucking them and us dry for every penny they can get.

    2. Mark P.

      Haven’t thought this out, have you?

      No government is easily going to tolerate another state using it. Militarily, it’s a giant orbital energy cannon.

      Civil-power-wise, if transmission of a power beam to Earth goes astray goes accidentally astray, you have an accident that could compare with nuclear.

    3. charles 2

      I agree completely that space based solar power is part of the future, but we have to crack the nut of really cheap transfer to orbit for that (and I mean cheap as what container shipping is today). This is very much a problem of power density, and nuclear can be helpful for that (for instance for powering laser-based space elevator).

      On an industrial basis, we are talking 22nd Century here…

      1. Ignim Brites

        Not going to happen. We have reached Peak Civilization. Population decline and economic contraction take over from here; for the next millennium.

  5. bdaltrey

    Isn’t it interesting how people will rail about their personal right to be free of secondhand smoke in a restaurant, but support dousing the entire planet with second-hand radiation for their own expediency.

    1. Wile E. Capote

      I don’t mind getting cancer to support the expediency of U.S. corporations’ quarterly profits. It’s getting cancer from Japanese radiation leaked due to their corrupt practices that I object to.

  6. Name (required)

    To ask if nuclear power is worth the risk is a Luddite approach – all machines are bad.

    Nuclear fission has exactly the risks described above – a highly toxic source material and as we’ve seen at Fukushima and elsewhere the huge risk that once control of the reaction is lost it quickly escalates.

    However nuclear fusion takes as its fuel a harmless, widespread material – deuterium, which the oceans are full of – it produces no radiation and just getting the reaction to start and maintain itself is the huge technical difficulty which is yet to be overcome sufficiently for commercial use, so any loss of control of it simply results in it fizzling out.

    Nuclear fusion – the other form of nuclear energy – is clean and green, would produce unlimited energy in perfect safety, and is closer to reality than nuclear fission was in 1940. All it would take is something equivalent to the Manhatten Project, with proper sufficient funding and the best brains, to make coal, oil and practically every other means of power-generation hopelessly uncompetitive.

    But if nuclear fusion is to be tarred with the same brush as nuclear fission in the minds of the great unwashed, electricity will go back to being a luxury for the rich and influential just as it was a century ago.

    1. bmeisen

      “It takes considerable energy to force nuclei to fuse…”

      From wikipedia.

      You seem to be suggesting that we invest billions more into a technology that has consumed billions and produced little usable results over the last 50 years, to address an issue that calls for usable solutions as soon as possible. The usable solutions are available with substantially less investment. The solutions are greater energy efficiency and solar, wind, wave, and geo-thermal alaternatives.

      some day nuclear power could be used but not on this planet.

    2. Mark P.

      Since the nuclear age’s dawn in late 1940’s-early ’50s, fusion has sat tantalizingly somewhere up ahead soon, soon, soon — somewhere n the upcoming decades or within the coming half-century. The standard physicists’ wisecrack for decades has been: “Fusion is the future of nuclear power and it always will be.”

      Go look at the massive infrastructure of ITER, which is the biggest international big science project after the Internation Space Station —

      http://www.iter.org/
      http://www.iter.org/proj
      http://www.iter.org/sci/plasmaconfinement
      http://www.efda.org/multimedia/original_size.php?pic_no=37&fol=original_size

      Understand that ITER’s infrastructure is necessary because this is a project “to put the sun in a bottle,” in one physicist’s words.

      Assuming a sustained fusion reaction could be made to work with this titanic complex of machinery within the next couple of decades, what makes you believe that would then have any practical smaller applications as a power technology? Indeed, what makes you think the harnessing of the force of the sun and the stars if that force slips out of control might not be orders of magnitudes more destructive than a mere meltdown of fissile material in one of our current puny little nuclear plants?

      There exist some left-field small-tech fusion projects, like the Bussard polywell —

      http://en.wikipedia.org/wiki/Polywell

      It would be pretty to think one of these approaches might bear fruit. I would like that. But it’s not realistic to expect that to happen.

      1. Toby

        Mark, you announced with warm praise the following in an earlier comment:

        “The good news is that industrial nuclear transmutation looks a couple of decades ahead.”

        Yet, here you say scornfully:

        “fusion has sat tantalizingly somewhere up ahead soon, soon, soon — somewhere n the upcoming decades or within the coming half-century.”

        A little hypocritical, don’t you think?

        As for up ahead, two Italian scientists successfully demonstrated their cold fusion device to journalists and scientists in Feb of this year. They are in talks with a Greek energy company at the moment regarding production and distribution. Apparently the US is refusing to patent the tech because cold fusion is impossible. Meanwhile an American company called Black Light Power has its machines in production right now in two US states at least. Theirs is a form of cold fusion, but don’t ask me about specifics.

        That said, before we crack the energy problem, we have to fix the economic one; perpetual growth. The last thing we want is humanity unleashed via unlimited energy to trash the planet even more. Earth is already exhausted and right at the point of collapse. Ask the fish and the bees. Check out soil fertility, water cleanliness, etc. Before anyone gives us the keys to the energy candy store, we must initiate a revolution in the money system, and in education and politics too.

        In short, we need to democratize all sectors of society, get off our addiction to growth, and focus on energy efficiency and sustainability. Only then will we be wise enough, culturally, safely to wield the power of the atom.

          1. Toby

            Thanks for the link. It doesn’t convince me though that BLP is a hoax, though of course it could be from where I’m sitting because I don’t have access to all the info. That said, it’s ultimately renewables that are the future, since only renewable energy guarantees sustainability.

        1. Dirk77

          Toby, ask any real physicist. Black Light Power is a scam.

          That said, I agree with your comments. There is no sense in helping humanity cure its energy problem if all that is going to do is make the earth a Trantor. Until we deal with the twin evils of economic and population growth, why should any reasonable person lift a finger to help out?

          1. Toby

            You may be right about the scam, but I’ve become so suspicious of ‘real’ physicists I try to keep an open mind. ‘Real’ physicists were very wrong about flight for example, they wrote books about how it was impossible. It took four to five years after flight was proved before the mainstream would accept it. Later, many argued putting a rocket on the moon was impossible. Over and over again the ‘experts’ refuse to accept anything that lies outside the orthodoxy, yet again and again orthodoxy gets overturned. I don’t see that pattern stopping any time soon.

            But yes, BLP is a minor detail. Clean, renewable energy is key if we want true sustainability.

        2. Mark P.

          @ Toby –

          Perhaps the word ‘fusion’ confuses you.

          Industrial nuclear transmutation means moving the advanced fuel reprocessing techniques and reactor designs that we already largely have onto an industrial footing. It’s not not much noted, but at this point we can essentially manufacture in small quantities any element or isotope we choose — it’s just not economically feasible to, say, manufacture gold.

          Reproducing the self-sustaining processes inside the heart of a sun on the other hand and containing those — that’s fusion, too, but perhaps many orders of magnitude more difficult.

          1. Toby

            Perhaps the word ‘fusion’ confuses you.

            It doesn’t. Fusion is joining, fission is splitting. But that is hardly relevant to my point.

            As to economic viability, the day mainstream economics works out what is viable and what is not will be a day I rejoice. Right now it looks to be even further in the future than clean nuclear waste.

      2. Another Gordon

        Another left-field small-tech project worth keeping an eye on is dense plasma focus (DPF). The big problem with “traditional” big tech approaches is that it’s virtually impossible to contain a plasma hot enough and dense enogh to sustain fusion because the plasma goes unstable.

        The DPF approach is to stand this on its head and use the instability. Very clever and so far it’s working well but it doesn’t solve the short to medium term problem.

        http://www.lawrencevilleplasmaphysics.com/

    3. charles 2

      “widespread material ” ? “produces no radiation” ? I think you are misinformed:
      Fusion as researched by ITER is Deuterium-Tritium fission, the first element is easy to find, the second is not. You can extract it as a by-product of fission (and it is actually a PITA for nuclear engineers) or “breed” it through Lithium Blankets which is a huge engineering challenge.
      DT Fusion produces 14 MeV Neutrons, more energetic that anything that fission produces, materials behavior under such irradiation is still a developing field.

    4. John L

      Fusion is the carrot sitting on the end of a 30 year long stick, and its proponents keep saying “just another 50 billion dollars and we can get there!”

      1. cenobite

        Yes. We already have a fusion reactor at known safe distance, using gravitational confinement which is known to work over billion-year timescales, thousands of times our species lifetime to date.

        The amount of energy reaching the earth from the sun is many many orders of magnitude greater than that used by humanity.

        The big problems with moving to renewable energy are not really in the scientific or engineering layers, they’re in the political layer.

      2. jonboinAR

        But it’s diffuse. Concentrating it enought to provide the energy to sustain our desired lifestyles is kind of hard to do.

        1. Toby

          The rest of nature seems to have sorted it out, so there is precedent. Also, solar energy generates wind, a wonderful source of power. And of course there are other renewable energies. As for storing energy, there are some sweet ideas out there. This one looks really interesting, though is by far not the only one:

          http://www.zeitnews.org/energy/cryoenergy-system-uses-liquid-air-to-store-energy.html

          From the article:

          In Highview’s CryoEnergy System (CES), excess energy is used to run refrigeration units which cool air down to a temperature of -196C (-320.8F), at which point it liquifies. The liquid air, also known as cryogen, can be stored in an insulated tank, at an ambient pressure of about 1 bar.

          At higher-demand periods, when the direct output of existing energy sources can’t meet the needs of the municipal power grid, the liquid air is released into a confined space. The liquid boils as soon as it is heated above -196C, so even room temperatures will superheat it, causing it to regasify and expand in volume by approximately 700 percent. From there, a steam engine effect comes into play, with the high-pressure gas spinning a turbine which in turn powers a generator.”

          I’m pretty sure what people really want is clean, renewable energy. What we don’t want to be doing is poisoning the ecosystems which enable our continued existence. That we feel we ‘have to’ do so is part of a story called Economics, but this story–including the stupidity of interest bearing debt-money, a fantasy called The Invisible Hand, and the illusion Perpetual Growth–is bullshit.

          If we want to survive as a species with a healthy future, sustainability must be our top priority.

  7. Brett

    I’m sorry to be the first to comment, with nothing positive to say but can someone really offer anything beyond magical thinking about future technologies? I think nuclear power in its present form is an atrocity and I would like to see it “un-invented”, but that being said we still have global warming, pollution, peak oil, collapsing biodiversity, blah, blah, blah. We are at war with the equilibrium of the world. If our growth/consumption is not addressed then it is only a question of who suffers. Us, our children, or our grandchildren? I don’t live in the woods, I don’t own gold or silver and I don’t stockpile ammunition, though sometimes I wish I did. I’m not what I would consider a “doomer”, but we seem to consistently overestimate our ability.

  8. bs23

    As a physicist, I’m positively predisposed to the interesting new reactor designs and the possibility of getting a lot of energy out of not a lot of fuel, not to mention not releasing tons of CO2 in the process.

    Nevertheless, recent events in Japan lead me to think that we require several orders of magnitude improvement in design if it’s going to be possible at all to have nuclear power with acceptable risks and at an affordable cost (which I’m not convinced of). The trouble is not that nuclear plants can’t be designed to operate safely, but rather that it is apparently difficult to operate them safely, given the inevitable errors/fraud in operation itself.

    The situation in the spent fuel pool demonstrates this dramatically. “No one could have predicted” (which I seem to be hearing a lot lately) that there would be trouble storing so many spent fuel rods in a way that the cooling system could be rendered inoperational. WTF? Why take the risk? Instead, store the spent fuel in a way which doesn’t require active supervision. It’s pretty much unconscionable to both generate so much dangerous waste and have no real plan for its storage which will require thousands of years.

    Or, for instance, take the next-generation design for a sodium-cooled reactor. Sounds great — don’t need a containment vessel capable of withstanding very high pressures, so that’s one less thing to worry about. But then you learn that sodium explodes when it comes into contact with water or air. WTF? That’s never going to happen? Oh, and the sodium coolant becomes radioactive, (though with a 15 hour half-life), so there’s that to worry about, too.

    At this point I’m unconvinced. Do I see a good alternative? No. =/

    1. DownSouth

      bs23 said: “…we require several orders of magnitude improvement in design if it’s going to be possible at all to have nuclear power with acceptable risks…”

      Improvement in design of nuclear power plants, or of human beings?

      Trust me, guys like Bush and Obama and their corporate masters could fuck up a crowbar.

      1. bs23

        both—the whole system’s gotta work, from the guys working in the control room to the inspectors making sure that all the welds that were supposed to be x-rayed really were x-rayed.

        this is why i don’t have a lot of confidence in the whole enterprise…

    2. DownSouth

      bs23 said: “…we require several orders of magnitude improvement in design if it’s going to be possible at all to have nuclear power with acceptable risks…”

      Improvement in design of nuclear power plants, or of human beings?

      Trust me, guys like Bush and Obama and their corporate masters could f_ck up a crowbar.

    3. Wile E. Capote

      Are they dusting off the “no one could have predicted” meme again? Because this time there were years and decades of articles, papers, unheeded warnings, whistleblowers, you name it about absolutely everything that could and did go wrong.

      1. bs23

        I only meant that tongue-in-cheek for all the writing out there about how well the design stood up, considering a huge earthquake and tsunami isn’t what it was designed for. That’s precisely the point! It’s not exactly confidence-inspiring to hear the supporters talking about all the stuff that wasn’t thought of…

        As someone with at least a passing familiarity with how reactors work, I can see that it’s easy to get drawn into thinking that various problems are unlikely to arise. “It’s such an elegant design!”, etc. Simply too much faith in the improbability of high-risk events.

        As you say, many people have certainly raised all these issues before, but were for whatever reason ignored.

        1. Hal Horvath

          There’s a good point in there, which occurred to me the other day.

          We had a cheap reactor design, also design requires extensive cooling after shutdown, 30-40 years, imprudent extention of licenses, multiple reactors/pools in close proximity, and a massive earthquake and big tsunami, power failures, backup power failures, and any radiation releases become problematic for working at nearby reactors, skeleton rescue crew…and…

          no massive meltdown?

          Kinda impressive in a way.

          Sure they could do things a lot better on so many fronts, but it’s kinda impressive as it is even.

  9. kievite

    So far there was no death caused by radiation in Japan so all those talks about whether nuclear power is worth the risks should start with calculating the real risks.

    The baseline here is the number of victims of road crashes (37K per year for the USA). Society tolerates this level so it is clear that this amount of radiation deaths per year represents acceptable risk. The fact that it will be radiation induced deaths is immaterial to the discussion.

    If we adopt this framework I think it is clear that even with current Japaneses nuclear disaster risks are low.

    And the key question here is not about nuclear energy but about energy in general. Can mankind continue on sustainable path if energy inputs are dropping? What parts of lifestyle need to be sacrificed? What price we are ready to pay?

    If we assume the this is the peak of fossil fuel extraction and the next phase is the decline, what are no real alternatives to nuclear power even if those incidents prove that cost of nuclear energy if much higher that it was initially assumed? And it will rise anyway because uranium reserves, especially the high-grade ores, are depleting rapidly. But with recycling of fuel it probably can last for several hundred years.

    Another question is how black is the Japanese nuclear swan? There were several gross errors in design and placement of the reactors that now became textbook like happened with gross errors committed in building Chernobyl (actually all new projects of reactors of this type were killed after the incident). This “survival of the fittest” process can bring some positive results.

    Also reactors were very old and there probably should be obligatory process of modernising or decommissioning of this types of reactors in the USA. That might improve safety as well.

    The idea is reactors should have scheme for passive defence (including the “killing switch” that injects boron or something similar) and that the redundant systems should be duplicated in military style to minimises the chances of complete loss of reserve generators energy common.

    Like with Chernobil, this loss of power that was the reason of the catastrophe was completly avoidable. Reporcessing of used nuclear fuel also is an important problms that needs to be solved. And as there are no alternatives it will be solved.

    It’s really sad that the possibility for cascading failures were simply ignored. Issues of reserve generators location and protection were botched really badly and here criminal investigation probably is warranted. May be some small gas powered energy plant should be built in visinity to provide reserve energy feeds into all critical systems. Why those reserve feeds need to be built now with heroic efforts?

    1. DownSouth

      Kievite said: “The fact that it will be radiation induced deaths is immaterial to the discussion.”

      That is simply not true.

      Behavioral psychologists have demonstrated that certain and immediate consequences result in entirely different emotional responses than uncertain and delayed consequences.

      Heightened unpredictability actually produces more anxiety, more fear and more dread—-leading to heightened frustration aggression—-than sure and predictable consequences. This documentary film discusses the research (beginning at minute 1:14:00).

      The broader discussion of frustration aggression begins at minute 55:20. As the moderator notes, engendering frustration aggression in a population is one way the ruling class maintains control of the population, because frustration aggression is like putty in the hands of politicians and other demagogues:

      Frustration aggression is one of the most effective ways of managing a population. By directing a person’s rage against selected minorities, or outside enemies, the true cause of an individual’s frustration can be effectively diverted.

      1. Dan Duncan

        Actually, Downsouth…the ruling powers maintain control of the population by using the works of selected behavioral psychologists who are employed to foster delusions.

        It’s called “Frustration Paranoia”.

        Heightened Paranoia actually produces more anxiety, fear and dread than rational thought.

        The noted British intellectual Ozzie Osborne, along with several colleagues from The University of Black Sabbath wrote a documentary this subject in 1970.

        The broader discussion of Frustration Paranoia begins with the following entreaty:

        “All day long I think of things but nothing seems to satisfy
        Think I’ll lose my mind if I don’t find something to pacify
        Can you help me occupy my brain?”

        [The irony, of course, is that interlocutor is paralyzed into a form of pacification by allowing paranoid, but generalized non-pacific thoughts to treadmill in his mind…all…day…long.]

        It is a well known phenomenon that engendering Frustration Paranoia in a population is one way the ruling class maintains control of the population; because frustration Paranoia is like putty in the hands of politicians and other demagogues:

        By directing a person’s rage against selected ” generalized conspiracies”, an effective dispensation of the individual’s frustration can be thwarted… because when everything is a conspiracy nothing is random. If nothing is random, then all is pre-determined. And if all is pre-determined then the rat gets back on his treadmill and…

        “Why am I receiving this reply? Why would this person tell me that my dot connection between “Ruling Power Control”, “delayed and unpredictable outcomes (which really isn’t unpredictable after all since The Ruling Power is using it for mind control)” and nuclear power deaths is bat-shit crazy?

        “This person is trying to trick me. AHAH! He is one of THEM!”

        1. DownSouth

          It comes as no surprise that you would viciously attack anyone who tries to unmask the social, political and intellectual structures that are used to justify and perpetuate the inequalities of modern society. Of course you, just like Glen Beck or Rush Limbaugh, cannot do this by citing empirical or experiential evidence. You are thus forced to resort to these highly rhetorical and emotional appeals which are totally devoid of evidence or human experience. Exaggeration, stigmatization, derision, and mockery define you, or as Martin Luther King put it: “closed-minded reactionaries [who] gain prominence and power by the dissemination of false ideas, and by deliberately appealing to the deepest hate responses within the human mind.”

          There was a time when the rhetorical strategies employed by you, Beck and Limbaugh worked. I suspect that this may no longer be the case. Stephen Toulmin explains:

          The function of cosmopolitical arguments is to show members of the lower orders that their dreams of democracy are against nature; or conversely to reassure the upper class that they are superior citizens by nature.

          [….]

          Doctrinally…the trajectory of Modernity has closed back on itself, into an Omega; but experimentally it has headed broadly upward. As people in Europe and North America have learned from the experience of modernity, and have attacked the inequalities built into the “modern” scaffolding, they have developed a discriminating care for human interests. In the 1770s and 1780s, the revolutions in America and France successively challenged social self-awareness; and, ever since, the emancipation of the classes which the New Cosmopolis labeled as the “lower orders”—-those human groups whose needs and interests were long disregarded without compunction—-has been a consistent theme of political debate. Despite setbacks and counter-revolutions, there has since 1776 been a growing perception that such inequalities cannot be justified by appeals to “the Nature of Things” or “the “Will of God” or any other doctrine.

          The same has been true in the natural sciences. In outgrowing the scaffolding of Modernity, it has finally become the “experimental” quest that Bacon foretold in the late 16th century. La Mettrie and Priestly, Hutton and Darwin, Marx and Freud had to take seriously the objections of “respectable opinion”; but, now that the last timbers of that scaffolding—-the separation of humanity from nature, and the distrust of emotion—-have lost their intellectual credibility, no obstacle remains to studying nature however our experience requires. Of a dozen recent examples, the most striking is perhaps that of the conservative Surgeon General of the United States, forced to choose between his scientific understanding and his ideology, sponsoring a campaign for sex education and for wider use of condoms.

          Since the 1960s, then, both philosophy and science are back in the intellectual postures of the last generation before Descartes. In natural science, the imperial dominion of physics over all other fields has come to an end: ecologists and anthropologists can now look astronomers and physicists straight in the eye. In philosophy, Descartes’ formalist wish—-to refute the skepticism of the Renaissance humanists, by substituting the abstract demands of logical certainty for their concrete reliance on human experience—-is now seen to have led the enterprise of philosophy into a dead end. Scientifically and philosophically, that is, we are freed from the exclusively theoretical agenda of rationalism, and can take up again the practical issues sidelined by Descartes’ coup d’état some 300 years ago.
          ▬Stephen Toulmin, Cosmopolis

      2. Hal Horvath

        “By directing a person’s rage against selected minorities, or outside enemies, the true cause of an individual’s frustration can be effectively diverted.”

        mmm…that might be worth mulling over, don’t you think?

    2. rjs

      that no deaths have yet been caused immediately is not the issue; the issue is the mutagenic properties of nuclear radiation; it takes four generations for the effects to show up in mice; we’ve barely gone thru 2 since the widepread atmospheric testing in the 50s…

      http://marketwatch666.blogspot.com/2010/06/go-nuke-youself.html

      biological life is a fragile counter-entropic (i.e., generating organization rather than defaulting to randomness) force on our planet, and you dont want to upset the delicate balance that allows life to organize molecules…free radiation is known to be one of the most potent disorganizers of life, and as such would further lower the ecological potential of the planet…deaths in auto accidents of from coal induced pollution dont upset the unpinning of life on the planet…

    3. DownSouth

      Kievite said:

      There were several gross errors in design and placement of the reactors that now became textbook like happened with gross errors committed in building Chernobyl (actually all new projects of reactors of this type were killed after the incident). This “survival of the fittest” process can bring some positive results.

      Can this “survival of the fittest” process indeed bring positive results?

      We have a culture that teaches people that economic priorities—-profits—-trump human priorities. In a capitalistic system, profit is God, and competition is the Holy Grail.

      This is “mad rationality,” as Amitai Etzioni explains in The Moral Dimension, and the antidote to it is morality:

      [N]ormative values, as factors that influence the choice of means, help ensure the primacy of ends. The preoccupation with means, with enhancing their strength, scope, quantity and quality, is the essence of industrialization, market economics and economics, technology and applied science, in short, of the modern age. However, this preoccupation, through a process known as goal displacement, tends to lead to the primacy of means over ends. Studies of organizations are replete with reports of organizations designed to serve a specific goal: however, when the design proved to be inappropriate, rather than adjust it, the organizational goal was replaced to suit the existing design. Multi-millionaires work themselves to a frazzle to increase their income. Executives work “for their families,” destroying their family life in the process. Societies undermine their fabric in order to accelerate economic growth. This phenomenon has been referred to as irrational rationality, or mad rationality.

      Alfie Kohn describes how capitalism has become so completely disconnected from the goal of elevated human flourishing in the documentary film I linked above (beginning at minute 55:10):

      Competition builds character. In fact, what we find is that by any reasonable notion of character, in terms of psychological health or self esteem, that competition undermines that and creates a kind of neurosis because we come to think of ourselves as good and competent only to the extent that we have defeated other people. And so we’re always playing this desperate king-of-the-mountain game where we’re all worried about triumphing over other people and stepping on their faces and looking at them as if they’re going to step on our faces. That has two effects. One is it’s horrible for us in terms of psychological development because there’s a perpetual sense of dis-ease and anxiety. Second, it very logically has a destructive effect on our relationships.

      We compete because we’re raised that was, not because we’re born that way. Take, for instance, the belief in “survival of the fittest,” which is seen as a Darwinian notion. In fact, Charles Darwin never even used the phrase “survival of the fittest.” That was coined by a right-wing social thinker in the 19th century named Herbert Spencer who tried to corrupt Darwin’s thinking into his own reactionary political purposes. What Darwin talked about was natural selection, which means that the individual organism that’s best able to adapt to a changing environment is more likely to be around to survive and reproduce. Well that doesn’t specify competition as a mechanism. In fact, often the active avoidance of competition, if not the deliberate pursuit of cooperative strategies, turns out to make it more likely that organisms or entire species will survive.

      The research consistently shows that competition not only isn’t necessary for excellence, but tends to impede excellence on most tasks, and the more challenging the task, the more ingenuity and problem-solving skills it requires, competition tends to disrupt that achievement. Excellence pulls in one direction and competition pulls in another.

      And in fact another kind of research study corroborates that. If you take a whole bunch of people and give them a task to do, some kind of problem to work out, and half of them are told “see if you can figure out how to do this task,” and the other half are told “this is a contest with a prize to whoever wins, whoever does the best job,” study after study after study, across cultures, across gender, across ages, find that the people who compete, who have to compete, end up doing an inferior job on that task.

      At the moment, it appears that much of what happens in schools in North America is really for the convenience of people who have most of the power. There is if anything an act of discouragement of critical questioning. Corporations claim they want kids who are able to think outside the box, but only so far as they’re caught within a larger box. That works to the advantage of the free market, which means that the market economy, based on competition, based on economic rather than human considerations, ends up controlling the system.

      1. Xavier Onassis

        Alfie Kohn! Yes Yes! Man of brightbrilliant brawny brain! His “No Contest – The case against competition” is a must-read, life changing, let the obvious finally be seen for what it is, perspective-shifting book! Bless you big time for contributing him here, DownSouth (takes a brawny brain to know one?!)

        Search on my screen name (it’s pronounced ‘save yer own asses’) and his name, and you’ll get a grand review I didn’t write plus links to where you can read 4 chapters online free.

        Alfie’s re-thinking is vitally connected to re-thinking nukes.

    1. eric anderson

      You are correct, sir.

      The question asked was, Is nuclear power worth the risk?

      It depends on what kind of nuclear power. Thorium? Yes. A pebble bed uranium reactor is much safer, too. It will not melt down.

      Really, what we need are simple systems that are fault-tolerant.

  10. Conscience of a Conservative

    You left out the terrible environmental damage cause by hydro-electric power. The loss of species in our rivers caused by dams should cause every one to be taken down.

    1. Wile E. Capote

      Don’t you know that the principles of ecology are a left-wing conspiracy to try to subvert Man’s God-given right to rule over other species?

    1. Mark P.

      I’ve thought the same thing.

      Maybe it’s indeed time for our species to engineer its adulthood and think long-term. The problems we now face are increasingly ones whose solutions, if they are to occur, require long-term management and stewardship.

    2. bmeisen

      If we could re-engineer our species to the demands of technology then we would all be dead shots – not with guns but with spears.

  11. SqueekyFromm

    The scariest thing to me about nuclear power, is that we have a lot of people in government who do not believe in either firm and effective government regulation of anything, or in corporate responsibility.

    Assuming there was an American Fukishima, or two, or three, what would we discover in its wake??? Probably the same thing we have with the financial meltdown. Is it out of the question that we would have Republicans trying to find a way to blame poor people and minorities when the Dagny Taggart Atomic Power Plant blew a gasket and lit up a few million citizens??? You know, the “Well, if we weren’t trying to make electrical power available to people on welfare, this never would have happened???”

    Is it out of the question that the Democratic Barney Franks and Dodds Clone of the day would gut effective reform efforts or that the Nuclear Energy Equivalent (NEE) of Larry Summers would be a Goldman Alumni, who just coincidentally worked overtime to insure that new regulations would not the pocketbooks of the Wall Street crowd???

    Frankly, I would not trust this country to try to handle an increased nuclear energy load, even though it is called for. We would be treated to Boehner or that Kentucky Goober, whats-iz-name, in the Senate to blame the “loose rads” on taxes being too high.

    Squeeky Fromm
    Girl Reporter

    1. Wile E. Capote

      I believe the official position will be that there is no problem, and the radiation was confined, and the birth defects and cancers were an anomaly caused by eating bananas. To the extent there was a problem, it was caused by too much regulation so that deregulation is in order.

    2. reslez

      Should a meltdown happen in the U.S. the right-wing noise machine would go something like this:

      1) Those People were warned of the dangers and should have evacuated! (Even if there was no adequate warning or transportation to leave.)

      2) Stop crying about a little radiation! I microwave a breakfast burrito every morning and I’m perfectly fine. Radiation sickness is all in their heads. Those People are just whining.

      3) People hurt by the radiation are poor anyway, or minorities, and therefore don’t deserve much compensation for their injuries. If they do get any compensation, the government should pay for it.

      3) Government regulators failed to ensure the safety of this nuclear plant. Therefore, government regulation is the problem! If a private corporation had run the plant unrestrained by government meddling no disaster would have occurred. We will pass some measures into law that pretend to fix the problem but in actuality repeal the remaining (expensive) safety regulations.

      4) This is all the liberals’ fault for complaining about greenhouse emissions. They wanted nuclear power because it produces less CO2. So blame the fallout on them!

      5) Duck duck blame, duck duck nonsense, duck duck tribal idiocy etc.

  12. Sleeper

    Ah nuclear – the shining future.

    Unfortunately nuclear has several problems.

    Nuclear is one of the most expensive means of electric power generation usually a premium of 30% to 50%.

    Reguardless of the cleverness of our designers and engineers, the dilgence of our inspectors, or the skill of our crafts people accidents and failures are bound to happen as they do with all electro mechanicalsystems.

    The nuclear waste is a deadly poision for many years – the half life of nuclear waste is much longer than any human civilization. we have not yet found a method of disposal.

    The way out of this mess requires that several steps be taken:

    1) Stop new nuke plants. This is simple simply remove the loan guarantees and the Price Anderson insurance support.

    2) Repower exisiting hydropwer plants with higher efficiency turbines.

    3)

    1. charles 2

      “Nuclear is one of the most expensive means of electric power generation usually a premium of 30% to 50%.”

      When you have domestic gas and coal resources and don’t care about global warning that is.

      For the rest of us, it is still competitive.

      1. sleeper

        Hydropower is emission free and some 9.000 megawatts could be generated by simply re powering existing plants with high efficiency turbines. Perhaps 10 times as much could be generated at exisrinf dams which do not now have hydropower capabilities.

        Hydropower cost is estimated at 2 cents to 4 cent per kWh.

        Nukes are estimated at 9 cents to 11 cents per kWh.

        1. Wile E. Capote

          This is why we need to market nuclear power as being “bespoke power for the discerning gentleman” and “exclusive power with the prestige of modern technology”.

        2. Cedric Regula

          For comparison purposes, the DOE says our national average power cost is 11.9 cent/kw-hr. The breakdown on sources is roughly 50% coal, 20% natural gas, 20% nuke, 10% everything else with hydro weighing in at around 4%.

          The DOE describes hydro as “nearly fully exploited”. The effiency of water turbines and electric generators have been very high for around 100 years I would think, but even if we greatly improved it, found more Hoover Dam locations, and doubled the 4%, we only get 8% of current electric supply from hydro.

          I’m curious where you get your data from?

          1. sleeper

            The source I used is http://hydro.org/why-hydro/affordable/
            For further information try the INEEL hydro prospector site.

            In addition a good source of info is DOE’s 20% wind and AWEA-Wind.

            Nuclear certainly has its place on naval ships but it is too expensive and dangerous for any other power generation.

            And remember much of the fuel for nukes will be fabricated by a French company Airva (sp) at the federally subsidized MOX plant – Savannah River. We will contine to rely on foriegn entities to supply fuel for US reactors.

            Nukes are expensive, dangerous, and there is no safe way of disposing of the spent fuel which is a deadly poison.

        3. Hal Horvath

          sleeper, re your 4:48 comment: Using the storage problem as reason not to have nuclear isn’t thought out in one sense — we *already* have the challenge, past tense. Shutting plants, other than any which should be shut for safety in time, doesn’t solve the storage problem or even help it> storing 140 units is approximately the *same problem* as storing 100 units. etc.

          In other words, it’s already here, so that has no bearing at all on whether to operate plants going forward. None. Other factors do, but not the storage problem.

          1. sleeper

            So since it is already there we should add to the problem by creating more poisonous waste ?
            If this logic is followed all manner of aliments should be accepted since they already exist.

            Let’s accept the fact that Yellow Fever, Plague, and Cholera exist and do nothing to control them.

      2. Lyle

        Actually I saw that the price of electricity has risen enough that for the first time they are planning to put hydropower on the dams on the Ohio (they are there for barge navigation). While the heads on the dams are not high the water flow is high. Likewise we should investigate run of river plants on the lower Mississippi. (A run of river plant does not have a large reservoir, and basically just lets the river run thru the turbines. Again the head is not high but the volume of water is immense. (You would have to build locks for barges but we do know how to do that).

  13. sleeper

    Sorry folks here’s the rest of it.

    2) Repower existing hydroplants with higher efficiency turbines.

    3) Install hydropower turbines on existing dams and impondments. By some counts there are some 80,000 dams and impoundments used for flood control, water supply and recreation that could be used for power generation.

    4)Develop wind power. We are blessed with a large wind resource enough to supply 20% or more of our energy needs.

    5) Encourage Combined Heat and Power installations for all large campuses i.e. universitire hospitals, coporate headquarters. CHP can be 30% more efficient than electric power plants and line losses are much smaller.

    6) Require that power companies report givebacks as income.

    There’s more but that shoud do it for now.

  14. charles 2

    I think it is worth the risk, but we should get out of the “local minimum” in which “conservative evolutionary” development of nuclear power plant lead us into. We tended to be focused too much on the “controllability”, I.e. the guarantee that the reactor can be shut down in any circumstances, but we clearly underestimated the need for long term cooling systems even after a successful scram, and the infrastructure that it supposes. There is also the, related, ENORMOUS hole that the advent of WAR is never part of nuclear power plant “Design Basis Assumptions” anywhere. Tsunamis can be powerful “correlating” event that break down redundant and supposingly “independent” safety systems, but WAR is another. We have to take the opportunity of the re-questioning that Fukushima will trigger to put that crucial question on the table. We went through an extended period of peace since 1945, but it may not last for the next 60 years or so, precisely because energy will be scarce !

    Personally, I favor underwater nuclear power plants. If they are not too big, they are very close to a “walk-away” safety after they have been shut down (in emergencies, flooding the section surrounding the pressure vessel should provide enough passive cooling). Even if they are subject to an act of war, the maximum failure mode is not utterly catastrophic (there are already a few nuclear submarines at the bottom of the oceans, some of them in small pieces, and we are still here, not to even bother about it !).

    The French are on it, http://en.dcnsgroup.com/energie/civil-nuclear-engineering/flexblue/ but the US can get in the game very quickly thanks to their extensive navy-nukes know-how. On a short term basis, the short term in nuclear engineering being several years, PWR in submarines seems to me to be the only solution that can be subject to industrial development (I.e. the hundreds of units that are needed to satisfy the world’s energy hunger).

    On a longer term basis, the Russians have an interesting concept with lead-cooled reactors, which can be fast reactors and solve the problem of long term uranium scarcity, and can also be submarine-based (http://www.akmeengineering.com/ ). Kirk Sorensen on http://energyfromthorium.com/ believes Molten Salt in submarines is the solution. I think he underestimates the engineering and industrial developments challenges of MSRs but he and other molten salt supporters are smart enough to prove me wrong…

  15. lxm

    I don’t think the problem is with nuclear energy per se.

    The real problem is that the power generation plants are too big and concentrated and their failures cause cascading effects. Big power plants are kind of like too big to fail banks.

    Imagine one giant coal fired plant generating all the energy for America. What happens if it fails? The whole country stops. Kind of like what is happening in Japan now.

    Now imagine a system of smaller, more disbursed power generators where the failure of some will not impact the rest. Like those batteries that take a licking but keep on ticking!

    Can nuclear be part of this? Sure. In fact, unless we learn to live with less energy, which is unlikely, nuclear will have to used.

    Smaller, redundant, though less efficient, power generation should not suffer catastrophic failures. Bigger is not always better. Now if we could only fix the banks.

  16. Bruce Krasting

    Interesting comments. Both for and against.

    Let’s face it. Nukes are dead. No new ones will be built in the USA for another decade. A number of the older ones (Indian Point) will be closed in less than a year as a result of what has happened.

    I doubt Japan will build any either. I suspect they will shut down a number that are either on the water on or on a fault. Europe too will look at this and turn the other way.

    There is no science behind these conclusions. Just human nature. But what the heck are we going to do? The cost electric energy is going to sky rocket as a result.

    1. Wile E. Capote

      That happens to be right. Economically speaking, nukes just don’t compete. Even while externalizing all costs of environmental “accidents” to citizens by letting insurers and plant operators off the hook. Even while government picks up the tab for many disposal problems and loosens regulations beyond the point of lack of safety.

      The only reason we are having this discussion is that there have been several million dollars worth of campaign contributions by the nuclear industry from which they expect several tens of billions in grants and loan guarantees. And, no one has any doubt they’ll have their way.

    2. DownSouth

      Bruce Krasting said: “There is no science behind these conclusions. Just human nature.”

      Geeze, are you ever stuck in the 17th century. Stephen Toulmin explains:

      The chief girder in this framework of Modernity, to which all the other parts were connected, was the Cartesian dichotomy. The more the extent to which natural phenomena were explained in mechanical terms, as produced by cosmic clockwork, the more (by contrast) the affairs of humanity were allotted to a distinct sphere. The sharpness of this separation was new, and it is worth noticing how Descartes and Newton took it, and how their successors interpreted it. While it divides the modern framework both from Renaissance humanism and from the late 20th-century world view, it was seen around 1700 as having indispensable merits.

    3. charles 2

      Well, that would leave Russia, India and China to keep the industry alive, and that is more than enough. If they manage to get power for half the price for 20 or 30 years, Europe and the US will get back into it, but will be technological laggards by then.

      The only way to displace Nuclear Power worldwide is to demonstrate on a large scale that renewables are cheaper or same price. That may be possible, but remarks from renewable lobbyists themselves – like “BEE said an expansion could cure that problem, but could not happen as long as Germany is oversupplied with thermal capacity” – do not trigger optimism in this regard. ( http://www.reuters.com/article/2011/03/16/us-germany-renewables-idUSTRE72F2RD20110316 )

      I wish Germans could understand the causation :
      no nuclear power plant in Germany => more gas import from Russia => Less gas for Domestic Consumption in Russia => more Nuclear Power Plants in Russia (The Russian are already building a NPP in Kaliningrad mainly for export purposes)

      Ultimately, you exchange Nuclear Power Plants built, maintained and operated by Germans by Nuclear Power Plants built, maintained and operated by Russians. Not necessarily a wise move from a safety standpoint….

  17. Ray Duray

    A century ago Mark Twain described a gold mine as a “hole in the ground with a liar standing next to it”.

    Today a nuclear power plant is a technological wonder with a liar standing next to it.

    Comments about the technology miss the point. It is the (mis)management of nuclear facilities that is the fatal flaw in the system.

    Re-engineering ‘the manager’ would be Job #1 in a sensible world. The WSJ article on TEPCO’s penny-wise, pound-foolish management gets almost to the heart of the matter. http://tinyurl.com/6b2lesq What the WSJ reporter (part of the corporate team after all) fails to report is a history of deceit and cheating on the part of TEPCO management for decades.

  18. Random Blowhard

    We have only 3 viable options – burn fossil fuels (coal, natural gas) or atoms (nuclear). The 3rd option is shivering in the dark, which if we don’t get cracking on either option 1 or 2, will be the winner by default.

    1. sleeper

      Let’s rely on nukes

      We can shiver in the dark as the
      Japanese are doing while we die of radiation poisoning.

      1. Wile E. Capote

        The silver lining in that little cloud of yours is that we’ll be glowing in the dark, so at least we’ll be entertained.

  19. AnonPi

    Didn’t anyone notice that Chu asked for Federal Loan Guarantees for Nuclear to be tripled in the last week or so?

    Nuclear is here to stay, just as automobiles are. Like it or not, we consider the death toll on the roads as an acceptable cost for the freedom the automobile provides. And, when you look at the number of fatalities, nuclear power vs. automobiles, which is riskier?

    1. Give Sympathize Control

      “Like it or not, we consider the death toll on the roads as an acceptable cost for the freedom the automobile provides. And, when you look at the number of fatalities, nuclear power vs. automobiles, which is riskier?”

      Depends on your point of view. If there’s a car accident 10 miles away from my house, it generally doesn’t mean that I have to abandon my house, potentially forever, or suffer an agonizing 10-year death if I don’t evacuate in time. So while the death toll from the much-more-common-than-nuclear-reactors automobiles is currently higher, the impact of auto accidents is limited to those directly involved in the collision and to a short duration of time. So, I’ll take my chances crossing a busy highway, and you can live next a nuclear reactor designed and overseen by “the best and the brightest.”

      1. Wile E. Capote

        You’re wrong about it being forever. When you come back in 500,000 years, your house should be safe and sound.

        1. bob goodwin

          Your exagerating. The half life of Cesium is 300 years. Safe levels can be achieved from any accident in 3000 years. :)

      2. charles 2

        a) you will receive indemnification from the power company, they f*** up, they pay. TEPCO lost dozen(s) of billions of market capitalization, it is not for nothing.
        b) Evacuation will occur way before you receive a significant dose. An opportunity that was not given to the more than thousand homes that were taken by a broken dam in the same earthquake. http://www.kaj18.com/news/earthquake-causes-dam-to-break-in-japan/

        This being said, if some people want to sell their house at bargain price because it is 5 miles from an operating nuclear power plant, they will be plenty of rational buyers, including myself to place a bid.

        1. Give Sympathize Control

          “you will receive indemnification from the power company, they f*** up, they pay.”

          Just like British Petroleum and their gusher in the Gulf? I sincerely doubt that any power company would have enough cash to properly compensate any communities more densely populated than ghost town levels. Or the loss of good farmland or pasture. And how the hell do you put a price tag on radioactively contaminating, say, the Everglades or Yosemite?

  20. drfrank

    I think the disaster in Japan could have been averted had the utility been honest in its reports and had responsible parties, the utility, the government, the press, responded to data that would have required spending money to improve the design of a plant built 40 years ago. To me, these are human failures, ultimately stemming from greed and pursuit of short term gains.

    Also, like it or not, I don’t think there is any way to support contemporary lifestyles, to say nothing of business and industry, without nuclear generated electricity in ever increasing amounts, even though it comes from “dark satanic mills.”

    The “design flaw” may well be the human apparatus. Anthropologically speaking, the question is why our society has been unable to form institutions that protect us from what we like to call “natural disasters.”

  21. moslof

    My Father was one of the few people to operate a defense reactor for the AEC in the fifties and also inspect commercial reactors in the seventies for the NRC. He was surprised at the lack of “defense in depth” that the commercial engineers designed into their plants when compared with the government defense reactors.
    For example fuel pools in the defense reactors are underground and sealed on the outside with a dense sand/clay mix to prevent catastrophic leaking even if the three foot thick walls are cracked by an earthquake.
    Cost cutting and cozy regulators may have doomed this industry.

    1. DownSouth

      moslof said: “Cost cutting and cozy regulators may have doomed this industry.”

      Yep, like the old saw goes, pigs get fat, hogs get slaughtered.

  22. ian

    I don’t doubt that nuclear power plants using uranium and plutonium can be made safer, A lot safer, but here’s the thing. There will *always* be economic motivation to cut corners and drop safety features for immediate profit. That same motivation will forever stop us from finding meaningful ways of storing the waste.

    The problems aren’t technical. The problems are human. For *that* reason, nuclear power can never be safe. The engineering is good enough, but we’re not good enough.

      1. reslez

        Did not destroy the environment? You have no basis for this assertion. For all we know radioactive byproducts devastated the landscape surrounding the natural reactor for millions of years. It’s true that the reactor was deep underground which would limit its exposure to the biosphere, but that’s certainly not the case for the reactors built by humans today.

        If you mean it didn’t cause the destruction of the planet and/or terminate all life on Earth, I’ll agree.

        1. Lyle

          Given how far back that event happened, it is not possible to draw any conclusions on the effect on multicellular life. (There was none 2 billion years ago when the reactor was running). I suspect if they had not gone down subduction zones earlier such events also occured. In fact the likleyhood is that some of the waste was pumped out of a volcano a few tens to 100s of millions of years later. So it depends on what one means by the environment since at that time the atmosphere was mostly Co2. In fact this is during the time of an atmosphere with almost no oxygen.
          However no matter what given enough time mother nature will cope (humanity may go extinct but that may be Gia’s wish anyway as she may view humanity as a bad infection). After all nature recovered from the Permian extinction and the end of the dinosaurs, as well as the snowball earth 750 million or so years ago, which btw is when multicellular life arose.

    1. reslez

      As an alternative to writing off all of humankind we could instead lay the blame at the feet of our civilization’s economic system. We have zero hope of eliminating short-sighted corner cutting as long as we grant Croesus-like rewards for it.

      I don’t know what kind of ubermensch could run a “safe” nuclear plant (insofar as it is possible to make them safe) if there were no consequences and lots of advantages to running a shoddy one.

  23. pixiestix

    addicts often rob their own families as their situation degrades, and become increasingly violent in their demands for the resources to continue their flight from reality and become steadily less careful about the purity of substances ingested. Many a family fortune has been pissed away in this manner. Discussing the question of whether nuclear power is worth the risk is pointless and futile amid the white noise generated by scores of stark raving crackheads demanding their next rock. They will stay high until all that is left is a steaming pile of naught.

    1. 1whoknu

      Excellent analogy. We have become addicted to their drug while our dealers feed their addiction to profit.

  24. embee

    Released coal ash is more radioactive than the release from a nuclear power plant. Now, that amount is very low for both, way below background, but it does accumulate in the surrounding soil. The article is here. http://www.scientificamerican.com/article.cfm?id=coal-ash-is-more-radioactive-than-nuclear-waste

    At this link, you can calculate your typical exposure in mRems. (100 mRem = 1 mSv) I like to think in mSv, the number is smaller) http://www.new.ans.org/pi/resources/dosechart/ Know that this calculator seems to have low values for medical procedures.

    The average person’s avoidable exposure mostly comes from medical procedures. Normal background in the US is 3.6 mSv. Angioplasty is about 7.5-57 mSv. It depends on how long you are in the procedure. Here is the reference for this. http://www.hps.org/documents/meddiagimaging.pdf Again, this organization seems to have a bit of a bias that there is an acceptable level of radiation exposure. I’m not sure, just want to collect what information we do have.

    From the National Academy of Sciences 2006 Study on Health Risks of Low Levels of Radiation BEIR VII report. Estimates of risk associated with exposure.

    100 mSv dose above background leads to 1 additional cancer in a population of 100, while approximately 42 of the 100 individuals would be expected to develop solid cancer or leukemia from other causes.

    Allowed yearly dose for US workers is 50 mSv.

    You can find the NAS reference here: http://www.nap.edu/openbook.php?record_id=11340&page=R1

    Just want to add some numbers to the debate. All our options have consequences and risks.

    1. gepay

      It is surprising that so many people who think themselves clever think it is a good idea to use nuclear fission to boil water. Many even believe the risk is acceptable even though insurances companies who livelihood depends on estimating the costs of risk can’t come up with a good figure for nuclear power plants.The Price Andersen act was passed to exempt them from the liability that most other companies are expected to deal with. Have you ever noticed the nuclear exemption on your prosaic car insurance policy?
      I am amazed that there are people who don’t count the 6000 excess childhood thyroid cancers of children in Belarus in the path of the radiation plume as casualties of
      Chernobyl. Or acknowledge that because the Soviet Union collapsed soon after Chernobyl there were no extensive epidemiological studies of the many thousands of people who were exposed dealing with the cleanup and sarcophagus. Or even the effects to the people living in the large city of Gomel who were not evacuated as the radiation plume passed. Suppose the winds had gone to Kiev?
      The non-rigorous studies done after TMI used the usual methods of not doing extensive weather-wind modeling to find the most likely people exposed to the radiation and following them for decades. I suspect the usual method doing only short term studies and widening the pool of people until any statistical effects were diluted was used.
      The fact is there is no safe level of radiation. Exposure to more ionizing radiation is worse. There are statistical tipping points where the effects can be
      quantized.
      I think the case can be made that there is a negligible decrease in the use of coal because of the limited adoption of of nuclear power. There is just more electricity available for our wasteful power intensive lifestyles.
      There is also the fact that the newer plant designs will most likely have unanticipated flaws because they are new – ask any software designer. Not to mention that most incidents happen because of human error which increases with new and complex technologies.
      There will always be corruption, greed, and the inevitable human error. The consequences of those days (which don’t happen very often) when everything that can go wrong does are just too large for nuclear power.

      1. Cedric Regula

        “It is surprising that so many people who think themselves clever think it is a good idea to use nuclear fission to boil water.”

        It’s because we know we aren’t clever enough to use it for a warp drive.

        Then Greenies told us we can’t use coal to boil water anymore.

        Clearly, you don’t want us to boil water anymore.

  25. Toby

    No, fission energy is not worth the risk. The waste is too toxic for too long (100,000 years) and the risks of explosion, however small, simply too potentially destructive.

    There are perfectly viable alternatives; wind, solar, geothermal, wave, cold fusion, and so on, which, combined with a concerted effort properly to utilize plus-energy housing technologies, maglev transportation technologies, energy efficient city design, etc., would go a long way to building a second Garden of Eden (no, I’m not religious).

    Growth Forever is simply impossible, and considering global poverty has doubled since the 1970s, isn’t even delivering prosperity. Whatever we now choose to do as a species, we must first develop a sustainable socioeconomic system, which means ending bank-created debt-money for profit, then setting up democratic money-creation systems which use neither debt nor usury. Such systems are in use today in developing countries, and use no banks, only the Internet and mobile phones. They are the future.

    Without the sustainable way of life only democratic money can engender, we will destroy ourselves and much of the planet with us. The logic is real simple. If the system is not sustainable, it is going to collapse, by definition. Therefore, sustainability must be the top priority, if longevity and the health of our descendants concerns us.

  26. eclair

    Chose one: nuclear power or fossil-fuel power. Is there a reason other options are not offered?

    Why not a combination of wind power and large-scale solar arrays (with heat stored in melted salt)? Plus power produced by burning refuse, junk wood, etc. Lord knows the US has acres of land where the wind blows and the sun shines (Wyoming, the Dakotas, south-western Colorado, Arizona, Texas, come to mind). And, we’re the original throw-away society with unlimited supplies of garbage.

    My pet theory is that solar and wind are not “sexy.” No legendary wild-cat drillers, no tough-guy jobs on floating oil rigs, no jargon-spouting nuclear engineers or heroic white-suited techs putting their lives on the line as they try to prevent total melt-down.

    Who would make a movie about a guy who drives about maintaining wind turbines or film the agonizing slow movements of a solar array as it tracks the sun? And a career designing plants to burn garbage?

    I don’t mean to trivialize the discussion. But let’s consider options other than fossil fuel and nuclear.

    1. Paul

      The energy densities of wind and solar power are too small to be practical as primary energy sources. Each of those huge wind turbines (with 100 meter rotor diameter) that you can see are 2.5 MW. The six reactors at this site were about 750 MW each. Therefore this nuclear power facility is the equivalent of 1,800 wind turbines (being generous that all of those turbines will have favorable wind conditions to hit full load when needed). Replacing nuclear power in Japan would require the construction of tens of thousands of wind turbines. The energy infrastructure required to collect and transport wind power is also more expensive because you need an electric grid to collect the power from each of the wind turbines where nukes make all the power from one building which can be plugged directly into you high voltage transformer from which it can be efficiently transmitted to where needed.

  27. Jim the Skeptic

    “Is Nuclear Power Worth the Risk?”

    We don’t know. We have only been using this type of technology for about 60 years.

    Steam engines were first patented in about 1700 and about 170 years later Steamboat boilers were still exploding from misuse or manufacturing flaws.

    We are still learning how to design and build a safe nuclear power plant. And we are still learning about operating these plants. We have a huge source of heat which MUST be constantly cooled. Inadequate cooling allows steam to form in the cooling water and water pumps don’t like to pump water with steam. Any discussion of nuclear power plant safety seems to fixate on the strength of the reactor vessel or the storage of the spent fuel rods while ignoring the humdrum business of cooling.

    It appears that automatically shutting down a power plant worsens the ability to provide adequate cooling. Backup systems are seldom used and are not tested using real world criteria. A short monthly test does not prove anything. In this latest case, we are hearing that the diesel fuel may have been contaminated by water. Apparently there were also batteries which should have provided power for 8 hours.

    The operating parameters of nuclear power plants seem to be finely tuned and operate very well within those parameters. But once steam is forming, the operation quickly degrades and damages escalate quickly.

    The risk is a disaster like Chernobyl plant which will be a hazard to human beings for about 100,000 years. Nothing in our experience offers such a serious penalty for an accident. What is the appropriate level of safety to be required?

    We are not going back to the farm unless armed force is used. We need this source of energy, unless it is just inherently unsafe. I don’t believe we can say it is inherently unsafe. We are still in the learning phase.

    I have not seen any reported problems with the US Navy’s nuclear reactors used to power ships. Proponents of nuclear power point out that the Navy’s nuclear reactors are “gold plated”, which may well be the case. So the issue may be the costs of building and operating a “gold plated” commercial nuclear power plant.

  28. Maju

    Nuclear power exists primarily to make nuclear weapons. That’s the ugly truth. Additionally it may serve to give some diffuse sense of “energy independence” but nothing that you could not achieve by renewables (hydroelectric, solar, eolic, tidal – but also biogas, methane, which helps recycle sewage residues and other organic trash while generating energy).

    A fallacious reasoning argues that renewables are “expensive” but in fact polluting energy sources are much more, just that the costs of pollution are not factored in and are derived to the taxpayer a posteriori. In many cases the states have actively (but discretely) subsidized nuclear power, which is quite costly even before factoring in pollution.

    In addition to all that nuclear power is extremely dangerous, as we have seen repeatedly. Sure, maybe a severe accident only happens (as long as all safety measures are ongoing) every 20 years or so. But the resulting damage is forever. Uranium 235 has a half-life of 25 million years: more than all the history of not just our species (200,000 years) but all the history of the Great Apes altogether. There is simply no future for a radioactive area like Chernobyl or Fukushima: it will be toxic forever and ever: it’s a nuclear desert.

    And a high maintenance one… because the Chernobyl sarcophagus is already in need of replacement (but there’s no budget).

    With nuclear power we are only creating extremely severe problems for the long run. Even if no accidents happen, we still have loads of eternal extremely toxic waste we do not know what to do with it (surely best would be to send it to the Sun but it’s very expensive and risky – just imagine if the spacecraft explodes with all that radioactive junk in it).

    Nuclear is eternal doom and nothing else.

    “we need to get over our growth addiction and start adapting to less energy intensive lifestyles”

    Sure that too. But this is a problem for the wider economy and needs, first of all, to replace the Capitalist greed-based system for something else, some sort of communitarian-wellbeing-based Socialism with emphasis not on growth (as in the USSR or formerly China) but on happiness, satisfaction, well-being.

    We have developed such huge productive forces that we really do not need to grow (in general terms) anymore. We can perfectly satisfy the needs of everybody and more with a fraction of what we produce now, provided an effective planning and an egalitarian distribution. We would also get loads of free time (global unemployment is extremely high) and we could use it to gain in life quality.

    The 5-hrs. journey was already proposed by Paul Lafargue a century ago. Nowadays we could easily reach an even smaller average journey (maybe 1 year work, 2 years holiday? – at least 3 working-days’ weeks), provided we only produce what we really need (or strongly demand) and that work is rationally divided among all able people, qualifying those who may be needed, etc.

    We could have a really enjoyable life.

    “which if we were really serious about it, means much more urbanization in the US”

    Not necessarily because renewables are often decentralized and work best that way. Also nowadays with the Internet and such a lot of work can be done at distance. However increase in public transport would be indeed needed in your country, even in urban areas like LA. However if electricity is generated from environmentally friendly sources and cars/trucks are electric (or hydrogen-powered, with hydrogen being generated electrically) the impact can be greatly reduced even if private vehicles are used.

    1. John L

      By claiming the entombment around Chernobyl needs replacement and then saying the irradiated area is ‘forever’, that’s a nice bit of misleading you do there. Tests have already found the exclusion zone around Chernobyl has very little radiation in it except right around the plant itself, where the live reactor fuel was ejected. The Chernobyl plant still generates power without causing unreasonable hazards for the operators.

      As for the Japanese plants, the irradiated area at this time is mostly limited to the plant site itself, with mostly short-lived isotopes that decay very quickly. There’s really no way to compare the two, given that Chernobyl involved a major release of radioactivity and Japan has not.

      1. Wile E. Capote

        I will be so bold as to compare the two. Clearly, Chernobyl represents the abject failure of Communism, whereas Fukushima represents the triumph of Capitalism, since the reactors in Fukushima survived the tsunami.

      2. Maju

        You seem to be in the payroll of the nuclear lobby, John L.: you know too well the lies of the industry to be otherwise.

        Anyhow, guess I’m lucky I just happened to write a little blog entry after I wrote that above, and I have therefore gathered some data in the process:

        For example here there is a report on how the whole area is still polluted on as much radioactive cesium as used to be the very first day. Half-life expectations do not seem to work well for this material for reasons that scientists do not understand well.

        Radioactivity is forever. The best we can do is find geologically stable spots, make miles-deep holes and bury all that poisonous stuff in it. Better even would be to send it to outer space but what if the rocket explodes while still in the atmosphere? We have to put an end to this nightmare for good: it’s just too dangerous and the half-life of the radioactivity of materials used is longer than all human history. We are burdening the unborn generations with our wrong decisions (not the first time admittedly but the first time we do with full awareness probably and such a huge damage).

        1. bob goodwin

          Why is the standard rebuttal to use ad-hominums such as “you must be in the employ of the the nuclear industry.” Clearly not all commentary on one side of an argument is due to economic incentives.

          The fact that the accusation is made must be evidence of the insecurity of your own facts and prejudices, otherwise you could simply rebut the points directly.

          1. Yves Smith Post author

            You really aren’t up on the state of the art in the blogosphere and social media. Paid commentors are rife, along with employees who do it a bit on their own. Anyone new to NC who shows up and is dreadfully well informed and is hewing to industry talking points is very likely a plant. I’ve had them before on other topics.

          2. Maju

            Do you mean that I must be in the payroll of the solar energy lobby. I wish. ;)

            I even wish there was one such lobby, and hence one such industry…

            Let’s be serious: the industry has a standard of propaganda, which is repeated often, sometimes by their agents and sometimes by truly persuaded good will people without an opinion of their own (i.e. brainwashed). While you can’t spot a lobby agent (or bot, nowadays they are quite sophisticated) with 100% certainty, you can spot them with maybe 80% certainty, because they are standard issue.

            This does not just apply for this lobby, of course: other lobbies do the same thing. And I know for a fact (my once favorite cousin works in such things) that there is paid people writing opinion articles and the like, and even “objective” news items, as if it were genuine, for a salary. It’s a job, albeit an ethically questionable one.

            Nowadays they are beginning to replace them by personalized bots for what I’ve read… in this blog, mind you (well, actually via a link posted in this blog yesterday).

          3. charles 2

            Yves, I suggest that you do an experiment.

            Take all the assertions that have been put forward by all the commenters and research them carefully (I am sure you can do it and you are connected to people who can do it).
            Then classify them into :
            1 factually wrong
            2 factually right but insignificant or misleading
            3 factually right and significant

            I’ll trust that you will find more 3 and some 2s in the “pro” side and a lot of 1 on the “anti” side.

            BTW, When John L. asserts that Fukushima has not released a lot of long term radioactivity, it is technically true, but he conveniently forgets to point that the situation is still NOT STABILIZED YET. It will be over only when a closed circuit of freshwater (itself cooled by a secondary circuit) will cool all reactors and spent fuel pools.
            This one goes into bag #2 !

          4. DownSouth

            charles 2 said: “I’ll trust that you will find more 3 and some 2s in the ‘pro’ side and a lot of 1 on the ‘anti’ side.”

            I think you’re letting your own biases hang out here.

          5. bob goodwin

            I think Yves answered that John L could plausably be a nuclear energy paid advocate. I trust her judgement on that point. However what John L wrote seems consistent with my light research on the subject. I am surprised (in retrospect) at the relative low damage from Chernobyl, and the relatively low amounts of radiation released from Fukushima.

  29. dugsdale

    Just posted this at the end of yesterday’s (stone-cold) thread. Reposting here because, frankly, technical absolution of nuclear power doesn’t interest me a whit:

    First off, a link that enables finding the nuke plant closest to your zip code. (I found several plants very close by, including a GE Mark I job just like the ones currently in crisis; as I understand it, this plant has less than HALF the reserve battery power the Japanese plants had).

    http://money.cnn.com/news/specials/nuclear_power_plants_locations/index.html?hpt=C2

    And a “thanks” to Yves for creating my top “go-to” blog, and all the commenters for contributing in-kind.

    I’m not surprised that pro-nuke spokesmen are flooding the zone right now, dishing out misinformation and trying to tamp down justifiable fear and outrage, or basing pro-nuclear arguments on strictly technical criteria.

    One of the factors that led to so much pro-environment legislation in the 70s was the complete loss of faith and trust in the word of corporate classes and their legislative henchmen among the general public. I hope we’re entering such a time once again. I expect there to be legions of pro-nuke shills viciously picking apart the Japanese response, the subtext being, of course, “we’re smarter than that.” But, THEY AREN’T.

    Something about the idea of CEOs trying to wring every cent of profit possible out of running a nuclear plant, gives me the willies. That is not a trustworthy class of people to start with, and the errors the Japanese have made are part of being human, I think: folly, self-delusion, fond hopes, good intentions, wishful thinking. There may be circumstances under which nuclear power might be “safe,” but I’m doubtful we humans are capable of creating them.

  30. marketartist

    The problem with nuclear is that the ultimate risks are being passed on a hundred or more years into the future. Can anyone predict what the world will be like in a century or more? Our inability to understand complex risk has been well-documented, but our inability to predict the far future is just as serious. I believe that microgeneration technologies are much more manageable by human intelligence and circumvent our emotional biases by putting the feedback mechanisms right in our buildings and communities. It’s what we can handle well, and therefore should be our first option.

    1. marketartist

      There are agricultural approaches (such as hydroponics) which can help reduce some of our agricultural excesses, both in water and hydrocarbon use. They also can be adapted to more urban or suburban settings, reducing transportation costs. Urban farming is not a panacea but it’s a sensible step.

  31. don

    If the great mass of people are to live in urban areas, then that presumes that agriculture will remain highly energy and capital intensively industrial, which on its face is a contradiction.

    1. marketartist

      There are agricultural approaches (such as hydroponics) which can help reduce some of our agricultural excesses, both in water and hydrocarbon use. They also can be adapted to more urban or suburban settings, reducing transportation costs. Urban farming is not a panacea but it’s a sensible step.

  32. Peter Jalkotzy

    I think nuclear has it’s place. It is one of a variety of modern technologies that will continue to evolve. Along with new technologies, we will continue in some capacity to meet our own expectations of a ‘right’ to energy. For me, the reality is our world has grown up on abundant energy, quite literally from day one. We exist as we do because of the ready availability of energy, it its many forms, and by its many processes. Each choice we make to ‘power’ our world has consequences and benefits. Each of us has individual values onto which we measure those consequences and benefits, and collectively choices are made. Nobody move, and do nothing might be a ‘safety’ joke – the world would never have progressed had we subscribed to that thesis – and not my way of looking at the world. Nuclear has big risks and long term liabilities. I believe these aspects do need to be re-examined and better solutions developed. And for me, that follows cleverly along a particular path in our rapidly moving modern world – nothing is ever perfect – it never can be. There is no one size fits all anywhere for anything. The dynamisms of the modern world – and the old natural world for that matter – are infinitesimally infinite in unique combinations of events – there is no one time in history that will EVER be repeated EXACTLY as it happened the first time. So evolution, and modernization of technology will continue to learn from past failures, technologies will improve with new materials, production and maintenance, the effort will always be one in flux.

    Does this all mean we should not pursue nuclear, because of the potential for catastrophic losses from uncertainties? I think we would be doing our selves a disfavor to not continue to expand our knowledge and capacity in how we deliver energy to our world.

  33. Armchair Revolutionary

    In its current form, nuclear power should not be considered acceptable. While likelihoods of catastrophic failure are very low, the results of failure are just too costly.

    What if, however, the fuel could be delivered in a form (liquid) that would allow for efficient burning/reprocessing and the ability to discontinue the chain reaction almost immediately?

    There is clearly a difference between today’s uranium power and the real possibilities of thorium. Right now, we should be publicly investing heavily in thorium research.

  34. Maine Owl

    My belief is that nuclear power should be phased out. The argument we’re hearing now, “Fukushima is the 2nd worst nuclear disaster ever, and look, it’s not that bad!”, begs the question: Why don’t nuke firms then assume 100% of the liability for the industry? Of course not, nukes are a stunning example of socialized risk and private profit. In Japan, guess who pays for this: http://blogs.wsj.com/law/2011/03/18/japanese-taxpayers-likely-to-shoulder-nuclear-liability/. Would be the same in America. So what would replace it? A combination of distributed solar electric and wind. But that’s not feasible, you say! Yes it is: http://ieer.org/

    1. Xavier Onassis

      Thanks much for that ieer link, Maine Owl – I can’t wait to read around there and compare to the Stanford guys’ plan I’ve contributed below that (also) shows we can indeed go renewable with the technology we already have for the cost we already spend!

  35. optimader

    the incorrect assumption is that nuclear power has to be chracterized as a tight complex system, a “fail unsafe design” subject to unanticipated nested catastrophic fialure modes -victimized by unanticipated scenerios in the failure tree analysis.

    This is a fair prejudice percieved by the public in so far as there has been a longstanding complete and utter failure of the media and our “stewards” in government to inform the public of safer nuclear fuelcycle alternatives- notably Thorium.

    Rewinding the tape, the objective of commercializing public nuke utility plants was not merely to supply electricity but to breed plutonium for nuclear weapons. Had there been fidelity to the pure objective of SAFE production of electricity the political/engineering effort would have delivered Thorium fueled power plants. It was shunted becasue it doesnt breed weaponizable plutoniun, nor the bulk of teh long live transuranic waste taht future generations are burndened with as a result of our current nuke power establishment. LOng term storage of this waste IMO is the ULIMATE unfunded mandate

    It is yet another very sad story of the lost opportunity of US intellectual equity that is still playing out as the commercialization effort has been tsken up elsewhere (china india) where it is recognized as a rational alternative approach to power generation.

    An interesting irony that touches on previous threads on this blog is that cheap and abundant Thoruim is a coexisitng element and a byproduct of rare earth mining.

    http://www.telegraph.co.uk/finance/comment/7970619/Obama-could-kill-fossil-fuels-overnight-with-a-nuclear-dash-for-thorium.html

    http://energyfromthorium.com/

    http://www.youtube.com/watch?v=vEpnpyd-jbw&feature=player_embedded

  36. Greg

    Seems like the easy way to resolve the question of whether this is worth the risk is to correct the externality. Not only should nuclear power not be subsidized, not only should it pay the full costs of safely mining fuel, building safe reactors, and safely and correctly disposing of its waste, it should be taxed at a level heavy enough to compensate for the potential risk and very high cost of a disaster.

    Similarly, coal and oil should not only not be subsidized, but also taxed heavily to compensate society for the harm of the pollution.

    If we correct the externalities, companies would naturally shift to the lowest cost, lowest impact, and lowest risk sources of power.

  37. Eureka Springs

    As a hillbilly, I think the question of rural vs living like sardines is unnecessary. For it’s the hillbillies in the middle of nowhere I know who live high on the electric hog. People live entirely off the grid with no more square footage of solar panels than the size of a full size bed they sleep in. This powers everything from the well to the vacuum cleaner and the computer. And they never ever have a utility bill. Each year they travel abroad for a month or so and I look at the difference in our utility costs/zero costs and see exactly how they afford it. ANd they never ever require a Pandoras Pond for waste storage… not for one nano second.

    Renewable solar panels are being made from entirely organic materials now (Poke weed – which grows in abundance, preferring the worst of soils/conditions). India is now manufacturing compressed-air powered cars which even have air-conditioning.

    We, even on the so-called left keep asking ourselves entirely to many of the wrong questions. If something has the slightest possibility of leaving hundreds of square miles deadly toxic for millennia – it simply should not be considered. We are not that special.

  38. Max424

    Humans, you need to take back from your banks the trillions necessary for a wind power build-out on an unimaginable* scale. This is your last hope.

    And you better get started, pronto. By our calculations, you are already 25 years past the point of no return.

    * Unimaginable to the humans that matter, your psychotic and scatterbrained central planners; elite members of your species who are tripping the light fantastic, unaware that there is, at present, no alternate route to your planet’s suicide path, even for the high and mighty.

  39. noblejoanie

    Too many people on this planet, 6 billion now heading to 9 billion absent some correction–benign would be family planning, malignant war, disease, famine, nuclear meltdown.

    Why we need to talk about large centralized electric generation facilities goes back to supplying so many of us with reliable power to sustain a standard of living the planet probably can’t handle, especially as more and more scale up that ladder. I realize I’m speaking from a first-world perspective, but so long as we’re taking the big picture view, thought I’d toss this into the discussion.

  40. Low

    All it takes is one, just one “catastrophe” that can not be remedied and hundreds if not thousands of squares miles is destroyed/unusable….millions dead. Sensationalized? NO WAY, it is the truth, likely? No. Possible? yes. Can every possible safeguard be put in place and still fail? most definitely. Chernobyl was close to and well within the range of possibility of destroying all of Europe, huge portion of Russia, and Russians main aquifer with just one mishap.

    You people on here giving analogies of plane crashes and a few dieing from this or that compared to nukes are ridiculous…its not about a few people dieing..even in worst case scenario those do not destroy whole f’ing continents/countries and Millions of sick/dead when something goes wrong…..and odds say eventually, someday, it will go wrong again.

    http://video.google.com/videoplay?docid=-5384001427276447319#

    1. Wile E. Capote

      But can the destruction last 1/2 a million years or more?

      If it can’t last at least two million years, then we must conclude there can be no lasting harm.

  41. optimader

    interstingly, the uranium fuelcycle reactor failure roundups in media overlook: http://en.wikipedia.org/wiki/Windscale_fire

    This remains a ticking bomb in the UK

    The Calder Hall runnaway reaction/near meltdown remains a UK state secret attributed to being merely a turbine failure. It wasnt. Happening on the heels of Windscale disaster, the UK establishment covered this one up at all cost.
    http://hansard.millbanksystems.com/commons/1958/jul/03/calder-hall-turbine-failure

  42. gf

    It is the same formula used over and over again.

    Monster subsidies for projects that will yield giant profits for the few at the expense to the many.

    Lets just cut out the middle part and pay the nuc industry to go away. They get their entitled profits. We at least get nothing.

    Not in my fucking back yard!!!!!!!

  43. Brian

    Our desire for easy shows the anthropologist is correct. We ignore the people that master science, and listen only to the bosses running the show that spend the money for safety on bribes and lobbying (as if there were a difference) The science is not in charge, the money is.
    As long as we are willing to ignore practical science, give billions if not trillions to the oil and nuclear industry without responsible parties in control, we lose.

    We have the inventions, they are suppressed by the people that have money and want to sell oil or nuke. What if we spent the money on the science instead of spending 10% on the product, 90% on the lobbyist and bribes to the airheads making decisions.
    Imagine a trillion or so pointed at solar for 30 years. We wouldn’t have had any major discoveries to enhance that technology for 30 years if we don’t fund it. Imagine if we had.

    Sorry, the idiots are in charge.

  44. Paul Tioxon

    Nuclear power is an obsolete and unfinanceable option for electricity generation.

    1. Nuclear power is a weapon, produced as a weapon and inherent in the technology, produces nuclear bomb material as a waste byproduct, plutonium. A controlled nuclear blast has the unintended consequence of producing electricity. We are expected to willingly suspend our disbelief, that a controlled nuclear explosion, controlled only by the exceeding precision of laboratory controlled mechanisms that can not operate outside of the in vitro instrumentation, will always be under control.

    2. Passive solar designs, and solar panels, fuel cells aided by wind, geo thermal and what ever other non weaponized and non fossil fuel, renewable technologies are the only cost effective and life sustainable option.

    3. Oil, coal and gas are going nowhere, other than up in smoke, but will run out in a matter of time. Before that happens, the population that is sustained by fossil fuels will exceed the capacity of these fuels to feed, fuel, warm and cool as, as needed in the contemporary civilization that we have fabricated. Then, these fuels will gone for ever, not even available to whoever inherits the earth in the future.

    4. It is now cheaper to save energy through efficiencies to the existing fuel infrastructure than to buy the fuel to run it.

    http://move.rmi.org/files/oilmap/RMI_Oil_Imports_Final_large.html

    This chart show the cost per day in USD for oil imports and where the imports come from. We spend now, about a $1BB PER DAY ON OIL IMPORTS. Boeing’s new 787 runs 20% less jet fuel than previous models. Electric cars speak for themselves in this regard. Passive solar designed homes, with no central heating system, like an entire block built in the mid 1980’s in a working class North Philadelphia, do not draw home heating oil, gas or other fossil fuels, like wood burning or coal burning stoves.

    4. Finally, the increasing installed capacity of solar and wind for electricity is far exceeding in immediate costs and externalities any other fuel sources. Only simple design efficiencies can beat the ROI of solar and wind.

    http://move.rmi.org/markets-in-motion/case-studies/automotive/fiberforge-.html

    5. Finally, actions speak louder than words, or as Ozzie Myers, infamous Congressman from Philadelphia has said to the world, “Money talks, bullshit walks”. There have been no new oil refineries built in America since WWII and no new nuclear power plants built since the 1970’s either. Yet our economy grew and grew, based increased efficiencies to the existing infrastructure and consumer consumption product improvements, like the new electricity sipping light bulbs.

    If nuclear power was going to expand, it would have. The fact that Obama has to come up with $36BB in loan subsidies shows just what a lousy investment it is. Wall St has voted with its galactic piles of cash and the answer is no. One final crushing blow to nukes, is the Fleet Factors rule. In a landmark court decision, Fleet Bank was held equally liable for its role in financing, as the factory which polluted the ground in the process of its day to day operations as a manufacturer. As the manufacturer could not operate without the financing, Fleet was found to participate in pollution, in violation of the EPA regs. Try to get a commercial loan from a bank, without an EPA Level One survey, showing no signs of possible liability on the part of the bank. This review is as important as an appraisal on the market value for lending purposes.

    1. Wile E. Capote

      Well, the manufacturers can’t operate without customers. This is why customers should be liable for everything, and I think they are already. Non-customers too because they could become customers.

    2. John L

      1) Really? “Unintended consequence of producing electricity”? So those power plants just started making electricity by accident? Care to figure out how dangerous producing electricity by coal is?

      2) If you accept much higher costs and associated impacts (pollution from solar panel generation, lower efficiencies, etc), sure. None are sufficient or efficient enough to replace nuclear power generation

      3) Agree; oil first, then coal. Nuclear power has the potential to be somewhat self-renewing, using spent fuel reprocessing technology to create new fuel.

      4) The US conserving and becoming more efficient won’t change the fact that India, China and other developing nations will gladly buy up the oil we don’t use, depleting the reserves and driving the cost up anyway.

      5) Oil refineries don’t produce anything to generate electricity; you’ve conveniently neglected to mention we’ve added many new coal plants in that timeframe, and because we don’t have any new refineries, we’re reaching the point where we’ll be importing fuel, not just oil.

      1. Wile E. Capote

        John, I don’t think you go far enough in warning people of the imminent dangers of shining light on water towers to produce steam that turns a turbine. It’s a big hippie commie pinko tree-hugger conspiracy if you ask me!

          1. Paul Tioxon

            Please, people, I can defend myself. To my critics:

            1. EAT
            2. SHIT
            3. AND
            4. DIE.

            DELENDA EST gop.

      2. Maju

        There is no energy so costly as nuclear, John. There is no energy so polluting (each ounce of radioactive pollution/residue is forever) and no energy so dangerous. The extra costs are paid in advance to electricity companies by the states in form of subsidies and other advantages, why? because nuclear is militarily strategic and promises some degree of self-sufficiency. They are also deferred into the future in form of unpredictable accidents like this one and, specially in form of pollution and hyper-toxic waste nobody knows what to do with it.

        Nuclear is hyper-costly and ultra-dangerous, while solar and other renewables are every day more efficient and cheaper (and materials used are also more eco-friendly). No energy generation system has zero impact but no energy generation system is as bad as nuclear in all aspects.

        Besides, if everybody goes nuclear, as was being promoted by some until just a few weeks ago, there would be not enough uranium on Earth – so it’s not efficient in that aspect either: unlike solar (and other renewable) energy, which is available at huge amounts (effectively infinite), uranium is just like fossil fuels: finite and hence quite worthless for a long-term universalist solution to the energy generation problem.

        “The US conserving and becoming more efficient won’t change the fact that India, China and other developing nations will gladly buy up the oil we don’t use, depleting the reserves and driving the cost up anyway”.

        I am not talking “to the USA”, I am talking to the World. Your view is provincial, narrow and dampened in a geostrategic dialectic that must be and is being overcome by the very globalization that the USA itself has promoted so actively.

        “Oil refineries don’t produce anything to generate electricity”…

        Not sure but I get your point anyhow. Obviously there is a problem with private transport and transport in general. But there is an array of solutions going from the good ol’ bicycle and the electric car to hydrogen (electrically generated from water and producing just water as result). There are other options but anyhow the key problem is where do we get the electricity from and how.

        Because nuclear plants are not used to power cars or airplanes (and only in very few cases, ships).

  45. Mac

    I am glad somebody linked to the Thorium fuel cycle. Here is a (scientific talk – has technical terms) on LFTR (Liquid Fluoride Thorium Reactor):

    http://www.youtube.com/watch?v=WWUeBSoEnRk

    This produces *far* less waste, and that waste needs to be stored only 300 years. It will also burn the *existing* waste stored in say those spent fuel rod pools.

    These reactors don’t have a commercial push, except now the Chinese are looking into them, which might get the US interested (given the US won’t want to buy tech from China for a change).

    One reason mention on why no commercial push for LFTR power is profit. Reactors such as Fukishima and those in the US (Boiling Water Reactors and Pressurized Water Reactors) is that companies that build them (such as GE) do so according to this business principle:

    The Gillette business model

    They build the reactors for a profit of course, but the big profit comes from building and maintaining of the fuel rod assemblies – say for 40 years in the case of the above Fukishima plants.

    In LFTR plants, there are no fuel rod assemblies, thus no profit to be had.

    1. Mac

      Oh, I forgot to mention – it is *extremely* difficult to weaponize the intermediate by products in LFTR.

      1. Paul Tioxon

        Well if they are only good for commercial purposes, and can’t be weaponized, why bother? Why do N Korea, Iran and every other country start nuke power plants to begin with? Why then, does the national security crowd go ape shit over the alleged peaceful uses, claiming they can quickly be diverted to weapons programs. You know, there is technical paper somewhere showing conclusively with very advanced math that if wishes were horses, then beggars would ride. It is on the shelf next to N Tesla’s broadcast electricity system and the Royal Rife Microscope.

        1. Mac

          You mean Iran et al starting non-LFTR nuclear reactors? For the same reason the US, Russia, UK, France, China et al built non-LFTR nuclear reactors – to build bombs.

          Power generation is just a commercialized by-product.

  46. craazyman

    Just Wondering

    What happens if some crazy war breaks out between countries with nuke plants and some lunatic generallisimo priest king decides that deification is preferable to survival? I mean his picture is up on every mensroom wall. He can gaze down upon his people into enternity, keeping them forever safe, like a sheppard looks out after his flock. He’ll be The Man forever. Maybe even a God. Of course a God! That’s why he can do what he does.

    I wonder if a nuke plant could survive a modern day bunker buster. I doubt it. Or something like what they did to Dresden in WWII. Or what the Germans did to London. You really can think of lots of examples to illustrate the point.

    I doubt it. OK. Maybe they wouldn’t aim the missles directly at the nuke plant because that would be “over the top” even for a lunatic expecting he’d bring on the third coming of Mithra. So maybe they’d program the nuke plant’s coordinates into the missle launch system and code into the software: “Don’t fire it there”. That’s assuming they have a rare moment of gnostic reflection.

    But somehow I think they’d hit it anyway, if only by accident. In fact, they’d probably have a better chance of hitting it if they don’t aim at it. Because it seems like the aim is frequently off anyway. If you read the news.

    I think nuclear power is a big dice roll, unless they can figure out a way to make it lunatic-proof.

    And I don’t feel lucky.

    1. charles 2

      The “implicit” answer to that is that countries that have nuclear power plants also have military nuclear power and that therefore military attack on nuclear power plants would trigger a nuclear response. That covers most of the NPP in the world but not Germany and Japan. Safety by MAD is not a good idea in my opinion. This is why I prefer underwater plants.

  47. Hal Horvath

    “To this anthropologist, then, the lesson of Fukushima is not that we now know what we need to know to design the perfectly safe reactor, but that the perfectly safe reactor is always just around the corner.”

    Ok, so the old, *cheap*, past-planned life Mark-1 reactors which partly failed here show that a “safe reactor is always just around the corner”?

    Really?

    I’m frankly tired of reading the non-physicist, non-engineer alarmist stuff from people that frankly don’t have a clue.

    Would it be breaking the rules to have a nuclear engineer quote?

    To me, that is the minimum standard of what is worth quoting — scientific or engineering expertise — when discussing a physically technical question.

    From the angle of the quote itself as representative of the inferior popular thought, yeah, that’s worth analyzing.

    Why do so many people think other people know so little about what they work on?

    Oh yeah….heh heh.

    But, even if you don’t want to believe me, working engineers simply have an average integrity that is better than in the finance field.

    1. Wile E. Capote

      Right. But the engineer has to work in the industry. We don’t want any independent voices mucking up the debate.

      1. charles 2

        No, you can find professionals qualified in nuclear matters that do not belong to the power generation complex :
        – the military (especially navy in the US). A lot of them land in Nuclear Power plant when they reach the end of their career but not all of them.
        – the regulators who have in lots of countries the status of protected and independent civil servants, just like judges (not in Japan unfortunately). They do not have a stake in the survival of civilian nuclear power because even if we were stopping all plants today, the regulator would still be needed for decades. If you are a rational anti-nuke activist, it is the best place to go.

    2. Maju

      I know a lot of engineers, most of which (not all) are in favor of nuclear power just because that’s what their faith in technology brings them to (and probably because they think that solar power is too hippie or “gay” for their macho bourgeois psychology).

      In any case, they have no solutions for what to do with the residues and they only shake their heads in denial when faced with catastrophes like Chernobyl or Fukushima: blame the Soviets, blame the Japanese, blame mother nature, blame whatever but the fact that nuclear power is risky enough not to be worth it.

      Don’t get me started with engineers and their psycho-type… and how many of them are in denial of the damages caused not only by nuclear energy but by pollution in general. There are exceptions of course but in general they are too focused in career and money and abstract science to care about the very real risks of nuclear pollution, nuclear waste and nuclear accidents.

        1. Maju

          Forward where? To a sustainable satisfying life or to nuclear armaggedon?

          Anyhow, I am not talking of “the country”, I’m discussing the world. Your country only has 5% of all people (but over 20% of all wealth, and yet so unequally distributed that it makes me cringe). We cannot anymore design policies only for a country, small or large. These things are global and need global policies (and a global democracy, mind you).

      1. Hal Horvath

        More than one of your assertions is contrary to facts, and I don’t even feel like trying to convince you. If you don’t care to check your facts, it really is ultimately your own problem.

      2. craazyman

        I’m not an engineer, but I had a crazy idea that I think would solve the fuel storage problem.

        Why not load it all into a few big rockets and shoot them into the sun.

        That would be the ultimate in recycling. It would be a few minutes of raw nerves right after the launch, but just think of the relief when that sucker was out of earth’s gravity range and on it’s way.

        And even if we missed the sun, which would be hard to miss, it really doesn’t matter since there’s so much space out there. Like one huge trash can we can never, ever, ever fill up. LOL.

        1. craazyman

          all right. that might not fly with the squeamish types (although i’d go for it)

          so what about digging huge holes at the reactor sites, like 300 or 400 feet down and just surrounding the stuff with layers of lead and cement?

          I’d rather shoot it into the sun, personally, just because I think it’s more artistic. I don’t like having the stuff buried here on the earth. It reminds me of the suppression of a neurosis and I think it would just infuriate future generations and they’d feel compelled to dig it up and get it out of here — probably by shooting it into the sun themselves. It would be like sitting around with a splinter in your butt. Eventually you’d lose your mind.

          1. Cedric Regula

            I’m a little squeamish about all the rockets it would take.

            Seems to me the earth is an easier target to hit. Consider the surface area of a sphere is:

            A=4(pi)r(squared) where radius is 8000 miles for the earth.

            I’ll let you plug the numbers in your calculator, but the result is pretty big.

            In fact, living here in the SW, I go on airplane flights sometimes and look out the window. I go for long periods of time looking down on what looks very similar to Mars in some places. So it’s almost like shooting the crap into outer space.

            And contary to what NASA tells us, I don’t think we are ever moving to Mars. So if we can’t move to a few square miles of New Mexico for a few thousand years, who cares?

          2. Maju

            I like the idea, CrazyMan but there is a big risk (besides the costs that may also scare some): that one of those rockets fails and explodes scattering highly radioactive waste through a large area.

        2. Jojo

          It’s not practical to shoot the waste into space because that waste is damm heavy!

          =========
          DUF6 Cylinder Weight Comparisons

          A Ticonderoga-class cruiser is about equal in weight to 706 cylinders of depleted uranium hexafluoride (DUF6). It would take over 70 cruisers to weigh more than the Nation’s inventory of DUF6! The Navy owns only 27 Ticonderoga-class cruisers.
          DUF6 Cylinder Weight Comparisons

          7,142 cylinders of DUF6 weighs as much as a Nimitz-class aircraft carrier. The entire inventory of 57,634 cylinders weighs more than all eight of the Navy’s Nimitz-class aircraft carriers combined!

          http://web.ead.anl.gov/uranium/guide/facts/
          ==========

          Maybe we can drill a hole down to the center of the Earth and dump it there? :)

    3. DownSouth

      • Hal Horvath said: “I’m frankly tired of reading the non-physicist, non-engineer alarmist stuff from people that frankly don’t have a clue.”

      So physicists and engineers are the only people who have a clue?

      • Hal Horvath said: “Would it be breaking the rules to have a nuclear engineer quote?”

      “To me, that is the minimum standard of what is worth quoting — scientific or engineering expertise — when discussing a physically technical question.”

      You’re contradicting yourself. You just got through saying that scientific or engineering expertise is the only worthwhile standard, which is quite different from saying that scientific or engineering expertise is the “minimum” standard worth considering.

      •Hal Horvath said: “…the quote itself as representative of the inferior popular thought…”

      Yes, we certainly wouldn’t want to listen to anything the great unwashed have to say. All bow down to the all-seeing and all-knowing scientist kings!

      • Hal Horvath said: “Why do so many people think other people know so little about what they work on?”
      You tell me. You assume that Gusterson knows nothing of the field he works in, anthropology.

      • Hal Horvath said: “…working engineers simply have an average integrity that is better than in the finance field.”

      Right. And I suppose you also believe in Santa Clause, the Tooth Fairy and all that stuff about clicking your heels as seen in this video.

        1. Maju

          This is first of all a medical (and also ecological and economical) affair. Physicists and engineers care very little about health “side effects”. They do not care either where the money comes from to finance their dangerous toys.

          The other sector that will applaud nuclear energy are military officers and people with interests in the weapons industry.

          I’m not so sure that physicists make up a clear pro-nuclear group; unlike engineers, whose main goal in life is typically to make good bucks, physicists may be more motivated by the “philosophical” side of things, as theirs is a more widely scientific and less purely technical career.

          1. Hal Horvath

            “Physicists and engineers care very little about health “side effects”.

            C’mon.

            Think about that for a minute. Surely you have some doubt about that assertion.

  48. foghorn longhorn

    http://www.flickr.com/photos/digitalglobe-imagery/5537297305/sizes/l/in/photostream/

    http://english.kyodonews.jp/news/2011/03/79482.html

    Status of quake-stricken reactors at Fukushima nuclear power plants
    TOKYO, March 19, Kyodo

    The following is the known status as of Friday night of each of the six reactors at the Fukushima Daiichi nuclear power plant and the four reactors at the Fukushima Daini plant, both in Fukushima Prefecture, which were crippled by the magnitude 9.0 earthquake and the ensuing tsunami on March 11.

    Fukushima Daiichi plant

    — Reactor No. 1 – Operation suspended after quake, cooling failure, partial melting of core, vapor vented, building housing reactor damaged March 12 by hydrogen explosion, roof blown off, seawater being pumped in.

    — Reactor No. 2 – Operation suspended after quake, cooling failure, seawater being pumped in, fuel rods fully exposed temporarily, vapor vented, building housing reactor damaged Monday by blast at reactor No. 3, blast sound heard near suppression chamber of containment vessel on Tuesday, damage to containment structure feared.

    — Reactor No. 3 – Operation suspended after quake, cooling failure, partial melting of core feared, vapor vented, seawater being pumped in, building housing reactor badly damaged Monday by hydrogen explosion, high-level radiation measured nearby on Tuesday, plume of smoke observed Wednesday and presumed to have come from spent-fuel storage pool, seawater dumped over pool by helicopter on Thursday, water sprayed at it from ground on Thursday and Friday.

    — Reactor No. 4 – Under maintenance when quake struck, no fuel rods in reactor core, temperature in spent-fuel storage pool reached 84 C on Monday, fire Tuesday possibly caused by hydrogen explosion at pool holding spent fuel rods, fire observed Wednesday at building housing reactor, pool water level feared receding, renewed nuclear chain reaction feared, only frame remains of reactor building roof.

    — Reactors No. 5, 6 – Under maintenance when quake struck, some fuel rods left in reactor cores, water temperatures in spent-fuel storage pools increased to about 64 C on Thursday.

    — Spent-fuel storage pools – Cooling functions lost at reactors No. 1 to 4, water temperatures or levels unobservable at reactors No. 1 to 4, no immediate threat to water level at common spent fuel pool.

    Fukushima Daini plant

    — Reactors No. 1, 2, 4 – Operation suspended after quake, cooling failure, then cold shutdown.

    — Reactor No. 3 – Operation suspended after quake, cold shutdown.

    ==Kyodo

    The info on Daini still seems thin.

  49. Cedric Regula

    It’s always good to check up on how these wonderful “renewable” and proven energy alternatives are progressing.

    Here’s what happened to what was going to be the largest US wind farm installation. Also about a year before this press release, there was news that Pickens did try and get bank financing to build the connecting grid himself, but was refused financing.

    T. Boone Pickens cuts order for wind turbines, puts Panhandle wind farm on hold

    Published 13 January 2010

    T. Boone Pickens has cut his massive order for wind turbines from GE by more than half.

    The energy investor, who made wind power a key part of his plan to wean Americans off foreign oil, said Tuesday he will now take delivery of 300 turbines, which he will use for wind farms in Canada and Minnesota.

    None of the turbines will come to Texas, as originally planned.

    Meanwhile, Pickens continues his campaign to persuade Americans to use natural gas to fuel heavy trucks, rather than diesel. Doing so, he said, could cut Middle East oil imports in half.

    “You only have one resource in America that will compete with oil, and it’s natural gas,” he said Tuesday at an America’s Future Series event that benefited Big Brothers Big Sisters of North Texas.

    In May 2008, Pickens announced that his company, Mesa Power LP, would order 687 wind turbines, or 1,000 megawatts of capacity, from GE for about $2 billion. That’s about the size of a nuclear power plant.

    Those turbines were originally meant to become the world’s largest wind farm, in the Texas Panhandle.

    Now, Pickens says none of the turbines will go to Texas.

    He said the Panhandle will still lack transmission lines to carry the power when he begins to take delivery of the turbines in 2011.

    Pickens said he will build a wind farm in the Panhandle when transmission is built.

    When Pickens announced his energy plan in 2008, he offered two suggestions: Use natural gas for vehicles, and use wind for electric power.

    On Tuesday, Pickens said part of the reason wind has become less important to him is the drop in natural gas prices. Natural gas-fired power plants have become so cheap to operate that other types of plants have been displaced.

    He also acknowledged it’s not possible to use wind exclusively for power generation because of the cost of building wind farms and the fact that the wind sometimes stops blowing. Those who think otherwise he called “foolish.”

  50. Hugh

    A discussion of energy policy really needs to take a look at the broader picture of what factors determine our energy needs. I like to use a simple formula for this purpose:

    PT = ER

    where
    P is population
    T is level of technology
    E is environmental effects
    R is resource depletion

    This is pretty elementary stuff. If we currently had a population of 150 million, our energy needs would be a lot different than they are now or when we have a population of 400 million. Technology can save energy but overall large sophisticated industrial societies are huge energy consumers. Historically, higher energy use has led to both greater environmental pollution and increased consumption of finite non-renewable energy sources.

    It is another example of failure by our elites that our discussions do not take place in this broader context. Limiting population growth is a taboo, but then more generally so are all limits in our political and economic discourse. Limiting technology, say like gasoline powered cars, the idea of personal individual transportation, can’t be raised. Resource depletion? Phenomena like peak oil and peak energy are simply denied. About the only factor where there has been any legislation is the environment and it is incredibly weak.

    However my point is that it is pretty meaningless to be for or against nuclear energy without asking questions about how it fits or should fit into the structure of our current industrial society.

    1. Wile E. Capote

      Save your rationality Hugh. The only thing that matters these days is making a quick buck and hightailing it out of there.

  51. foghorn longhorn

    As an aside, could somebody point me to the pictures of Three Mile Island all blown up?
    Can’t seem to find any.
    Hal?

    1. Frank

      OK,

      Let us know when a working fusion plant is built.

      BTW, will it need taxpayer subsidies?

      Will they be able to buy liability insurance in the
      free market? The way that current nuclear plants
      do not thanks to the Price Anderson Act?

      If “new nuclear” is so safe why can’t they purchase
      liability insurance at any price?

  52. Frank

    Re co2 “replacement” by nuclear:

    “As you can see, every step of the nuclear power cycle involves the expenditure of energy derived from fossil fuels, which nuclear electricity cannot replace. Thus it is untrue to say that nuclear energy is greenhouse friendly.

    In the paper “Nuclear Power : the energy balance” by J.W. Storm and P. Smith (2005) download here, the authors calculate that with high quality ores, the CO2 produced by the full nuclear life cycle is about one half to one third of an equivalent sized gas-fired power station.

    For low quality ores (less than 0.02% of U3O8 per tonne of ore),
    the CO2 produced by the full nuclear life cycle is EQUAL TO
    that produced by the equivalent gas-fired power station.”

    http://www.peakoil.org.au/news/does_nuclear_energy_produce_no_co2.htm

    1. Cedric Regula

      Yes, yes. You can apply the same kind of analysis to steel and copper mining, processing the ore into metal, transporting it around the world in ships and trucks, and also amaze yourself that wind turbines make lots of CO2 as well.

    2. charles 2

      Storm and Smith calculation rely on obsolete and energy intensive enrichment technologies (Gaseous diffusion). Centrifuge and Laser enrichment are better by an order of magnitude at least.
      They also dismiss the possibility of breeder reactors, because it has not been yet widely deployed. They are not deployed because it is not yet cost effective to do so, not because it is impossible to make them. We still have plenty of time (30 years at least) before we need that technology. Doing it today in industrial scale would be as meaningless than going to extract oil in the North sea in the 50’s. There is a time for everything.

  53. foghorn longhorn

    Oh, but for a simpler time, when (nukiear physcisists) were still just the piano player in a whore house.

  54. Cedric Regula

    This article touches on the current state of the art using GEN III designs. We have 3 projects that just started in the US. Gen III is much more effficient, making less waste as well.

    The other thing is that GEN II of the 70s, which we have about a 100 plants in the US, had a 40 year design life. So they should be closing down any day now. They do try and get their licenses re-newed, say for 10 years, if they can prove to regulatory agencies the plant is still safe.

    Excerpt:
    Generation III-plus includes a handful of high-tech plant designs, many of which still await regulatory approval. Others, like France-based Areva’s Evolutionary Power Reactor (EPR) and Westinghouse’s AP1000 (both are pressurized water reactors) are already under construction, and they are designed to withstand exactly the crisis the 40-year-old Japanese reactors are failing to deal with, whether operators are around to trigger emergency countermeasures or not.

    “The new reactors really have a lot of features that were not available thirty, forty years ago,” says Michael Podowski, a visiting professor in MIT’s department of nuclear engineering and an expert on nuclear plant safety systems. “These new advanced reactors will employ more passive safety systems that will make them safe without any external intervention.”

    Areva is currently building four EPR reactors, two in China and two in Europe. The design includes four independent redundant cooling systems, two of which are engineered to survive an airplane crash.

    Westinghouse’s AP1000 packs a battery of passive systems that use natural air flow, gravity, and other natural phenomena to remove pumps and valves from the equation; if the plant begins to overheat these measures will automatically cool the core for up to three days with no external intervention whatsoever.

    http://www.frontsidebus.net/2011/03/can-next-generation-reactors-power-a-safe-nuclear-future/

    1. Hal Horvath

      Interesting about passive cooling for 3 days. supposing the most unlikely and overwhelming disaster, say 9.8 earthquake, etc., is the design for backup power and automation together to power an automatic (no person needs to be present) full shutdown?

      1. Cedric Regula

        This article is very light on details. I think the reactor design idea is to give the emergency response time to react. When they design the rest of the plant, then I hope the power plant design engineering firms think thru and provide the equipment and disaster response manual to the operator.

    2. sleeper

      The Westinghouse AP1000 sounds great now if we can only solve that operator error problem that caused big problems at TMI and in Japan.

      Sorry “new” nuke techology still has the same old problems – cost, the fact that all systems fail in time, and a waste that is a deadly poison for 1,000s of years.

  55. F. Beard

    Folks, before you lash out at growth, just remember that is is usury that requires exponential growth in order to pay it.

    Our banking and money system is the root of all sorts of evil. Reform that and many, many problems will go away including the environmental destruction that is funded by it, the need for socialism, extreme wealth disparity and the rat race for the benefit of the usury class and bankers.

  56. Dirk77

    I wasn’t going to weigh in, but just one thing: That article from the BofAS hits the nail on the head. By that I mean it isn’t about whether nuclear power can be made “safe” with waste cleanup on the order of decades rather than 100K years. All technologies can be made so if the benefits outweigh the costs, so there is always money for R&D—and I think there would be for nuclear power—and if safety regulations are realistic and always enforced. But this presupposes really a world of rational, wise people in charge. I think everyone who reads this blog knows that this is not the world we live in today. I am educated in the hard sciences and I am for nuclear power; however, as things stand I fear any “pro” decision coming out of DC because the record to me shows that whatever that decision will be, it will be the wrong one. I wish I knew what the answer was, but I think one part is to get people arguing about things, so they get an idea that they will always have a say no matter how bad things get. “The Future is Unwritten” as Strummer said. Thanks Yves for posting the question and everyone for your comments.

  57. Bernard

    love all Wile E Coyote’s responses. thanks, they help

    As if i could ever trust a businessman not to screw me for profit. lol. that about says it all nowadays. we have met the enemy and he is Business!

    one of the posts i read had a link to a woman from Kiev, i think, who has been taking pictures of the Chernobyl radiated areas. if anyone remembers what the link is, it’s worth visiting. elenafitova.com is what my history shows.

    show me a businessman who cares and i’ll show you a liar, thief and hypocrite.

    frankly, i think we as a planet are too far gone down the rabbit hole. unless of course, science can get us out of the mess greedy businessmen have gotten us. war, pollution, bribery, empires and all that sort of “paternalism” has made this planet a ticking time bomb for all the ecosystems no one wants to admit as being necessary to “live” and breathe with. which business has been so rapaciously destroying.

    But Science is a “liberal” commie threat to the Republican GOP. Not one of the 31 members of the House Energy committee believe in Climate change.

    ah the sweet smell of greed, courtesy of business’ ownership of America for the last umpteen years. I really am in awe of St. Ronnie for starting down the road to “Government is the Problem.”

    so much humanity.

    1. Rex

      “one of the posts i read had a link to a woman from Kiev, i think, who has been taking pictures of the Chernobyl radiated areas. if anyone remembers what the link is, it’s worth visiting. elenafitova.com is what my history shows.”

      I think this is what you are talking about. She has lots of interesting things on her pages.

      http://www.kiddofspeed.com/

      1. Rex

        ^^^^^^^^^^^^^^^^^^^^
        ||||||||||||||||||||

        *** LINK ABOVE — Is well worth a visit. ***
        ——————————————–

        Thanks for reminding me. This tread is long and getting old so I thought I’d make some noise here.

  58. foghorn longhorn

    Bernard
    When has the ‘climate’ ever stopped changing?
    Is there some special time in history where everything stood still?
    Fukushima is changing somebody’s climate/planet right now.
    The next large volcano will change it again.
    If you are for population control, which really seems to be the agenda of the AGW crowd, well just hang on.
    Just hope it doesn’t come to your neighborhood, eh?

    1. Cedric Regula

      Personally, I thought the Chinese were on to something with their one child law. But could you imagine the screams if anyone ever proposed that here?

  59. foghorn longhorn

    My spouse and I did the ‘no child’ policy.
    BFD, the world was just spared one crazy little MoFo.
    Thank me.

  60. Maju

    Just for the record, as I imagine you might be interested, the latest “conspiration theory” on Fukushima is that it was infected (accidentally, mind you) by Israel (and US?) developed industrial worm Stuxnet. This cyberwar-quality malware was designed to sabotage the Iranian nuclear program, yet it expanded beyond its intended target reaching other Asian countries (India, Indonesia, China, Japan). Some point that it’s possible that the worm was the real cause of the failure of the emergency shutdown systems, never properly explained.

    Not sure what to think but it would make some good sense. It would seem that you do not need anymore nuclear weapons to cause a nuclear disaster, even if this one happens at some other place than you intended to: you just need some malware programmers.

    1. Dirk77

      The Stuxnet worms that I know of were targeted at equipment using Siemens controllers, such as centrifuges for isotope separation. So using German hardware makes sense in a technologically backward society such as Japan. The Shogun was warned many times about this overdependency. On the other hand, creating the earthquake and tsunami to coverup the real crime would have taken some planning. I don’t think Israel and America could have done it alone. The plot and execution have the smell of Canada don’t you think?

      1. Maju

        Siemens is a highly respectable company, are you implying that the virus has the dubious virtue of forcing states and industry to purchase US (or Israeli or Russian maybe) technology? Either you buy American or you die in a nuclear holocaust?

        Or are you just being sarcastic and I fail to get it? :/

  61. skippy

    Ha! …one thing…not so sapiens must remember is, humans have a very bad habit of *concentrating risk* in order to control it, of taking that which they use, that has bad written all over it, removed from its natural state, processed in greater magnitudes of extreme potential, and locate it in nodes of convenience…globally.

    One good solar flare, a chain of natural disasters. war, and control is lost, containment of all that bad is released, chemical plants, storage facility’s, transient shipments, mining mitigation, military and civilian nuclear equipment, advanced biological product, lol human effluent systems, all that we have wrought during a peaceful period on this planet, et al, like butterfly’s released.

    Ah but what is done today with out consequence or sound or sight of it…is tomorrows problem…I don’t get payed for that stuff.

    Skippy…fractals baby…fractals…10% fail ratios become 100% inverted risk gradient…cough reestablishing control…I have my towel, do you.

    PS. whilst great minds work to further develop a general theory of cosmic relativity, can some divert and establish the one for *reality*…please!

  62. ebear

    I’m more concerned about the next Ice Age. We don’t seem to be making any preparations for that.

    ebear

  63. eric anderson

    Oddly, many people who do not trust the government to regulate nuclear power seem to believe the government can be trusted to manage a national health care system.

    Is national health care worth the risk?

    1. skippy

      Sigh…Its not government ie structure, its the people put there by monied interests. If you can’t keep undue influence out of the common discussion, what makes you think taking the leash off private will do once unfettered by all *pretense*…cough…the system as it is now!

      Skippy…handy tips for petroleum by-products removal from ones papillae is available upon request.

    2. Anonymous Jones

      Why do we trust the government with capital punishment?

      I could do a thousand of these. Seriously, let’s go.

      As to your point, why would we trust unregulated, profit-seeking corporations with our health care?

      I’m sorry, but not really seeing how the alternative is so much better. Isn’t that the real question? Ascertaining what the alternative is? Sure, everything is terrible, until we look at the alternative. Democracy is the worst form of government except for all the others. [obviously, not my own insight]

      How does it work in Sweden? We could at least do studies on actual comparative studies, no?

  64. Paul Tioxon

    Oddly, the government, when it expands medicare to all in the coming decade, will not manage health care, it will finance it through the economies of scale of a single payer system, where doctors will practice medicine, instead of book keeping and insurance administration, and hospitals will have administrators to manage the health care with the money collected by the medicare fund. The government will continue to take over when the private sector collapses. It is the civilized thing to do. Please tell me, what is the effective demand for nuclear disasters? Is there a futures market we can start to manage this risk. Reply.

  65. KC

    I would subscribe to what environmental economists would think:

    1. The extraction cost, including destruction cost, of the environment should be fully incorporated into the costing model of nuclear energy. Given the high cost (over a wide area) over long periods of time associated with an incident, nuclear power extracts a higher cost than its counterparts in lifecycle analysis.
    2. The way forward is to reduce power consumption and needs by changing our lifestyle. Lifestyle change needs several factors coming together, one of which is the appropriate transport infrastructure. We have to just go for it.

  66. Rex

    From Wiki page “Radioactive waste”
    http://en.wikipedia.org/wiki/Radioactive_waste

    “Certain radioactive elements (such as plutonium-239) in “spent” fuel will remain hazardous to humans and other creatures for hundreds of thousands of years. Other radioisotopes remain hazardous for millions of years. Thus, these wastes must be shielded for centuries and isolated from the living environment for millennia.”

    From this web page: http://www.philosophyforum.net/HistTimeline_files/HistTimeline.htm

    “Between 450,000. and 200,000 years ago.” — “H. erectus evolved into H. sapiens.”

    So we just need to package up the large quantities of nuclear waste that we are generating, safely, for a period of time on the order of, or perhaps longer, than our human species has existed.

    Well, that should be totally easy! Of course, sometime soon there will be new reactors that can turn the worst of the waste into safer stuff that would be easier to hide for ever and ever. OK. Where are the flying cars they were telling me in the 50’s I should have by now?

    A few years of electricity for my lights and TV are clearly worth stashing the resulting deadly stuff so no life forms will tangle up with it for a length of time where any surviving humans may evolve into a newer, better species. I can’t see any problem with that devil’s bargain.

    1. Maju

      Yes, we need to find a solution for all those radioactive materials FOREVER. The best is probably to dig a huge hyper-deep holes in the Sahara or the Gobi deserts and bury them there.

      But the issue is that we do have a HUGE LONG-TERM PROBLEM and not any long-term solution with nuclear energy.

      The only alternative is surely to make a nuclear holocaust and destroy Humankind (and a lot of other living beings) and hope that Earth will eventually be able to recover… without us.

      All the costs of all those nuclear products are never factored when the costs of nuclear energy are evaluated. The costs of the tremendous health and environmental hazards we incur in are not factored either.

      Choosing nuclear is a political decision meant to establish the basis for nuclear weaponry. There may be other considerations but that is the main one, even if unspoken.

  67. Allotropy

    There was once a beautiful lady, whose habit it was to sleep on disused railroad tracks.

    In that same country there lived also a brutal surveyor who ran the trains up and down the tracks. He was at heart an explorer and therefore was particularly attracted by those branches of the railroad system where no trains had passed within living memory. These were precisely those tracks where the lady delighted to slumber.

    So it happened over and over again that she would be disturbed in her sleep and compelled to retreat hastily while a powerful and smelly engine dashed over the very place she had been happily resting.

    Every time this happened there was a falling out between the lady and the gentleman. He maintained that she was an old-fashioned, trivial, and superstitious thing. She, in return, would spit out insults in a quite unladylike manner saying that he was indeed a thing, subhuman, and nothing but a small boy interested only in silly noisy toys.

    And so it went on. For about two thousand years she would always be finding new and unexplored parts of the railroad system upon which to sleep and he always choosing those very branches of the tracks for the exercise of his monstrous vehicles.

    He asserted that it was his right – and even duty – to map the railroad system and that the whole system was entirely his – especially the unexplored parts of it. He argued that the system was a single, entirely logical-causal network of tracks.

    She averred that the tracks were designed for the rest and peace of the human soul and cared nothing for his dreams of causality and logic.

    He mapped every detail of the tracks along which he ran his engines. She continually found other parts of the system not yet mapped.

    One day the engineer carelessly left one of his maps beside the track and the lady found it. Gingerly, holding it only with the tips of her fingers, she picked it up. She handled it as if it had been left there by the devil.

    It was curiosity that led her to open the map, unwilling to see what it might contain and therefore not really looking at its details. Looking at this from a distance through half-shut eyes, she was surprised to find that thus half-seen, the document was in itself beautiful.

    At the next confrontation between herself and the engineer she said without thinking, ‘And you don’t even know that your own maps are beautiful.’

    At this the surveyor was amazed. He gruffly replied that he was not interested in that.

    She said to herself ‘Ah, then there is something in the universe in which he is not interested. That something belongs to me.’

    ‘For ever,’ she said.

    After they parted, each considered what had been said. The surveyor was forced to agree that indeed the beauty of his maps and correspondingly the beauty of the railroad tracks were not within his province. She, on the other hand, was delighted and hugged to herself the secret knowledge that he would never invade what she most valued – the elegance and symmetry of the total system. Not its details but its foundations.

    At their next meeting he asked whether she was still interested in the so-called beauty of the maps. When she rather defensively replied in the affirmative, he said in an offhand manner that he had perhaps something to show her.

    He then confessed that while she slept upon the railroad tracks he had come quietly and had made a careful drawing of her body. It was this drawing that he wanted to show her.

    He unfolded and placed side by side before her his map of the railroad tracks and his drawing. He said it was ‘scientifically interesting’ that the map and the drawing appeared to resemble each other in many ‘formal’ characteristics. He specially wanted her to see this strange resemblance between the two documents.

    She briefly dismissed the matter. She said she had always known *that*. But, saying this, she looked away and smiled.

    “Allegory” – Gregory Bateson

  68. bob

    Tossing this around-

    A lot of material for nuclear already exists. Why not get some use out of it? We are going to have to safeguard it either way.

    There are no current long term storage options.

    Newer plants can take into account waste streams, and not produce as much, or use the material more efficiently.

    That much fuel sitting around, and requiring that much energy to “cool” would seem to be an energy source, not an engergy sync.

    I think that concentration of power generation is also problematic. I have seen estimates that 50% of the energy produced in the US if “wasted” on transmission.

    This does dovetail very well with “green” initiatives. Newer production and distribution systems will be required. Yes, the dreaded “smart grid”.

    Ultimately the problem is in the monopolies that most states have “granted” power companies, in both generation and distribution, two very different entities. They have given government guarantees of revenue, and any cut from the revenue will be fought over.

    Where I live a very friendly UK company, National Grid, owns distribution rights for electricity and gas. I am less than 60 miles from 3 big nukes, and pay THE highest electrical rates in the country.

    This is also taken against the backrdrop of the GPB loosing about 40% of it’s value over the past few years. National Grid now makes 120-140% of what it did 3 years ago, their accounting is in GBP. They want another rate cut.

    And those tree nukes 60 miles north? Perfect place for them. Very stable tectonically and environmentally. The residents all supplied major support when Entergy applied for permission to build another. Entergy decided against going froward.

    But, the other producer in the area is gas- Hydrofracking for gas. Major problems with the waste streams. Radioactive also. Marcellus shale has a ton of variable radioactive waste. In this case it is mixed with huge quantities of water.

    Water table damage is most notable. This area has never wanted for clean water, unlike the rest of the world. Pulling down on the water table to push into the earth, then adding to the water table thousands of feet below surely won’t have any effect of the long term hydrology of the area. Add to that the problem of them adding several percent “refinery waste” to the stuff they shove back down.

    On one hand we have an earth quake that preceded a nuclear meltdown, on the other we have gas drilling that preceded a series of earthquakes in Arkansas. To be more accurate, we had the waste stream of the gas drilling in another state causing a series of earthquakes.

    All of these problems have to be thought out, only all of them are and were put in place by monied interests who are not about to give them up. They bought them.

  69. Demented Chimp

    Civilizations and civil order ebbs and flows.
    Any period of instability turns these installations into unstable sources of dangerous pollution.
    Look at the soviet union after the breakup people went in and dismantled stations for scrap metal.
    Doesnt matter how well engineered they are are dangerous unless protected by a functioning technologically active state. If we do slip back a notch or two (quite likely) these sites will become nasty sources of silent poison – as if we dont have enough of these already.

  70. V

    Aside from the irony that a huge portion of the ‘blogosphere’ exchanging barbs at each other is powered in part by nuclear energy, what I think will ultimately happen is systems will be improved and alot will be learn’t from this disaster.

    Ultimately I take heart that nuclear engineers are ultimately incentivised to ‘build a better mousetrap’, and unlike financial engineering there is no upside to producing a reactor design solely to short it.

    1. Maju

      First Law of Murphy: if something may go wrong it will go wrong.

      There’s no such thing as a 100% safe nuclear reactor – or any other device but hardly any other device is so extremely dangerous.

      The next nuclear catastrophe may happen tomorrow or in 40 years but IT WILL HAPPEN for sure unless we make sure that it cannot happen by closing all reactors and storing all the residues in some safe place(s).

      1. John L

        Where’s the safe place, Maju? You believe in “Murphy’s first law”, so what if a meteorite strikes the fuel storage location? What if someone steals a nuclear weapon and detonates it at this “safe storage place”? For that matter, if you believe that “law”, then I suspect you don’t operate any machine more complicated than a lever, because after all, “if it can go wrong, it will go wrong”, right? You avoid cars, planes, elevators and anything electrical?

        Please, spare me the Luddite approach to living. Nuclear reactors are as safe an industry as anything we humans operate. Hydroelectric dams have killed far more people than nuclear power has; coal power generation has polluted far more land than nuclear power has. The consequences of a natural gas storage explosion is very similar to that of a nuclear explosion. Do you rant and cower in fear of these as well, and lobby to have them decommissioned too?

        1. Maju

          There is no “safe place” and that is more of a reason to stop processing uranium and plutonium right now. Yes or yes?

          We already have a problem we should not have, a most difficult problem and a problem that lacks a truly good solution.

          IMO the less bad place to store all that radioactive junk is in the middle of the Sahara, because Africa has a very old and stable plate (excepting the Eastern Great Rift and the crash zone of the Mediterranean) and the Sahara is nearly empty of people, minimizing the risk. However it sounds even to me, as a colonialist endeavor, because Africans have not developed nuclear reactors. So maybe other less optimal locations have to be found.

          Whatever the case it is a big big problem. No need to make it even bigger.

          As for “Murphy’s law” it was a simple way to explain the statistical inevitability of accidents. If you have oil tankers, many of them, sooner or later one will crash, and, if you have nuclear reactors, sooner or later one will fail. And these accidents imply extremely severe consequences, notably in the case of nuclear plants. The risks are simply not assumable, more so as they are FOREVER (an oil disaster affected area may be recovered with patience maybe but a nuclear-damaged one never ever recovers).

          If we stick to nuclear, it’s just a matter of time until all Earth is radioactive and uninhabitable. It may take centuries at the current rhythm, but it should happen anyhow. And we cannot guarantee that the rhythm of accidents does not increase, maybe a lot, because of social unrest, wars, cyber-attacks or whatever other unpredictable side causes.

          You have to integrate the risk of accident in the costs. Will an insurance company guarantee a nuclear power plant against all risks (no cap)? No way! Because the real risks cannot be assumed. If Fukushima is not insured, who is going to pay the damages, how was it allowed to be built in first place? I cannot have a car or a home without insurance, how can a nuclear plant exist that way when it’s such a brutal risk?

          As for having a “Luddite” approach, that is totally false. I favor technology, but the right technology: the one that works and not the one that hurts, as does nuclear. And anyhow Luddites destroyed machines for reasons of employment, not for environmental ones. They had their reasons and the problem has not been solved… but that would be another debate.

          “The consequences of a natural gas storage explosion is very similar to that of a nuclear explosion”.

          Absolutely false. The explosion as such may be similar but the consequences after the catastrophe are totally different: methane does not produce radioactive poisoning and we can rebuild right after the mourning. It’s bad but it’s not something we can’t overcome.

          Send my regards to your bosses in the Nuclear Lobby for offering me this opportunity to debate with such a reasonably ok spar. But you can’t win this debate because the matter is objective: nuclear sucks: it causes immense long term problems and solves nothing that cannot be solved by other approaches.

          The only true reason for nuclear energy is nuclear weapons. That’s it.

          1. Maju

            As long as it is platonic, I’m ok. :p

            I’m a boringly straight slightly nerdy rather fat man, by the way (so you know). ;)

    2. Hal Horvath

      Yes, irony prevails. You can just about judge the quality of thinking as exactly inverse to the level of personal animus/projection/characterization.

      It never fails.

  71. SteveA

    It is much too early to tally up economic costs of this disaster. Today the Japanese government announced that the plant will be decommissioned. The cost of this asset loss will be born by the shareholders (and quite likely bondholders) of TEPCO, as will at least a portion of the clean-up costs.

    The unknown is obviously the negative externalities for Japanese businesses, taxpayers, and consumers. It’s simply too soon to know. The other day, Chinese customs agents turned away an equipment shipment from Japan owing to detected radiation. This may be an isolated incident, or it may be the harbinger of a catastrophe for Japan’s export markets. Nissan has announced that all cars leaving its production line will be tested for radiation; clearly they fear foreign consumers will desert them. It is not inconceivable that air and sea freight shipments to Japan may be interrupted by radiation concerns.

  72. Xavier Onassis

    It’s beyond stupid to risk the very ability to survive on planet Earth unnecessarily, isn’t it?

    Stanford Report, January 26, 2011
    The world can be powered by alternative energy, using today’s technology, in 20-40 years, says Stanford researcher Mark Z. Jacobson

    A new study – co-authored by Stanford researcher Mark Z. Jacobson and UC-Davis researcher Mark A. Delucchi – analyzing what is needed to convert the world’s energy supplies to clean and sustainable sources says that it can be done with today’s technology at costs roughly comparable to conventional energy. But converting will be a massive undertaking on the scale of the moon landings. What is needed most is the societal and political will to make it happen.

    By Louis Bergeron:
    If someone told you there was a way you could save 2.5 million to 3 million lives a year and simultaneously halt global warming, reduce air and water pollution and develop secure, reliable energy sources – nearly all with existing technology and at costs comparable with what we spend on energy today – why wouldn’t you do it?

    According to a new study coauthored by Stanford researcher Mark Z. Jacobson, we could accomplish all that by converting the world to clean, renewable energy sources and forgoing fossil fuels.

    “Based on our findings, there are no technological or economic barriers to converting the entire world to clean, renewable energy sources,” said Jacobson, a professor of civil and environmental engineering. “It is a question of whether we have the societal and political will.”

    He and Mark Delucchi, of the University of California-Davis, have written a two-part paper in Energy Policy in which they assess the costs, technology and material requirements of converting the planet, using a plan they developed.

    http://news.stanford.edu/news/2011/january/jacobson-world-energy-012611.html

    “This really involves a large scale transformation,” he said. “It would require an effort comparable to the Apollo moon project or constructing the interstate highway system.”

    “But it is possible, without even having to go to new technologies,” Jacobson said. “We really need to just decide collectively that this is the direction we want to head as a society.”

    Jacobson is the director of Stanford’s Atmosphere/Energy Program and a senior fellow at Stanford’s Woods Institute for the Environment and the Precourt Institute for Energy.

    1. Maju

      Every time people make an investment they are thinking in the future, and that includes having children, which is a “long term” investment into at least 20-40 years or just indefinitely to the future.

      People, specially the wealthy, spend a lot of money in the education and well being of their children… and that is investing in the kind of project that Jacobson and Delucchi are proposing in terms of time into the future.

      But there are other reasons than mere sustainability or economy behind nuclear: it is of high military interest. That’s why nuclear energy is not being dropped anywhere in spite that it’s so obviously uneconomic: it is such a mighty source of power (destructive power, power of fear) that those who crave and most who hold power are enslaved by it.

      The kind of social transformation that such change requires is really radical. It does require to begin with a true participative democracy across the World (and that probably implies some form of socialism, i.e. no more private accumulation of power in form of monetary wealth). I have always sustained that “ecofascism” is impossible, an oxymoron in one single word. If we want a sustainable World we need radical democracy across it.

      At some point class war, the struggle for a genuine democracy and the fight for an environmentally friendly yet technologically advanced Humankind, converge. And we are more or less at that point.

  73. Captain Teeb

    I find the question to be silly. Regardless of the opinion of any person, nuclear power plants will continue, for the same reason that debt continues to pile up. The builders enjoy benefits today, at the expense of those who live elsewhere and in the future. As a race, how often do we forgo benefits now for the sake of the future? The future costs are irrelevant to today’s political calculations.

  74. Xavier Onassis

    http://www.globalresearch.ca/index.php?context=va&aid=23764

    That link goes to an excellent debunking of a highly inaccurate and misleading article by a nuke promoter name of Dr. Oehmen that is unfortunately making the rounds. “Nuclear Apocalypse in Japan” by Keith Harmon Snow deftly exposes the major deceptions being spread.

    Mr. Snow (read his creds in the article) was pro-nuclear until he investigated deeply the destruction of documents, the disappearing of evidence, the falsification of reports and records, the calculated fudging of risk and safety assessments. There have been countless exposes, such as Daniel Ford’s 1982 book The Cult of the Atom: The Secret Papers of the Atomic Energy Commission. Snow says we are “being denied the truth in keeping with more than sixty years of secrecy and denial by the cult of the atom and its incestuous cult of intelligence.”

    A few snips of the kind of info Snow gives, to entice all to read the whole article:

    “Daily contamination releases into water, soil and air occur at every operating nuclear reactor in the world. There is no ‘disposal’ of deadly nuclear toxins that now exist to perpetuity, and yet wastes are typically dumped on poor communities like Barnwell, South Carolina, or native American lands.”

    “Embrittlement of nickel-based superalloys that comprise reactor internals was flagged as a major safety issue as early as the 1960s, yet such problems were bureaucratically dismissed, covered over, buried in paperwork and regulatory studies produced by the NRC (“NUREG” documents), and ignored. Intergranular stress corrosion cracking of BWR core shrouds (the core shroud is next to fuel rods deep inside) is another major safety issue in GE designed BWRs built by Hitachi at Fukushima, and these plague every BWR reactor in the U.S.”

    “Nuclear advocates deride and dismiss public ignorance about radionuclides like, for example, the noble gases. Nuclear advocates frequently state that both xenon and krypton decay and disappear in a matter of seconds or minutes. What they don’t tell us is that these isotopes decay into daughter isotopes that are extremely deadly emitters…”

    I’ll stop there, but definitely worth the short time it takes to read it all.

    Also, from Dr. Helen Caldicott:

    http://www.globalresearch.ca/index.php?context=va&aid=23663

    “Nuclear power, apart from nuclear war, is the greatest medical threat posed to life on this planet. Nuclear waste will last for 500,000 years, and there is no safe means to prevent these radioactive elements from entering and concentrating in the food chain. These elements, which are tasteless, odorless and invisible, are highly carcinogenic and mutagenic. Over time, they will induce epidemics of cancer and leukemia. This is particularly true for children, who are 10 to 20 times more radiosensitive than adults, and are therefore much more susceptible to cancer.

    The nuclear waste will also induce epidemics of genetic diseases and congenital abnormalities in humans (as well as in animals and plants) for the rest of time. This is rather like a process of random compulsory genetic engineering.

    Because of the quantity of fossil fuel used to mine, separate and enrich uranium, and to build the massive reactor, a nuclear power plant has to operate for 18 years before one net calorie of energy is realized! And the CO2 produced from the fossil fuel adds significantly to global warming. Uranium enrichment also produces 88% of the CFC gases emitted from the U.S., a potent global warming gas and a significant ozone depleter in the upper atmosphere. The production and release of CFCs has been internationally banned in the Montreal Protocol, but the U.S. nuclear industry openly flouts this international treaty.

    So nuclear power causes global warming, ozone depletion, epidemics of malignancy and the destruction of the delicate process of evolution.

    Of course there are alternatives. Solar power is ubiquitous but almost untapped because of the political power of the oil companies. There is also enough wind west of the Missisippi to provide the whole of the U.S. with energy. It’s worth noting that Europeans use half as much energy as Americans, but have the same standard of living because they practice pragmatic conservation.”

    Seriously, what is left to argue after knowing this bit:

    Because of the quantity of fossil fuel used to mine, separate and enrich uranium, and to build the massive reactor, a nuclear power plant has to operate for 18 years before one net calorie of energy is realized!

    I would love to hear from the nuclear defenders the very good reason we should choose to continue nuclear power when we already know we can switch to renewables for no more cost than we are currently spending and with technology we already have.

    1. Maju

      What really called my attention, Xavier, is a hidden link at the bottom of your first link: an LA Times article on the direct victims of Fukushima I, who are being left on their own.

      Particularly revealing is how the lies of the pro-nuclear field, notably the authorities, are revealed by this hecatomb:

      “I’m disgusted by the whole thing,” Kori said.

      “We were told our whole lives that the nuclear plant was safe,” he said. “They told us even if there is a big earthquake or tsunami, it will never collapse. It all turned out to be lies.”

      This is the price you pay for trusting the pro-nuclear camp. Beware!

  75. sherparick

    One can pretty while compute the risk of nuclear power plants since they have in operation in significant numbers for 40 years. Right now there are 442 plants. Let’s give a average of 20 years in operation (most of the U.S., Russian, and West European plants are older, most China, India, and others are younger. That works out a total of 8840 years of operation and we have had three major accidents. One, Chernobyl, was a total melt down and created a 100 square mile no man land, although a fine wilderness area), and with various estimates of about deaths and illness as the result of the accident (4,000 according to the WHO, although sadly, there has not been much official interest in actually tracking cancer rates in Europe that might be affected by Chernobyl. http://www.who.int/mediacentre/news/releases/2005/pr38/en/index.html

    This has to be compared and contrasted with the harm that coal burning for electric power causes, since Nuclear Power is usually used in lieu of coal. Besides the AGW effects of coal being the worse of the fossil fuels, burning coal means you have no risk of a high impact, if largely local, disaster, that Three Mile Island, Chernobyl, and Fukishima. However, burning coal is like a permanent, low level, disaster health wise, including mercury and radiation release and the he problem of handling the coal ash waste, plus the local pollution problems burning coal causes. http://www.cacefindiana.org/node/programs/cieep/global_warming/health

    If I had to choose between nuclear power and conservation, solar, and wind, I would of course choose the conservation, solar, and wind alternative. But all power sources, including solar and wind, have negative environmental impacts, including mining for rare earths. And if the choice is between nuclear and coal, I think based on what I know now, I would choose nuclear and realize that even with upgrades and using Thorium in place of Uranium, there will be accident and the problem of waste disposal (which is more a problem of politics than science as nuclear waste long ago could have placed in underground salt formations, rather than the abandon Yucca Mountain site, a dormant Volcano in very seismically active region (See John McPhee’s “Basin and Range” for the geological history and future of Yucca Mountain). I think we need to keep Nuclear in the picture. The far from establishment “Bulletin of Atomic Scientist” is a good place to examine. http://thebulletin.org/web-edition/op-eds/facing-fears-facts-and-reason-managing-energy-risks-after-fukushima

  76. Xavier Onassis

    http://www.globalresearch.ca/index.php?context=va&aid=23764

    That link goes to an excellent debunking of a highly inaccurate and misleading article by a nuke promoter name of Dr. Oehmen. “Nuclear Apocalypse in Japan” by Keith Harmon Snow deftly exposes major deceptions being passed around.

    Mr. Snow (read his creds in the article) was pro-nuclear until he investigated deeply the destruction of documents, the disappearing of evidence, the falsification of reports and records, the calculated fudging of risk and safety assessments. There have been countless exposes, such as Daniel Ford’s 1982 book The Cult of the Atom: The Secret Papers of the Atomic Energy Commission. Snow says we are “being denied the truth in keeping with more than sixty years of secrecy and denial by the cult of the atom and its incestuous cult of intelligence.”

    A few snips of the kind of info Snow gives, to entice all to read the whole article:

    “Daily contamination releases into water, soil and air occur at every operating nuclear reactor in the world. There is no ‘disposal’ of deadly nuclear toxins that now exist to perpetuity, and yet wastes are typically dumped on poor communities like Barnwell, South Carolina, or native American lands.”

    “Embrittlement of nickel-based superalloys that comprise reactor internals was flagged as a major safety issue as early as the 1960s, yet such problems were bureaucratically dismissed, covered over, buried in paperwork and regulatory studies produced by the NRC (“NUREG” documents), and ignored. Intergranular stress corrosion cracking of BWR core shrouds (the core shroud is next to fuel rods deep inside) is another major safety issue in GE designed BWRs built by Hitachi at Fukushima, and these plague every BWR reactor in the U.S.”

    “Nuclear advocates deride and dismiss public ignorance about radionuclides like, for example, the noble gases. Nuclear advocates frequently state that both xenon and krypton decay and disappear in a matter of seconds or minutes. What they don’t tell us is that these isotopes decay into daughter isotopes that are extremely deadly emitters…”

    I’ll stop there, but definitely worth the short time it takes to read it all and get the real technical info.

  77. Xavier Onassis

    Maju, you said somewhere above that this is first of all a medical affair – and Dr. Helen Caldicott certainly agrees:

    http://www.globalresearch.ca/index.php?context=va&aid=23663

    Dr. Caldicott: “Nuclear power, apart from nuclear war, is the greatest medical threat posed to life on this planet. Nuclear waste will last for 500,000 years, and there is no safe means to prevent these radioactive elements from entering and concentrating in the food chain. These elements, which are tasteless, odorless and invisible, are highly carcinogenic and mutagenic.

    Over time, they will induce epidemics of cancer and leukemia. This is particularly true for children, who are 10 to 20 times more radiosensitive than adults, and are therefore much more susceptible to cancer.

    The nuclear waste will also induce epidemics of genetic diseases and congenital abnormalities in humans (as well as in animals and plants) for the rest of time.

    This is rather like a process of random compulsory genetic engineering.

    Because of the quantity of fossil fuel used to mine, separate and enrich uranium, and to build the massive reactor, a nuclear power plant has to operate for 18 years before one net calorie of energy is realized!

    And the CO2 produced from the fossil fuel adds significantly to global warming. Uranium enrichment also produces 88% of the CFC gases emitted from the U.S., a potent global warming gas and a significant ozone depleter in the upper atmosphere. The production and release of CFCs has been internationally banned in the Montreal Protocol, but the U.S. nuclear industry openly flouts this international treaty.

    So nuclear power causes global warming, ozone depletion, epidemics of malignancy and the destruction of the delicate process of evolution.

    Of course there are alternatives. Solar power is ubiquitous but almost untapped because of the political power of the oil companies. There is also enough wind west of the Missisippi to provide the whole of the U.S. with energy. It’s worth noting that Europeans use half as much energy as Americans, but have the same standard of living because they practice pragmatic conservation.”

    end snip

    Seriously, WHAT is left to argue after knowing renewables are possible with current technology and for no more cost than we already spend, plus this bit added:

    Because of the quantity of fossil fuel used to mine, separate and enrich uranium, and to build the massive reactor, a nuclear power plant has to operate for 18 years before one net calorie of energy is realized!

    Nuclear power simply makes no SENSE.

    1. Maju

      Excellent catch, Xavier.

      Not only boars in Germany are today still gathering more radiation than what allows them to be sold to the public but also often sheep in Scotland. Restrictions on food commercialization based on radioactivity still stand 25 years after Chernobyl.

      We can, I guess, discard them but it’d be irresponsible. Still we are surely getting quite more radiation than we should with unclear consequences.

      For some this seem to be an assumed risk, but neither states nor electric companies nor advocates of nuclear power will pay them (us) due indemnities for the damage caused. Not that I’d settle for money but they do not even pay us for the risk: we pay them instead.

      It’s a murderous ripoff. Sometimes I feel I’m living in some sort of hell, really.

  78. tz

    As others have pointed out, the question ought not be “Is nuclear power worth it (using technology when the telephone was a huge desktop appliance with a dial, or if you could afford it, touch tone)”, but “Which nuclear powered technologies are worth it”.

    Other technologies aren’t as green as might be assumed – windmills kill birds, so they want them banned in CA. Or off the shores where there is wind because they spoil the view. Hydro makes a lot of property underwater and I don’t mean mortgages – could the three gorges dam (I so want to misspell it :) be built here in the USA with the endangered species act and the EPA? We can’t even drill in a few square miles on a barren section of the Arctic tundra with limited biodiversity.

    This is an economics blog. If you make people poorer by raising the price of energy much higher, people will die. People died in Chicago when the temperature went too high and they couldn’t afford air conditioning or electricity, much as homeless freeze to death in the winter. Food – at least unless we are to return to the farm is energy intensive. Just let people freeze or starve to death? Only at the margins, you understand.

    We have chosen to use other power sources. The dead in Iraq and our intervention in the oil states are part of the casualty count. Those dead from coal pollution are too.

    But too much of this is like Pfizer – they can lie, cheat, and steal, and aren’t held accountable (small fines are a cost of doing business). Electricity becomes just another transfer from the taxpayers to blessed corporations – can we ever stop trying to recreate the East India Company? There is nothing “free market” about it, no more than the TARP banks.

  79. David Doty

    Anyone interested in serious analysis of nuclear issues should check out the Nuclear Green site http://nucleargreen.blogspot.com/ . Yes, its primary author, CJ Barton, Jr., does maintain an irrational position on all renewables other than nuclear, but otherwise his analysis is always sound and extremely well informed. He’s a second-generation brilliant nuclear physicist. The site truly is “best in class” — if you just ignore everything he has about wind and solar.

    There is a recent re-posting there of one of his father’s last reports to the National Academy of Sciences in the 1970’s, which shows that the general public is exposed to far more radiation from “natural” sources than from everything associated with the nuclear industry.

    In many cases, the largest “natural” source of radiation is radon, delivered to the homes via natural gas.

    This report was never published by the NAS. It was clearly suppressed. The natural gas industry would not tolerate it.

    Anyone reasonably well informed has known for many decades that coal results in far more radiation exposure to the public than nuclear energy. That message has been driven home by the anti-coal forces. But almost no one knows that natural gas was a larger source of radiation exposure than the nuclear industry forty years ago, and the disparity keeps increasing as natural gas usage increases.

    Anyone really concerned about the effects of radiation exposure on public health should be up in arms about the free pass being given to the radon in shale gas.

    Another unmentionable for shale gas is that much of it contains over 20% CO2, and the CO2 fraction can sometimes exceed 70%. At 40% CO2, it’s no better than coal because of the penalties associated with the separation and sequestration processes.

    I’ll also chime in on fusion. I’ve shown here
    http://dotyenergy.com/Markets/Fusion.htm
    that even if development goes as projected by fusion advocates, fusion energy in 2040 will be 400 times more expensive than wind energy.

    Finally, on the related subject of transportation fuels, the oil industry continues to insure that DOE R&D funding is not directed to anything that could really give them competition. There is a real, viable solution that has been clearly laid out for several years, but languishes because the DOE is beholden to the fossil fuel industry. We have shown very clearly that it will be possible to synthesize all the transportation fuels we need from CO2 using off-peak wind or nuclear energy, and these Windfuels will easily compete when oil is above $70/bbl. The technical and economic details are clearly presented at http://dotyenergy.com . We suspect the DoD will take notice before long, as the need for energy security is becoming increasingly appreciated.

    1. Hal Horvath

      Thanks. Nice post. I very much agree with the general concept of using off-peak windpower to create stored energy. A way that occurred to me in Texas, which has such hot summers, and a giant wind farm near Sweetwater, was to work on ideas like:
      Using airconditioners overnight during summer to cool water in storage tanks (for individual buildings and even homes perhaps), which in turn could be used for cooling during the heat of the day using a simple circulation through a pre-cooler in the A/C unit.

      1. Hal Horvath

        You’d do the above using various “smart” technologies, such as turning on the units remotely from a command center. As a customer you just get the equipment and sign up, and then the center activates your chiller when it is efficient to do so.

        1. David Doty

          Thanks Hal. Yes, there’s actually a company already commercializing essentially what you just suggested, Ice-Energy, http://www.ice-energy.com/ .
          A worthy concept. We’re just planning to take energy storage another step further, and use off-peak wind energy to power our jets, trucks, trains, and cars.

  80. Maju

    I would like to ask the members of the pro-nuclear camp, to present themselves as volunteers to help contain the radiation at Fukushima or bring food and aid to the victims, as it seems they have been abandoned even by their own government.

    I think that the members of the “nuclear yes” camp have that moral obligation. I point my finger at them and put the blame of all the casualties and other victimization on them.

    The least they can do is to put their lives at the service of minimizing the consequences of what they so wholeheartedly seem to believe in.

  81. Martin

    If one doesn’t want nuclear power, the question is, what is the alternative?
    In terms of dead/kwh most likely wind and solar power are the best. But it is non-trivial to deploy them in the time scale needed. Even the wind and solar industry advocates in Germany, which is very well suited for the transition to these sources, because of a shrinking population, capital surplus, good connections to a geographically diverse neighborhood to bring power to those countries where currently no wind blows, and a population, that is much more willing to pay a high price for such a transition than the US seems to be (think about gas prices), believe that they need ~ until 2050 to deliver 100% renewable energy. So at least for the next 40 years, we need some other source of power.
    In the US so far not even the needed political consensus for such a goal is build so far. One has to be willing to put up real money, and one has to be faster than consuption demand raises. Therefore the US is not on a good way despite the geography and the low density of population are generally good conditions for such a transition.
    The greens advocate mainly natural gas as transition technology.

    Natural gas has at least two problems, it is far from abundant and it still causes climate change.
    How bad is climate change? Mostly climatists try to talk the effects of climate change down to say, we know it is at least this bad, and likely that bad. But what is a safe limit? Can there be positive feedback loops, that bring climate totally out of control? There are some scientist, who believe this is possible. E.g. climate change can free gases from the permafrost, that are toxic in high concentration. Another risk are the Boreal woods. They produce 2/3 of the oxygen in the world, and could be destroyed by fast climate change. So far they are few insects and illnesses, that can affect them in the cold north, but fast climate change could allow vermin to affect those woods, and the trees will die off much faster than other types of wood replace the original wood. Another thing to keep in mind is the fact, that Hurricanes depend critically on the water temperature. If the oceans get several degrees warmer, Hurricanes will become dramtically more often, possibly making life near a coast not in the far north almost impossible. Finally massive draughts could be the cause for WW III, and thereby indirectly the cause for the end of humanity.
    To be sure something like that can’t happen, we should look into historical data of CO2 in the atmosphere. Something like such feedback loops shouldn’t happen then. We are already far above such a save rate, and the rate at which the CO2 levels would fall in case we stop emitting it is mostly overestimated. As a consequence the IPCC doesn’t give a number any more in their most recent report. Therefore I would advocate to try to limit the emissions to the absolute necessary.

    The other issue with natural gas is the availability. In the CIA world fact book know reserves are estimated about 30 times current world gas consumption. If we ramp up the consumption considerably, the time is even shorter. More over, because of the high flexibility of natural gas, it eases the problems with smoothing wind and solar energy production supply. If one uses it for baseload, we need to get the real full 100% transition faster. After all, keep in mind, that poor countries have to make the transition to solar and wind as well. As natural gas is useful in this transition, the rich nations should not use up all of the bridge technology resources.

    Finally what about nuclear itself. I think there are ways to deal with the nuclear waste later. We can shoot it on the moon or into the sun, once we have mastered the current very critical situation of earth. If we destroy the biodiversity of planet earth, it takes a million years to regenerate. We should pollute now in ways, that are reversable in a couple of hundred or thousand years, and try to avoid pollution, that is irreversable on that time scale.
    Even accidents like the Tschernobyl one pose very limited danger for nature. If we increase the cancer risk for humans in a large area like the Tokyo metropolitan are by a few percent, this means millions of dead people, and is considered unacceptable, but ecological systems are not dramatically altered by such radiation levels. Therefore in Tschernobyl, only areas quite close to the plant suffered seriously, and in a few hundred years even those areas will be reclaimed by nature.
    But Tschernobyl is a bad example for the damage to humans anyhow. Had the very Tschernobyl catastrophy happened in the west, there would have been much fewer people with cancer. Most people affected by radiation, according to the WHO study mentioned above, were affected by eating contaminated food. With a clearer information policy and controls many health problems could have been avoided. In addition the liquidators were not as good as possible protected from the radiation.
    No terrorist flying an airplane into a power plant will be able to force people to eat contaminated food or work badly protected on the cleanup.

    Therefore in total I think, nuclear is for the next decades a possibility, that should be build in parallel to the renewable energy build up. While money is fungible, people are it only so far, so there is no reason to believe, that going forward with nuclear has to be a large drain on the wind and solar front. To do so, in any case there has to be a willingness to put up some serious money.

    1. Maju

      It is largely a matter of will and planning. Somewhere above a commenter linked to a plan to make such transition in the USA in 20 years, so in Germany can be done too in such a short time if there is the political will.

      Of course, it will need some time but the important thing is to begin the transition and, I’d even dare say, to understand, to accept, that we need to make such transition.

      Then it’s a matter to put the means and doing it.

      If we are lucky no other major nuclear accidents will happen in the meantime – but as long as there is nuclear fuel around, there is a risk of accident (not to mention baseline pollution), so the sooner we act, the better.

      While I do fear that nuclear energy will not be completely abandoned in the short term because of military reasons primarily, I do think that those countries taking the lead into renewables should have an economic advantage in the near future. Because the other resources, not just nuclear, are all too costly in terms of environmental, health (and hence indirect economic) damage.

  82. Skippy

    Less production and consumption would alleviate most of our problems, why that is not on the table first and foremost, is the first clue.

    Skippy…so much wailing and gnashing of teeth over how to have more…shezzz. To the cliffs!!!!

  83. PD

    Hmm, less production would require a population decrease worldwide. Seems to me many people would have an issue with that, the MidEast is rising over an increase in food prices. Or do the Greens not believe their own propaganda, that the average human is clinging to life by a rather thin thread? Decrease in production = starvation, and that’s the way of it. It will certainly reduce population, but the war that will follow as people refuse to starve quietly will most certainly reduce population still further. Given that most of these responders come from the country that uses more fossil fuel than any other, and also uses the most nuclear power, AND has the worlds biggest military, we have to wonder if they have a legitimate leg to stand on?

    Solar is nice, but the only way to get 24/7 reliable baseline solar is to put satellites in orbit and beam the power down. OMG, we might cook a goose (ridiculous if you know ANYTHING about beam energies or densities or natural microwave sources or even feathers and birds, but whatever) or something. So there are dams, which is solar at a remove. Apart from the flooding of farmland near the river you dammed, you’ve got a small issue with earthquakes if you are looking for “perfectly safe”. OFC, you won’t have the whole TV drama thing going on, if there’d been a major dam above that city they’d have flushed out to sea like rats down a commode ten minutes after the quake hit, and that would have been the end of it.

    As for fossil fuel, there’s the radiation risk from coal and oil (yes, there is):

    http://www.scientificamerican.com/article.cfm?id=coal-ash-is-more-radioactive-than-nuclear-waste

    http://www.ornl.gov/info/ornlreview/rev26-34/text/colmain.html

    Funny that, eh? Perfect doesn’t exist. Get over it.

    1. PD

      Ah, my fault for not saying this, but WHY ON EARTH does anyone listen to anthropologists or sociologists or preachers about matters involving physics and engineering and geology? They know NOTHING about the subject under discussion, and can’t learn enough to make an intelligent contribution without essentially taking degrees in these subjects. All they can do is parrot what someone else has said, filtered through their prejudices. How does that contribute anything but noise and confusion? WHY did anyone listen to the guy who wrote the California Nuclear Shutdown Initiative after he publically stated “the only Physics I ever took was ExLax”? If you know little or nothing about a subject, just how does that qualify you as an expert?

      1. skippy

        Sorry to knock you off the cliff, of no growth, but, seriously how much garbage do we make and need, how much is thrown away as surplus and most not long after purchase ie fashion, intentional design defects, reduction in material quality (source constrictions / depletion), etc, ad infinity…cough the ways a sales person can imagine.

        Skippy…yes, yes, I know, it is the anathema to modern monetary functioning, a design flaw or feature? BTW FYI I was asked first to work in a missile MOS whence joining the military but, was determined for the wrong reasons to fall from the sky…myself.

        PS. thanks for illustrating perfectly my earlier observation “so much wailing and gnashing of teeth over how to have more…shezzz” with your response.

    2. Maju

      Less production does not imply less population, it implies less concentration of wealth in few hands and maybe less consumption from industrial actors as well.

      Most people’s consumption is pretty low, even those living in wealthy countries have their ecological imprint massively expanded by public expending in the military, roads, nuclear reactors…

      We can cut production by careful planning: it’s not as simple as “one person = a zillion goods”, in fact most people do not consume much and when they do it is because they have little choice (for example: compulsory plastic packing, lack of choices in electricity generation, house design, etc.)

      “… the only way to get 24/7 reliable baseline solar is to put satellites in orbit and beam the power down”.

      Nonsense. You do not generate solar by night, you accumulate it by day (there are several means) and you diversify your sources (wind, tidal, hydro…)

      Sometimes it seems people can’t think beyond the box… the box they have themselves put on their own heads.

      1. Toby

        Maju, thank you for your wonderful and patient work on this thread. You are a credit to us all.

        As to night-time solar panels, there is this:

        http://www.engadget.com/2008/02/02/infrared-solar-panels-even-work-at-night-but-cant-output-energ/

        Whether or not they crack the problem of accessing the generated energy, I can’t say. But if they do, that would be quite something!

        And of course storing energy generated in excess of requirements has all sorts of clean and ingenious solutions as you know, but I just though I’d mention that again. ;-)

    3. Toby

      I’m a fan of technology, which I define as human ingenuity made real. But scientific positivism goes too far. The “Gee wizz, the Future!” brigade have been promising too much for too long. Whatever we do with our gifts, as a species on this beautiful planet, we must first and foremost ask if it is sustainable. Of course we cannot ‘know’ all variables for all time–unlike the perfectly informed homo economicus!–but humility and caution are wiser than “we’ll fix that in version 1.x!”

      Where is it written that we must grow exponentially? In the money system. Debt-based interest bearing money is a pyramid scheme and as such demands perpetual growth. Constantly burning more and more fuel to power such insanity is the problem, not the solution.

      Currently, and for a long time, we have been culturally addicted to perpetual growth. ‘Growth Forever’ is a physical impossibility. To defend nuclear power and all its risks because it is the ‘only’ way we can sustain our lifestyles, is to overlook that we should not be sustaining our lifestyles. We cannot sustain our lifestyles, no matter how much we want to. However, I feel sure a very healthy and high-tech life is possible for all, if we get off the growth bandwagon and focus on energy efficiency and renewables.

      It is in fact precisely because we cannot know all variables for all time that we must first reorient society to live within what is ‘renewably’ and sustainably attainable. To argue that because nothing can be perfect we should therefore carry on blindly as we are now is dangerously fallacious. Empires rise and fall, nothing lasts forever, a fact we should all get used to. Even if we were able to produce the energy necessary for everyone on the planet to live like an ‘average’ American, the fish couldn’t take it, the soil couldn’t take it, neither can our fresh water supplies, and much else besides.

      The energy problem is not a bunch of numbers to be analyzed in isolation from the rest of the planet, or the rest of society. Nothing makes sense in isolation. If you’ll excuse the cliche, we have to see the bigger picture.

  84. August von Stradonitz

    “Kekulé dreams the Great Serpent holding its own tail in its mouth, the dreaming Serpent which surrounds the World. But the meanness, the cynicism with which this dream is to be used. The Serpent that announces, “The World is a closed thing, cyclical, resonant, eternally-returning,” is to be delivered into a system whose only aim is to *violate* the Cycle. Taking and not giving back, demanding that “productivity” and “earnings” keep on increasing with time, the System removing from the rest of the World these vast quantities of energy to keep its own tiny desperate fraction showing a profit: and not only most of humanity – most of the World, animal, vegetable, and mineral, is laid waste in the process. The System may or may not understand that it’s only buying time. And that time is an artificial resource to being with, of no value to anyone or anything but the System, which must sooner or later crash to its death, when its addiction to energy has become more than the rest of the World can supply, dragging with it innocent souls all along the chain of life.” – Thomas Pynchon “Gravity’s Rainbow”

      1. Toby

        Each individual life experience growth, decline and death. But it is wrong, I believe, to say all life is just buying time. Life learns, gathers wisdom, develops. If it weren’t so, there would be no intelligence in the universe, nor would we be having this discussion.

  85. rob

    Nuclear power is worth it for the investors, but not for the rest of us, especially if they recieve government support and cover-ups when reactors leak…It’s like asking if it is a good idea to break into my house and steal my TV. Sure it is! Great idea! Don’t you just love the ‘free’ market? all the opportunities to get rich! – spot the sucker: YOU, ME and all the billions of people in the world that can’t rub 2 cents together and don’t have any power to protect themselves. The latter could perhaps hypothetically be able to live off the risk premium that many industries owe them since their air, water and genetic code are constantly being damaged, however subtly in some places, however directly in other places, as much as ours: and none of this *belongs* to the investors any less than my TV belongs to a competent thief, it is simply not their property to take or to put at risk in any way, nor does the environment belong to the government without reference to the people. So asking whether it is worth it or not completely misses the point: it depends which side of the sleight of hand you are on. Just like stealing my TV or driving without insurance and then having an expensive accident.

    It’s not as if they pay each of us a risk premium in terms of ‘free’ electricity produced or anything, is it? We still have to pay market rates for their product. Whilst the price will reduce due to increased supply, this price doesn’t take into account the full extent of risk and damage undertaken with respect to third parties or the environment, let alone the majority of the world that is put at risk but uses minimal, if any electricity. They sure don’t benefit. This miss-pricing is the problem, and ONLY this is the problem. Sufficient damages, risk premiums, environmental maintenance fees and rents are NOT being paid, and I for one want my payment. And as with any non-paying debtor, if they don’t send me my money for my risk I should close them down, I would be a total FOOL, a total sucker, to want them to continue taking from me for some imagined greater good like ‘we all need energy’, and everybody on the planet should be paid too, not just me…then we’ll see the right pricing in the market (I’d settle for 150 dollars a year per existing reactor for a run down period of 10 years if they care to call me, future reactors I would want to negotiate a personal equity stake and specific health and safety terms of my own devising before I sell my share of semi-breathable air).

    Which means: close them all down – because they can never pay the right price!

  86. Maju

    I’d like to mention one of my best sources of news on Fukushima, which is Energy News blog. This blog is a spinoff of Florida Oil Spill Law, which has been covering the Deepwater Horizon catastrophe since the beginning. For several days it was FOSL which reported on Fukushima but eventually they seem to have decided another blog was needed. Both are making a great news coverage job for whoever wants to stay tuned, always from the “eco” viewpoint, indeed.

    For example, here we have a (Google-translated) German expert opinion in which it is explained that Fukushima is in practical terms 20-30 times worse than Chernobyl.

    No kidding.

    1. toxymoron

      Hi Maju,

      I think this German expert is kidding, is unknowledgeable, or somebody else in the US is completely off track.
      He is about Cesium 137, which is a danger, but is only present in very small quantities (and barely above what you get everyday aynyhow).
      He is also right about the huge amounts of radioactivity present in Fukushima, but somebody omitted to state that that activity disappears after days, not tens of years. It is precisely for that reason that people are asked to remain indoors: not to contaminated with short-term elements, coming as soot.
      You may want to lend an eye or an ear to this: http://online.itp.ucsb.edu/online/plecture/bmonreal11/
      (not to say I am pronuclear, or nothing serious happened in Fukushima, but we should keep our heads cool).

      1. Maju

        “He is also right about the huge amounts of radioactivity present in Fukushima, but somebody omitted to state that that activity disappears after days, not tens of years”.

        Radioactivity does not disappear but very slowly (the rule of thumb of 10x the half-life – at least, it will depend on original concentrations actually). It can be blown away if it’s in the air but it goes elsewhere, it can be hidden in the soil and other stuff, including living beings like crops, livestock, fish or people. That’s why we impose restrictions to food exports to affected areas, that’s why we fear contact with people who have been contaminated.

        As Lengfelder puts it: no more sushi… at least no without preliminary radiation controls, which are expensive.

        I do not understand well what is the radioactive inventory and why this is 20-30 times Chernobyl. But some were arguing about listening to the experts, right? Well, Lengfelder is an expert in radiation, so he should know what he says and he has no reason to be kidding as you claim (still 11 days to April’s Fools Day and this is no joking matter anyhow).

        1. Hal Horvath

          But that begs the question of whether you think the official, relatively low radiation readings are a large-scale conspiracy: requiring the perfect cooperation of dozens, and eventually, hundreds of people, from more than one nation and more than 2 or 3 organizations.

          1. Maju

            We do not have clear information of which are the levels of radiation at the Fukushima area. Do you? If so please provide a link.

            Mostly authorities are more concerned about minimizing the scale of the problem and therefore they tend to hide the data and restrict independent researchers (if they exist at all) from reaching out to where the damage is. We saw that in Chernobyl, we have seen in the Gulf of Mexico and we see at Fukushima. And these are just a few, particularly severe, examples of environmental, health and economical catastrophes caused by “trigger happy” developism of certain kinds, disdaining the risks and redirecting the costs a priori and a posteriori to the public.

          2. Skippy

            Sorry Hal, you don’t need a conspiracy, just an environment that establishes like minded thinking, independent / loosely affiliated actors seeking the same out come[s.

    2. Hal Horvath

      Here:

      http://www.stubbytour.com/nuc/index_en.asp

      Fatal dose (over time) is typically well over 2 Sv. a mSv is 1/1000th of an Sv. For example 1.91mSv/y means that if you live there for an entire year (and also if the source of the radiation level remained constant!) then you’d accumulate 2/1000 Sv or 0.002 Sv.

      Of course, the radiation levels will decay even from these very low levels.

  87. Hal Horvath

    Current Radiation readings (official, from 2 nations)

    http://www.stubbytour.com/nuc/index_en.asp

    Fatal dose (over time) is typically well over 2 Sv. a mSv is 1/1000th of an Sv. For example 1.91mSv/y means that if you live there for an entire year (and also if the source of the radiation level remained constant!) then you’d accumulate 2/1000 Sv or 0.002 Sv.

    Of course, the radiation levels will decay even from these very low levels.

    1. skippy

      Hal with all do respect, with out a 3D presentation showing all relevant data ie: time, space (topographical, before and after building *blueprint quality*), local meteorological data, data collection reference points, etc. Static values offered to the MSM or handed out by government / quasi government agency’s local or international are useless.

      Hell in situations like this time to discovery_is every other moment_and even when you get discovery, is it just one sq meter or the hole site, after which the hole enchilada morphs into a burrito.

      Skippy…lets be honest gentlemen, desires and bias aside…their_winging it_and you know that.

      1. Hal Horvath

        No, not useless at all. Simply good approximations of a point in time, which in combination with *many* such approximations actually do paint a reasonably good picture that lets us guess at the reality (that complex, shifting reality of various levels in time and space).

        1. skippy

          Six reactors with out power for days, shut down or not require constant cooling to advert partial / full melt down.

          Numerous explosions, some of which raised very large and heavy bits of re-bar and concrete over a 1,000 ft (work out the force applied), how many cubic meters of gas at what concentrations[?] before ignition, what was ignition source, effects of internal structural blast wave and ensuing status of systems.

          Long term heat and its effect on both rods, building integrity and its internal service infrastructure.

          Actual action taken by US military vs. MSM / governmental posture.

          The use of sea water and its effects, damage to stainless steel cooling components, electrical systems, salt weighs a lot eh, do the math on evaporation and residue, accelerated corrosion[!] days to weeks depending on other a fore mentioned deterius events to functional compromise.

          To whit, there are more unknowns than knows and as someone that has extensive experience in both explosives and construction, of most types, hence I am completely unsatisfied with a few readings / stats from those with the most to lose.

          Skippy…Very much like the GFC…eh…trust bankinfiend much?

          PS. distrust in power is my default position… me wonders why?

  88. Ellen Anderson

    Is this the longest thread ever on NK? I read about half of it yesterday and was so surprised and horrified that I stopped. I decided that I would cross this site off of my list. But these dialogues are so interesting that I just had to have another peek. Bless those of you who remain patient. It is just beyond me that anyone thinks that the current resource-in garbage-out model (a) can endure or (b) should endure.

    Once you give up the notion that the industrial way of life can or should go on then the answer as to whether nuclear energy is “worth it” is obvious.

    1. Hal Horvath

      To get a non-violent change to a less destructive economy, we need to “internalize the externalities” — to have businesses (coal burning utilities, etc.) actually pay the *full total costs*, environmental and health, etc. that they are imposing on the common, which is you and me and our world.

      1. Maju

        Absolutely, Hal.

        But that won’t happen because it’s the business who run the show (that’s what Marx and Lenin meant by dictatorship of the bourgeoisie, even if it has democratic forms). They orchestrate everything to get away and if you protest, they look at you with a look that implies that you are naive and they are “in the know”, i.e. they know it’s just plunder and ripoff with a varnish of respectability.

        Nonviolent or violent, the change will only come when a majority or at least a clear plurality of the people know all this, stop being naive while not accepting either the abuse the system implies.

    2. Toby

      I find it fascinating that a discussion of energy has hit such a nerve; I always expect discussions of the money system to do so!

      And, at risk of being overly repetitive, I’m pointing out here yet again that Perpetual Growth is a systemic necessity of the money system, which is a pyramid (or ponzi) scheme. If the economy is not growing, it is collapsing. ‘Growth’ has become ‘Good’ not because it is always and only good, but because the Extremely Important money system requires it to survive. This is an untenable situation.

      Any discussion about the global economy’s energy requirements that do not include discussion of the money system’s inherent and fatal flaw, or ignore resources such as soil and water and planetary carrying capacity generally, are woefully inadequate. Any assessment of nuclear energy which focuses solely on limits to radiation, lowering the number of peopled killed or injured in explosions, near-perfect safety at some future date (decades away as usual) is a waste of time at best, deadly to many at worst.

      Because we can be sure that perpetual growth is impossible, that pursuing it blindly to protect the money system is self-destructive, we can also be sure that only renewable and clean energies are sensible. If we have the goal of long term survival and continuing civilization, renewable energies are obviously the only choice we have. We are of course free to choose self-destruction, but I don’t want that outcome for us. For all our violence and stupidity, I remain confident that humanity has a fine future, should it get its act together in time.

      1. Maju

        You are absolutely right, Toby. How come I could not think of such obvious comparison, identity, before?

        Are the Capitalist economy (or “perpetual growth” paradigm) and a Ponzi scheme the same thing? Oddly enough the answer is yes and for the reasons you say: the scheme has to keep growing or face collapse and consists in robbing people of their present by offering them “a future”, which may or not come depending on whether the Ponzi scheme is still expanding or has gone into default mode (crisis).

        The differences are just of size: a Ponzi scheme is generally thought of limited in size, while the Capitalist economy’s only limit seems to the Earth. But the comparison is totally illuminating, revealing of what is going on.

        You deserve the Nobel Prize of Economy for that unveiling act, Toby, really.

        1. Toby

          I’m happy to pass the idea on, but I’m no original thinker, so your high praise must go elsewhere.

          My leading lights these days are Charles Eisenstein (www.ascentofhumanity.com), Bernard Lietaer, Franz Hoermann (his writings are mostly in German though), Jacque Fresco (for hard core radicals!), Richard Douthwaite (his “The Ecology of Money” is available online), Bernd Senf (only available in German, except at my blog) and a handful of others.

          Before we can do anything sustainably, we must change the global money system at its root. Before we do that, we have to want to do so, in our millions, and know well why we want to. That is the challenge before us, and it is mighty. So study money and its alternatives, then spread the word!

          Demote money, promote wealth.

          1. Xavier Onassis

            Toby, Maju, Down South, Yves, other like and capable minds:

            Do you know Damon Vrable, csper dot org, Renaissance 2.0 series of short videos?

            Toby, I confidently assert there exists no better means for showing the underinformed/misinformed masses just how excruciatingly correct you are, and you should be passing this around as fast as you can:

            http://www.csper.org/renaissance-20.html

            Each video is about 10 minutes long. Try lessons#6 first, then tell me what u think – PLEASE??

          2. Xavier Onassis

            Maju, no worries, eye nose yr busy, view #6 first 2 parts after you’ve finally a chance to take a breath. I promise you’ll be overjoyed to make the discoveries. I mean it – no better presentation exists – but it’ll be there when yr ready.

            Oh, and: platonic, yes. Housewife from midwest usa here. It’s your egalitarian rational brain I’m adoring.

            (Egalitarians are the only sane people on this planet.)
            Here – you might enjoy this I wrote:

            Survival of planet Earth, solar system EB213, galaxy MaDas/5

            situation critical; very close to destruction; request immediate intervention; inequity factor is 1,000,000,000 [one billion] and rising; most powerful species is victim of bigpicture blindness; cannot see the overall situation; this species has technology and weapons capable of destroying entire planet 60 times over; please send emergency response team asap; code 10; need equipment for disabling uranium-based bombs; extreme inequity increasing exponentially, driving violence to infinity; planet could burn at any moment; species has very confused and imperfect ideas of value of justice, and of connection of injustice and violence; recommend you send 1000 indestructible teachers; species has driven itself mad; most are vulnerable to psychosis and panic mentality; obedience to custom and convention is solidified, submission to accepted ideas is extreme; sense of alarm at situation is extremely slight; recommend extreme caution; planet is worth saving, has some interesting lifeforms

      2. Hal Horvath

        re: “…or ignore resources such as soil and water and planetary carrying capacity generally…”

        See my comment just above. Is it too abstract? The costs should include *all* the costs, that’s what I’m saying. So, for example, degrading a water source is very expensive. Degrading a lot of topsoil is very expensive. Those can be large numbers, for instance tens of billions of $ in some instances, and there are a lot of instances. I suppose what I wrote was too abstract?

        1. Toby

          Is it too abstract? The costs should include *all* the costs, that’s what I’m saying.

          It is too abstract until we have had a thorough discussion of how to measure costs, which we currently do with money. I am convinced that money has so far failed to be able to measure the ‘value’ of resources and the long term costs of perpetual growth. Besides, while we are forced by the monetary system to grow perpetually our measurement of energy ‘needs’ going forward is polluted to the point of irrelevance.

          Moreover, in the current money system money has top priority, steers all decisions, and this is very corrupting; I can hardly trust any data from anywhere anymore, since everything seems to be a vested interest of some kind. We really need to be able to trust one another.

          In the end it is obvious that we are constrained far more by the planet’s carrying capacity than we are by our ability to provide ourselves with electricity, something we are very foolish to ignore. We should seek to establish, therefore, a resource-based economy, which is the only economics I know of which inherently prioritizes resources over a medium of exchange/unit of account.

          I am not against technology, comfort, showers, air conditioning, fridges, and so on. I am for any system which allows maximum possible good sense and wisdom to run the show. Debt-based interest-bearing money is not it. By my reckoning a resource-based economy would be.

  89. Xavier Onassis

    This is the magnitude of murder (our ignorance about) the money system commits:

    One million children will die this year from lack of zinc.

    Lack of zinc.

    A million children.

    Another million will go blind from lack of 4cents worth of vitamin A.

    Another million girls will lie down in the sex trade this year, trying to get enough pay to stay alive.

    The “third world” experiences a September eleventh 2001 every 4 hours right ’round the clock, in terms of the number of casualties.

    Pay in this world ranges from $1,000 per second to $1,000 per lifetime.

    50 million WORKING people will die this year from lack of enough pay to keep them alive – and still no economist gets it.

    Overpayunderpay is killing us all and this planet.

    Bottom line, the human species will either murder the idea to allow unlimited personal fortunes or we will succumb to the results of having the next and the next and the next wealthpower giants, ad infinitum until the violence attached to the extreme and genosadistic inequality sets off the global bombs that reside under every chair, bed, and movieseat on this planet.

    (anyone can write to me at payjustice at fastmail dot fm)

    1. Hal Horvath

      Good points.

      How can people be so oblivious to all the suffering? — They get oblivious I think by pigeonholing each other, and getting bogged down into us vs them psychology. Too many people are too busy fighting their imagined enemies. That’s the center of the human tragedy.

      1. Xavier Onassis

        Hal, The entire human species is playing a lose-lose economic game that came down to us cobbled-together by default as much as by design. It’s called All grab for all, grab off and grab back. The rule is: Everybody go after getting all you can get out of what is a finite pool of wealth.

        When the species finally agrees to play Everybody go after getting out what you put into that pool by virtue of your own work, no more and no less, only then do we discover we’ve been creating enough wealth all along to give everyone a happy life with all needs met and plenty of desires, only then do we get back our chance to have a future…and a golden age of peace and plenty and prosperity and safety for this weary, inert, slumbering, blind, unrealistic, needlessly suffering species.

        Wealth is created only by the sacrifice of a person’s time and energies. (If anyone is unsure of that fact, now is an excellent time for you to take a dollar bill out of your wallet – and tell it to fix you a sandwich.)

        The work in this world is spread billions of times more equally than the wealth it produces is spread. Reward became divorced from sacrifice when wealth became portable and storable and we began division of labor and started trading the workproducts of our sacrifice of time to working.

        But No one has seen that the inherent flaw in capitalism is in the very nature of transaction itself – and that it can and must be corrected for to keep inequality from going to infinity. It’s too much to explain more here, but I’ve written about the fundamental misconceptions our economics is erected upon elsewhere…and the way out of all this madness.

        We have hung a depraved encumbrance about our own necks by swallowing irrational excuses for overpay-underpay…which is obviously overpower-underpower. Since money is power, democracy is by definition made impossible by extremes of wealth far smaller than ours actually are (yet we speak of democracy as if it exists).

        “The rich get richer and the poor get poorer” means precisely this:

        The rich are getting more and more for a unit of work while the poor are getting less and less for the same unit of work (= sacrifice of time).

        Why would anyone court extinction by cutting corners and falsifying information in the nuclear arena of all places?

        Because the least scrupulous amongst us will ALWAYS be going the hardest after the carrot of unlimited personal fortunes we hilarious humans dangle in front of everyone without one thought about how wealth is produced and what amount constitutes fairpay justice capitalism – that’s why.

        And yes – our genosadism CAN be taken too far, Humanity.

        The children dying from lack of zinc?

        Society stripped them all of their very first crucial birthright – a place to put their feet – LAND – and never compensated them for that legal theft with even enough money to get a bit of zinc, let alone any compensation for their share of land that was all owned before they arrived. Google up Mark Twain Archimedes. Twain said it better than I can, and in a short story you never knew existed til now i bet.

        1. Hal Horvath

          A lot of ideas. Let me offer a couple of responses you’ll see:

          The big one: in capitalism, in theory, the new inventions create opportunities for new wealth (instead of only old, immortal wealth), second whatever monetary wealth exists has to be spent or invested typically, and either typically (sans some kinds of gambling) results in economic activity = jobs (as there are more jobs, more people can make a good living). Finally, there have to be starters somewhere — people who walk out of the current place and start something new — and they are the ones that create the basis for most of the wealth in an absolute sense.

          There are plenty of places to discuss/argue, but you have to be ready for these points.

          About the nuclear falsification: this is another instance of the “tragedy of the commons”, in which we allow businesses (sometimes we have instances of actual corrupt (PURCHASED)laws that limit liability!) pollute, etc., and *not pay the costs* this imposes on everyone else. There are so many instances of this, and it is a form of criminal behaviour, but we’ve often made it legal.

          Finally, kudos on the insight we all have a *right to land*. This is a very fundamental insight, and has been well developed by Henry George. It has widespread implications, and economics — and is actually worth looking over and learning. It is one of the best insights I’ve come across, even after 30 years of reading economics, philosophy, and various economic philosophies. We have only poorly implemented some of George’s valuable contribution, in the form of a flawed but better than nothing property tax.

          1. Hal Horvath

            Back to the original topic for a moment:

            http://www.nytimes.com/2011/03/21/business/energy-environment/21green.html?ref=energy-environment

            “Each year globally, thousands of coal miners die on the job. They die underground, away from the cameras. In China, officials say, 2,442 people were killed in 1,401 coal mining incidents across the country last year. In the United States, 18 coal miners died in 2009.

            “We are more accepting of a smaller number on a regular basis than we are for the potential for a huge number (from a well publicized nuclear accident),” said Davitt McAteer, the vice president of Wheeling Jesuit University in West Virginia, a coal mining state, who served during the 1990s as the U.S. assistant secretary for mine safety and health.

            “Coal plants pose an even larger threat than mining, however: pollution. Coal plants emit soot, sulfur dioxide, nitrogen oxides and other pollutants. These can cause smog and sicknesses like lung cancer. Last week, the U.S. Environmental Protection Agency announced a proposed rule aimed at the coal industry that would limit emissions of mercury and other toxins from power plants. The rule would prevent “as many as 17,000 premature deaths and 11,000 heart attacks a year,” the agency said. ”

            30,000-35,000 deaths a year from the other big energy source.

            Why don’t we count these lives as publicly as we count as much as the 6,000 from Chernobyl? I think it’s all about information. I hope it isn’t about politics or values.

          2. Toby

            in capitalism, in theory

            After that, what you say about capitalism really doesn’t matter, for it is what happens in practice that counts.

            Also, jobs for wages is an outdated concept, and the idea that there is a job market is just laughable. We need nurses and technicians and scientists, but how well do they get paid? How badly do we need all those bankers? A professor of economics and one of Germany’s most successful businessmen, Goetz Werner, puts it like this (my translation):

            The economy does not have the role of creating jobs. On the contrary. The role of the economy is to free people from jobs. And that is precisely what we have managed to do wonderfully well over the last 50 years. [snip]

            No businessman wakes up in the morning wondering how to stuff more employees into his business. The very idea is absurd. The question should be asked the other way around: How can I, with as little effort and as few resources as possible do as much for the customer as I possibly can? How can I better organize my business? And better organized always means: minimize work. This is a foundational business principle. [snip]

            Indeed a fisherman told me that Icelanders today, thanks to factory-ships, can catch four times as much fish with four times fewer fishermen than 30 years ago. Do you see? 75 percent of people are simply no longer needed. There are such examples everywhere. Our ability to produce things exceeds our need to consume them. This is a simple fact, and no job market reform can change that. [snip]

            We really have to ask ourselves: What is the real role of the economy? There are two. The first: It must supply people with goods and services. Never in history has the economy fulfilled this role as well as today. In fact we see a massive oversupply. Even though most factories are by far under-utilized, we can produce everything we can desire. [snip]

            We’re moving into a society in which jobs are disappearing. The simple question is what all these people are to do with all their time, which is a cultural question. The problem we have does not lie in the jobs market, but in the culture. Sadly, this subject is barely visible in society today. And yet it is precisely here that we have work to do.

            The whole thing is upside down. Until we have a sensible attitude to money (have you read either Soddy or Gesell, Lietaer or Eisenstein, or looked into post-scarcity or resource-based economics?) we rearranging deck chairs and digging our own graves. Quite the multi-tasking feat.

          3. Hal Horvath

            Toby, yes I agree. I’ve considered for some time now the implications of automation and mass production together. The factor which has prevented the loss of all jobs for decades isn’t weak either though — innovation of new products/services. It has been that new products could not be produced as efficiently as old ones. That could change/is changing to some degree. We inevitably will have a very different economy in the future, sooner or later, by necessity. I’ve thought on that for years now.

  90. Maju

    A very interesting translation of (part of) an interview with Hiroshi Takashi is at CounterPunch today.

    He claims (reality has sadly proven him right today as well) that the cooling system has been damaged by the salt of the evaporated water and cannot be reinstated as was hoped. He also says that pouring water is useless: “like pouring water on lava”, that only a Chernobyl solution (sarcophagus) will work.

    More interesting data:

    As the area becomes radioactive, there is a risk of domino effect to other reactors and even other power plants such as nearby Fukushima II. This is because the radiation impedes workers to keep control of the reactors and this is highly delicate stuff that just can’t lay idle: it needs systematic surveillance and maintenance.

    [Me: when people claim “nuclear is safe” they forget even this: that, unlike coal or a windmill, this stuff just can’t lay on its own without causing big problems].

    Comparison with X-rays and CT scans are pointless because you do not breath or eat radioactive material in such processes. Ingested radioactive ions make you radioactive inside, receiving constant radiation from those sources.

    [Me: He does not say but, obviously, what is on the outside (dust, etc.) also keeps emitting radiation every single second, so if you are nearby your get radiated (only thick lead can protect you but it’s impractical)].

    1. Hal Horvath

      The label or pigeonhole for me isn’t the “pro” or “anti” nuclear, but instead, it’s the “stickler for accuracy”.

      But hey, I think I transcend that label.

      I’m actually after people listening to each other and having a productive discussion.

      One thing they need is accuracy.

      Here’s a problem area:

      “Comparison with X-rays and CT scans are pointless because you do not breath or eat radioactive material in such processes. Ingested radioactive ions make you radioactive inside, receiving constant radiation from those sources.”

      Ok:

      Ingesting is very different from outside radiating — correct.

      “constant radiation” — very incorrect. If that was correct, we all would not exist, due to simple physics — the earth has had a large amount of radioactive elements. If they never decayed (half life = infinity), but just magically emitted *constant* radiation the earth would be incompatible with life as we know it. The radiation actually comes from the decay itself — part and parcel. As the amount of emitters decay in a location, so does the intensity of the radiation.

      This isn’t a minor point.

      When seriously wrong ideas are woven into good ideas, the result is discrediting to all the ideas.

  91. Xavier Onassis

    Dear Ann Coulter: According to you, radiation is actually good for us. So then… tell me again… why are we spending a truly stratospheric fortune on the military, intelligence, and security industries trying to prevent a dirty bomb going off in Dodge City, eh?

  92. Hal Horvath

    Back to the original topic for a moment:

    http://www.nytimes.com/2011/03/21/business/energy-environment/21green.html?ref=energy-environment

    “Each year globally, thousands of coal miners die on the job. They die underground, away from the cameras. In China, officials say, 2,442 people were killed in 1,401 coal mining incidents across the country last year. In the United States, 18 coal miners died in 2009.

    “We are more accepting of a smaller number on a regular basis than we are for the potential for a huge number (from a well publicized nuclear accident),” said Davitt McAteer, the vice president of Wheeling Jesuit University in West Virginia, a coal mining state, who served during the 1990s as the U.S. assistant secretary for mine safety and health.

    “Coal plants pose an even larger threat than mining, however: pollution. Coal plants emit soot, sulfur dioxide, nitrogen oxides and other pollutants. These can cause smog and sicknesses like lung cancer. Last week, the U.S. Environmental Protection Agency announced a proposed rule aimed at the coal industry that would limit emissions of mercury and other toxins from power plants. The rule would prevent “as many as 17,000 premature deaths and 11,000 heart attacks a year,” the agency said. ”

    30,000-35,000 deaths a year from the other big energy source.

    Why don’t we count these lives as publicly as we count as much as the 6,000 from Chernobyl? I think it’s all about information. I hope it isn’t about politics or values.

    1. Xavier Onassis

      Chernobyl took 6,000?

      No. Chernobyl took almost a million lives between 1986 when it happened and 2004.

      Just had to correct that error.

  93. Electrical Engineer

    Nuclear power is not worth the risk, and here’s why:
    All nuclear power plants are permitted regular venting of CFC’s, and radioactive materials into the environment, counting on dilution by natural water and air to hide the radioactive isotopes being added to background radiation, but they are slowly being added nonetheless, and are not the same as natural background radiation. Nuclear is not worth the price no matter how cheap its short-term electricity is claimed to be. The long-term products are emissions, waste, weapons and accidents that WILL happen, because the risk is not zero.

    Nuclear is not domestic; according to the DOE, America imports 86% of its nuclear fuel. With the risk and cost of building nuclear plants, plus the trade deficit, I would have thought that the fuel cost, just to heat water, would be a tenth of coal’s, or less? I was wrong! The World Nuclear Association states that nuclear fuel costs amount to a third of the same size coal-fired plant. Americans can’t afford to pay for nuclear electricity, just so uranium can be profitably mined, and weapons material created. The waste of heat that could be converted to electricity by thermocouple proves nuclear is just a money game. Also, we wouldn’t see the persistent defense by shills; nobody’s so ignorant that they don’t get these facts. Forget these pointless debates and write your representatives.

    Now the economic/energy strategy to replace nuclear and fossil: Build no new nuclear plants, phase out existing ones and over the next 30 years replace nuclear and fossil with solar, starting now. No waiting 40 years. No more “drawing board” solutions, thorium, fusion, pie in the sky, space-based solar, etc. Sorry, this is just human nature wanting to get something for nothing, or very cheaply. Economics 101: there is no free lunch. From here on history, fact, and what we know. Dr. John O. Blackburn and Sam Cunningham have shown that nuclear has already lost its cost advantage to solar, in a heavily-referenced paper for the nonprofit North Carolina Waste Awareness and Reduction Network. For them to make this claim, how could solar not be cheaper than nuclear everywhere soon? And especially in ten years, when new nuclear plants could first start adding power to our grid, if construction started today? If we can afford tax breaks for the richest and largest corporations and bailouts for the largest financial institutions, we can afford solar.

    Solar energy can meet all our energy needs, while being far safer than nuclear or coal. The Solar Energy Industries Association has factual, irrefutable information on the scale and speed possible for solar panel installation http://www.seia.org/cs/news_detail?pressrelease.id=342 The United States has more solar potential for a given area than Germany does, and Germany has installed a total of 12,000 MW of solar panels, and installed 3800 MW in 2009 alone. In 2010 alone the EU, with a comparable population to the US, installed 12,000 MW of solar, and 9300 MW of windpower, but only 145 MW of nuclear. The US installed less than a tenth of the solar the EU did in 2010.
    http://www.renewablepowernews.com/archives/2212
    Solar can be installed far faster than nuclear plants, which take 10 years or more to build one plant. There is no physical limit to how many megawatts of solar panels can be manufactured or installed, just political resistance, There is no excuse anymore. Are you going to try and tell me that the United States can’t do even better than Europe has? Are you going to tell me that the greatest, richest country in the world can’t afford power sources free of nuclear risk? We are falling behind the rest of the developed world, providing pork-barrel tax breaks to fossil and nuclear industries who have experienced large profits, and don’t care that we are falling far behind in renewables.

    Statements that solar can only meet a fraction of our electricity needs are based on past rates of installation and current use, projected linearly into the future, and are refuted by Europe’s example. In the same way, it could be predicted that nuclear energy will comprise 0% of our energy mix in decades, it the Atomic Energy Act amendments to the Constitution were repealed (which should be done if that’s what it takes to kick the nuclear habit).

    Statements that we need nuclear for baseload power are false. Solar plants with thermal energy storage are now being built. The American Hydrogen Association has information on the technology to store non-toxic, carbon-free liquid fuel, and there now exist hydrogen-powered cars, trucks and a tractor by Massey-Ferguson, which means we can have scalable generators fo nigttime generation. If current nuclear investments were put into solar, wind and storage technologies, we could build the fuel infrastructure for production, transport and fueling, like America did a century ago for gasoline. What is needed is political pressure to cut current government subsidies and tax breaks from nuclear (twice as much is spent on nuclear research and development than on solar) and fossil energy.

    The priority for siting utility-scale plants should be closed nuclear plants, (if the radiation could be totally contained with zero emissions, so as not to endanger the health of construction or maintenance workers) and coal plants, since the transmission lines are already in place. Contaminated nuclear sites, some quite large, are the obvious choice for waste storage, encased with concrete, covered with solar panels. Other good sites are abandoned strip mines, factories, Superfund sites, landfills, ore milling plants, south-facing dams. This type of of siting will remove the environmental objections to large-scale solar and the safety objections raised by nuclear defenders criticizing rooftop solar. But, we would only have to cover a third of our total building roof area to absorb enough energy to power our whole country. Of course excess capacity and storage of heat or hydrogen need to be installed, for generation during sunless times. As an engineer, I believe wind power should generate primarily hydrogen for transportation and nighttime baseload power (which would eliminate the intermittency problem of connecting wind to the grid). Generation of electricity from stored hydrogen, made with wind power, would work even in the Alaskan winter.

    More jobs will be created by renewables, in the manufacturing, installation, cleaning and security of large solar installations, than nuclear ever will. The tops of prison buildings would not be bad solar sites, since they have guarded expansive roofs, and could meet all their needs and supply their current load to the grid with existing power lines. Farms, food processing plants, warehouses, truck stops, schools, government buildings, malls could all be self-sufficient and supply their current load to the grid, and sooner than nuclear can be built.

Comments are closed.